22. november 1984 22 november 1984 22 novembre 1984...

71
Amtsblatt des Europäischen Patentamts 22. November 1984 Jahrgang 7 / Heft 11 Seiten 551-604 Official Journal of the European Patent Office 22 November 1984 Year 7 / Number 11 Pages 551-604 Journal officiel de l´Office européen des brevets 22 novembre 1984 7 e année / numéro 11 Pages 551-604 ENTSCHEIDUNGEN DER BESCHWERDEKAMMERN Entscheidung der Technischen Beschwerdekammer 3.5.1 vom 26. Juni 1984 T 161/82* Zusammensetzung der Kammer: Vorsitzender: G. Korsakoff Mitglieder: J. van Voorthuizen P. Ford Anmelderin: AMP Inc. Stichwort: Steckkontakt/AMP" EPÜ Artikel 54 (1), (2), 113 (1) "Neuheit," "zufällige Vorwegnahme" "Rechtliches Gehör," "Gelegenheit zur Stellungnahme" Leitsatz I. Nach Artikel 113 (1) EPÜ ist es nicht erforderlich, dem Anmelder wiederholt Gelegenheit zu geben, sich zu dem Vor- bringen der Prüfungsabteilung zu äußern, wenn die entscheidenden Einwände gegen die Erteilung des europäischen Patents bestehenbleiben. II. Hat eine Vorwegnahme insofern Zu- fallscharakter, als das in einer Vorver- öffentlichung Offenbarte zufällig unter einen zur Neuheitsprüfung anstehenden Anspruch fallen könnte, ohne daß jedoch eine gemeinsame technische Aufgabe vorliegt, so ist besonders sorgfältig gegeneinander abzuwägen, was billiger- weise als vom Anspruch erfaßt gelten kann und was tatsächlich aus der Vor- veröffentlichung hervorgeht. Sachverhalt und Anträge I. Die am 12. Juni 1980 eingereichte europäische Patentanmeldung Nr. DECISIONS OF THE BOARDS OF APPEAL Decisions of the Technical Board of Appeal 3.5.1 dated 26 June 1984 T 161/82* Composition of the Board: Chairman: G. Korsakoff Members: J. van Voorthuizen P. Ford Applicant: AMP Inc. Headword: Electrical contact/AMP EPC Articles 54 (1), (2), 113 (1) "Novelty," "anticipation of a chance nature" "Basis of decision," "opportunity to present comments" Headnote I. Article 113 (1) EPC does not require that the applicant be given a repeated opportunity to comment on the argu- mentation of the Examining Division so long as the decisive objections against the grant of the European patent remain the same. II. In cases where an anticipation is of a chance nature, in that what is disclosed in a prior document could accidentally fall within the wording of a claim to be examined for novelty without there being a common technical problem, a particularly careful comparison has to be made between what can fairly be con- sidered to fall within the wording of the claim and what is effectively shown in the document. Summary of Facts and Submissions I. European Patent Application No. 80 301 975.1 filed on 12 June 1980 DECISIONS DES CHAMBRES DE RECOURS Décision de la Chambre de recours technique 3.5.1 du 26 juin 1984 T 161/82* Composition de la Chambre: Président: G. Korsakoff Membres: J. van Voorthuizen P. Ford Demanderesse: AMP Inc. Référence: "Contact électrique/AMP" Articles 54 (1) et (2), 113 (1) de la CBE "Nouveauté," "antériorisation fortuite" "Motifs de la décision," "possibilité de prendre position" Sommaire I. L´article 113 (1) CBE n´exige pas que soit donnée aux demandeurs la possi- bilité de prendre à nouveau position sur l´argumentation de la Division d´examen, tant que les objections essentielles s´opposant à la délivrance du brevet européen restent les mêmes. II. Lorsque l´antériorisation de l´objet revendiqué est fortuite, en ce sens que ce qui est divulgué dans un document antérieur pourrait accidentellement se retrouver dans les termes d´une reven- dication soumise à l´examen relatif à la nouveauté, sans qu´on ait affaire à un problème technique commun, il faut établir une distinction particulièrement soigneuse entre ce qui peut être raison- nablement considéré comme se re- trouvant dans les termes de la revendi- cation et ce dont le document fait effec- tivement état. Exposé des faits et conclusions I. La demande de brevet européen n° 80 301 975.1 déposée le 12 juin 1980 * Übersetzung. * Official text * Traduction.

Upload: others

Post on 18-Oct-2019

0 views

Category:

Documents


0 download

TRANSCRIPT

Page 1: 22. November 1984 22 November 1984 22 novembre 1984 ...archive.epo.org/epo/pubs/oj1984/p549_604.pdf · tout Etat contractant doit, pour ce qui le concerne, excuser pour des motifs

Amtsblatt desEuropäischenPatentamts

22. November 1984Jahrgang 7 / Heft 11Seiten 551-604

Official Journalof the EuropeanPatent Office

22 November 1984Year 7 / Number 11Pages 551-604

Journal officielde l´Officeeuropéen desbrevets

22 novembre 19847e année / numéro 11Pages 551-604

ENTSCHEIDUNGEN DERBESCHWERDEKAMMERN

Entscheidung der TechnischenBeschwerdekammer 3.5.1 vom26. Juni 1984T 161/82*Zusammensetzung der Kammer:Vorsitzender: G. KorsakoffMitglieder: J. van Voorthuizen

P. Ford

Anmelderin: AMP Inc.

Stichwort: Steckkontakt/AMP"EPÜ Artikel 54 (1), (2), 113 (1)"Neuheit," "zufälligeVorwegnahme" "RechtlichesGehör," "Gelegenheit zurStellungnahme"

LeitsatzI. Nach Artikel 113 (1) EPÜ ist es nichterforderlich, dem Anmelder wiederholtGelegenheit zu geben, sich zu dem Vor-bringen der Prüfungsabteilung zu äußern,wenn die entscheidenden Einwändegegen die Erteilung des europäischenPatents bestehenbleiben.II. Hat eine Vorwegnahme insofern Zu-fallscharakter, als das in einer Vorver-öffentlichung Offenbarte zufällig untereinen zur Neuheitsprüfung anstehendenAnspruch fallen könnte, ohne daß jedocheine gemeinsame technische Aufgabevorliegt, so ist besonders sorgfältiggegeneinander abzuwägen, was billiger-weise als vom Anspruch erfaßt geltenkann und was tatsächlich aus der Vor-veröffentlichung hervorgeht.

Sachverhalt und Anträge

I. Die am 12. Juni 1980 eingereichteeuropäische Patentanmeldung Nr.

DECISIONS OF THEBOARDS OF APPEAL

Decisions of the TechnicalBoard of Appeal 3.5.1 dated26 June 1984T 161/82*Composition of the Board:

Chairman: G. KorsakoffMembers: J. van Voorthuizen

P. Ford

Applicant: AMP Inc.Headword: Electrical contact/AMPEPC Articles 54 (1), (2), 113 (1)

"Novelty," "anticipation of achance nature" "Basis ofdecision," "opportunity to presentcomments"

HeadnoteI. Article 113 (1) EPC does not requirethat the applicant be given a repeatedopportunity to comment on the argu-mentation of the Examining Division solong as the decisive objections againstthe grant of the European patent remainthe same.II. In cases where an anticipation is of achance nature, in that what is disclosedin a prior document could accidentallyfall within the wording of a claim to beexamined for novelty without therebeing a common technical problem, aparticularly careful comparison has to bemade between what can fairly be con-sidered to fall within the wording of theclaim and what is effectively shown inthe document.

Summary of Facts and SubmissionsI. European Patent Application No.80 301 975.1 filed on 12 June 1980

DECISIONS DES CHAMBRESDE RECOURS

Décision de la Chambrede recours technique 3.5.1 du26 juin 1984T 161/82*Composition de la Chambre:

Président: G. KorsakoffMembres: J. van Voorthuizen

P. Ford

Demanderesse: AMP Inc.Référence: "Contact électrique/AMP"Articles 54 (1) et (2), 113 (1) de la CBE"Nouveauté," "antériorisationfortuite" "Motifs de ladécision," "possibilité de prendreposition"

SommaireI. L´article 113 (1) CBE n´exige pas quesoit donnée aux demandeurs la possi-bilité de prendre à nouveau position surl´argumentation de la Division d´examen,tant que les objections essentielless´opposant à la délivrance du breveteuropéen restent les mêmes.II. Lorsque l´antériorisation de l´objetrevendiqué est fortuite, en ce sens quece qui est divulgué dans un documentantérieur pourrait accidentellement seretrouver dans les termes d´une reven-dication soumise à l´examen relatif à lanouveauté, sans qu´on ait affaire à unproblème technique commun, il fautétablir une distinction particulièrementsoigneuse entre ce qui peut être raison-nablement considéré comme se re-trouvant dans les termes de la revendi-cation et ce dont le document fait effec-tivement état.

Exposé des faits et conclusionsI. La demande de brevet européen n°80 301 975.1 déposée le 12 juin 1980

* Übersetzung. * Official text * Traduction.

Page 2: 22. November 1984 22 November 1984 22 novembre 1984 ...archive.epo.org/epo/pubs/oj1984/p549_604.pdf · tout Etat contractant doit, pour ce qui le concerne, excuser pour des motifs

80 301 975.1 (Veröffentlichungsnum-mer 0 021 730), die eine Priorität vom29. Juni 1979 (FR) in Anspruch nimmt,wurde mit Entscheidung der Prüfungs-abteilung des EPA vom 27. August 1982zurückgewiesen. Der Entscheidung lagder am 17. Mai 1982 eingereichte ein-zige Anspruch zugrunde.

II. Die Zurückweisung wurde damit be-gründet, daß der Gegenstand des An-spruchs im Hinblick auf die Druck-schrift DE-A-1 465 461 nicht neu sei.

III. Die Anmelderin legte gegen dieseEntscheidung am 21. September 1982Beschwerde ein. Die Begründung war inder Beschwerdeschrift enthalten. Die Be-schwerdegebühr war bereits am 14.September 1982 entrichtet worden.

IV. Die Anmelderin macht im wesent-lichen geltend, daß der aus der DE-Druckschrift bekannte Steckkontakt zumEinstecken eines Steckers in jeweilseinen von zwei im rechten Winkel zu-einander stehenden Schlitzen nichtgeeignet sei. Insbesondere würde derbetreffende Steckkontakt (wie bereitsin der Beschwerdeschrift erwähnt) keinebrauchbare elektrische Verbindung er-geben, da aus der Entgegenhaltung ein-deutig hervorgehe, daß der zweiteSchlitz parallel angeordnet sei. Außer-dem lege keine der genannten Ent-gegenhaltungen eine Änderung nahe,um diesen Schlitz für die Aufnahmeeines Steckers geeignet zu machen.

V. Die Anmelderin reichte zusammen mitder Beschwerdeschrift und der Be-gründung "im Hinblick auf das der Zu-rückweisungsentscheidung zugrunde lie-gende Argument" einen geänderten An-spruch ein. Sie beantragte die Erteilungeines europäischen Patents auf derGrundlage dieses Anspruchs, der wiefolgt lautet:"Elektrischer Steckkontakt mit einemSteckabschnitt (2) mit zwei Paaren sichgegenüberliegender Kontaktarme (22),die sich alle in dieselbe Richtung er-strecken, wobei jeder Arm (22) jedesPaares einen vorspringenden Kontakt-punkt (23) aufweist, der einem ähn-lichen Punkt (23) auf dem gegenüber-liegenden Arm (22) zugewandt ist unddie beiden sich gegenüberliegendenArmpaare (22) einen Einsteckschlitz füreinen Stecker bilden, dadurch gekenn-zeichnet, daß jeder Arm (22) jedesPaares einen zweiten Kontaktvor-sprung (24) aufweist, der einem ähn-lichen Vorsprung (24) auf dem benach-barten Arm (22) des anderen Paareszugewandt ist, wobei die beiden Paareder zweiten Kontaktvorsprünge (24)einen zweiten Einsteckschlitz fürStecker bilden, der im rechten Winkelzu dem ersten Einsteckschlitz ange-ordnet ist."

VI. Die Anmelderin behauptet, daß diePrüfungsabteilung von "technischenGegebenheiten" ausgegangen sei, diekeine seien, und daß ihr vorher keine

(Publication No. 0021 730) claiming apriority of 29 June 1979 (FR), wasrefused by a decision of the ExaminingDivision of the EPO of 27 August 1982.That decision was based on the singleclaim filed on 17 May 1982.

II. The reason given for the refusal wasthat the subject-matter of the claimlacked novelty having regard to DE-A-1 465 461.

III. The applicant lodged an appealagainst this decision on 21 September1982. In the Notice of Appeal thegrounds for the appeal were set out. Theappeal fee was paid on 14 September1982.

IV. The applicant argues essentiallythat the contact known from the DEspecification is not suitable for receivinga male contact in either of two mutuallyperpendicular orientations. In particular(as was pointed out for the first time inthe Notice of Appeal) the contact inquestion would not provide an accept-able electric connection as it was clearlyshown in this document that the secondslot is parallel sided. Furthermore thereis no suggestion in any of the citeddocuments which would suggest amodification to make this slot suitablefor receiving a male contact.

V. With the Notice and Grounds ofAppeal the applicant submitted anamended claim, "in view of the argu-ment relied on in the decision to refuse".He requested that a European patentbe granted on the basis of this claimwhich reads as follows:

"An electrical contact having a contactportion (2) comprising two pairs ofopposed cantilever contact arms (22) allextending in the same direction, eacharm (22) of each pair presenting a con-tact projection (23) to a similar projec-tion (23) on the opposed arm (22), thetwo pairs of opposed arms (22) defininga common male-contact-receiving slot,characterised in that each arm (22) ofeach pair presents a second contactprojection (24) of a similar projection(24) on an adjacent arm (22) of the otherpair, the two pairs of second contactprojections (24) defining a second male-contact-receiving slot which is perpen-dicular to the first such slot".

VI. The applicant alleges that certain"technical facts" relied on by theExamining Division are not facts and thathe was not previously given an oppor-

et publiée sous le numéro 0 021 730,pour laquelle est revendiquée la prioritéd´une demande antérieure française du29 juin 1979, a été rejetée pardécision de la Division d´examen del´Office européen des brevets en datedu 27 août 1982. Cette décision a étérendue sur la base d´une revendicationunique, déposée le 17 mai 1982.

II. La demande a été rejetée pour défautde nouveauté de l´objet de la revendica-tion par rapport au document DE-A-1 465 461.

III. La demanderesse a introduit unrecours contre cette décision le 21 sep-tembre 1982 et a exposé les motifs durecours dans son mémoire joint àl´acte de recours. La taxe de recours aété payée le 14 septembre 1982.

IV. La requérante allègue essentielle-ment que le contact connu d´après ledocument allemand ne convient pas pourrecevoir un contact mâle suivant l´uneou l´autre de deux orientations mutuelle-ment perpendiculaires. En particulier(comme cela a été indiqué pour la pre-mière fois dans l´acte de recours), lecontact en question ne permettrait pasd´établir une connexion électriqueacceptable, la seconde encoche étantà faces parallèles, ce que montre claire-ment ce document. En outre, aucun desdocuments cités ne contient d´élémentqui suggérerait une modification en vuede rendre cette encoche apte à recevoirun contact mâle.

V. Simultanément à l´acte de recours etau mémoire en exposant les motifs, larequérante a soumis à la Chambre unerevendication modifiée "compte tenudes motifs de la décision de rejet". Ellea demandé la délivrance d´un breveteuropéen sur la base de cette revendica-tion, qui s´énonce comme suit:

"Contact électrique possédant unepartie de contact (2) comprenant deuxpaires de bras de contact opposés enporte-à-faux (22) qui s´étendent tousdans la même direction, chaque bras(22) de chaque paire présentant unesurface convexe de contact (23) tournéevers une surface convexe identique (23)située sur le bras opposé (22), les deuxpaires de bras opposés (22) délimitantune encoche commune destinée àrecevoir un contact mâle, caractériséen ce que chaque bras (22) de chaquepaire présente une seconde surface con-vexe de contact (24) correspondant àune surface convexe identique (24) surun bras voisin (22) de l´autre paire, lesdeux paires des secondes surfaces con-vexes de contact (24) délimitant uneseconde encoche qui est destinée àrecevoir un contact mâle et est perpen-diculaire à la première encoche".

VI. La requérante allègue que certains"faits techniques" sur lesquels s´étaitfondée la Division d´examen ne corres-pondent pas à la réalité et qu´on ne lui

* Übersetzung. * Official text. * Traduction.

Page 3: 22. November 1984 22 November 1984 22 novembre 1984 ...archive.epo.org/epo/pubs/oj1984/p549_604.pdf · tout Etat contractant doit, pour ce qui le concerne, excuser pour des motifs

Gelegenheit gegeben worden sei, sichhierzu zu äußern. Sie machte geltend,daß Artikel 113 (1) EPÜ nicht erfülltworden sei, und beantragte die Rück-zahlung der Beschwerdegebühr.

Entscheidungsgründe1. Die Beschwerde entspricht den Arti-keln 106 bis 108 und Regel 64 EPÜ; sieist somit zulässig.

2. Die Prüfungsabteilung hat ihre Ent-scheidung auf die Lehre der DruckschriftDE-A-1 465 461 gestützt. Die Abbil-dungen 1 bis 3 dieser Druckschriftzeigen einen elektrischen Steckkontakt,der mit dem elektrischen Kontakt nachdem am 17. Mai 1982 eingereichteneinzigen Anspruch der vorliegendenAnmeldung Ähnlichkeit aufweist undeinen im rechten Winkel zu dem erstenEinsteckschlitz angeordneten zweitenSchlitz enthält. Der erste Schlitz isteindeutig zur Aufnahme eines Steckersgedacht; es gilt nun die entscheidendeFrage zu klären, ob auch der zweiteSchlitz ein "Einsteckschlitz für Stecker"ist, d. h. ob es für den Fachmann offen-sichtlich gewesen wäre, daß dieserSchlitz diesem Zweck dienen könne.

3. Bei der Beuteilung dieser Frage ist zubeachten, daß es bei der DE-Druckschriftum die Lösung einer ganz anderen tech-nischen Aufgabe geht als bei der vor-liegenden Anmeldung, nämlich darum,einen Steckkontakt bereitzustellen, inden sowohl ein Stiftstecker als aucheine Kontaktplatte eingesteckt werdenkönnen.

4. Hat eine Vorwegnahme insofern Zu-fallscharakter, als das in einer Vorveröf-fentlichung Offenbarte zufällig untereinen zur Neuheitsprüfung anstehendenAnspruch fallen könnte, ohne daß jedocheine gemeinsame technische Aufgabevorliegt, so ist besonders sorgfältiggegeneinander abzuwägen, was billi-gerweise als vom Anspruch erfaßt geltenkann und was tatsächlich aus der Vor-veröffentlichung hervorgeht.

5. Dieser Vergleich zeigt im vorliegendenFall, daß der Steckkontakt nach DE-A-1 465 461 keine sich nach außen erwei-ternde oder sonstwie geformte Führungam zweiten Schlitz aufweist und daß die"Kontaktflächen" in ihrer gesamtenLänge genau parallel zueinander ver-laufen. Dies schließt praktisch aus, daßder zweite Schlitz als "Einsteckschlitz"betrachtet werden kann. Dementsprechendist die Kammer der Auffassung, daß derSteckkontakt nach dem der Prüfungs-abteilung vorgelegten Anspruch gegen-über der entgegengehaltenen DE-Schriftals neu anzusehen ist.

tunity to comment thereon. He sub-mitted that the requirements of Article113 (1) EPC had not been met andrequested a refund of the appeal fee.

Reasons for the Decision

1. The appeal complies with Articles 106to 108 and Rule 64 EPC and is thereforeadmissible.

2. The Examining Division has based itsdecision on the disclosure in DE-A-1 465 461. In Fig. 1 to 3 of this docu-ment an electrical contact is shownwhich bears a resemblance to the elec-trical contact described in the singleclaim of the present application filed on17 May 1982 and which comprises asecond slot perpendicular to a first one.It is clear that the first slot is intendedto receive a male contact but the essen-tial question to be examined is whetherthe second slot also consitutes a "malecontact receiving slot" i.e. whether itwould be evident to a person skilled inthe art that the slot could serve thispurpose.

3. In considering this question it has tobe borne in mind that the DE documentis concerned with the solution of a prob-lem totally different from that stated inthe present application, viz to providea female contact in which either a roundor a flat male contact can be inserted.

4. In particular, in cases where theanticipation is of a chance nature in thatwhat is disclosed in a prior documentcould accidentally fall within the wordingof a claim to, be examined for noveltywithout there being a common tech-nical problem, a particularly careful com-parison has to be made between whatcan fairly be considered to fall withinthe wording of the claim and what iseffectively shown in the document.

5. When carrying out the comparisonin the present case it is found that in thecontact shown in DE-A-1 465 461 noflared or otherwise formed lead-in por-tion for the second slot is present andthat the "contact surfaces" are strictlyparallel sided over their entire longitu-dinal dimension. This practically excludesthe possibility of regarding the secondslot as "male contact receiving". Accord-ingly, the Board is of the opinion thatthe contact according to the claimbefore the Examining Division has to beregarded as novel over the cited DEspecification.

avait pas donné préalablement la possi-bilité de prendre position à ce sujet.Selon elle, les exigences de l´article113 (1) CBE n´ont pas été satisfaites,et elle demande le remboursement de lataxe de recours.

Motifs de la décision

1. Le recours répond aux conditionsénoncées aux articles 106, 107 et 108et à la règle 64 CBE; il est doncrecevable.

2. La Division d´examen a fondé sa déci-sion sur la divulgation constituée par ledocument DE-A-1 465 461, dont lesfigures 1 à 3 représentent un contactélectrique ayant une ressemblance avecle contact électrique décrit dans l´uniquerevendication qui a été déposée le 17mai 1982 pour la présente demande etcomportant une seconde encoche per-pendiculaire à une première encoche.S´il est vrai que la première encochedivulguée dans ledit document est mani-festement conçue pour loger un contactmâle, il faut néanmoins se poser laquestion fondamentale de savoir si laseconde encoche constitue égalementune "encoche destinée à recevoir uncontact mâle", c´est-à-dire s´il seraitévident, pour l´homme du métier, quecette encoche pourrait être utilisée àcette fin.3. En examinant cette question, il con-vient de tenir compte du fait que ledocument allemand concerne la solutiond´un problème, à savoir fournir un con-tact femelle dans lequel il soit possibled´insérer un contact mâle aussi bienrond que plat, qui est totalement dif-férent du problème indiqué dans la pré-sente demande.

4. On notera en particulier que, dans lecas où l´antériorisation de l´objet reven-diqué est fortuite, en ce sens que ce quiest divulgué dans un document antérieurpourrait accidentellement se retrouverdans les termes d´une revendication sou-mise à l´examen relatif à la nouveauté,sans qu´on ait affaire à un problèmetechnique commun, il faut établir unedistinction particulièrement soigneuseentre ce qui peut être raisonnable-ment considéré comme se retrouvantdans les termes de la revendication etce dont le document fait effectivementétat.

5. Si l´on s´emploie à établir une telledistinction dans le cas d´espèce, on voitque le contact représenté dans le docu-ment DE-A-1 465 461 ne comporte, pource qui est de la seconde encoche,aucune partie d´entrée évasée ou deforme particulière et que les "surfacesde contact" sont à faces strictementparallèles sur toute leur longueur. Ceciexclut pratiquement la possibilité deconsidérer la seconde encoche comme"destinée à recevoir un contact mâle".Par conséquent, la Chambre est d´avisque le contact selon la revendication surla base de laquelle la Division d´examena rendu sa décision doit être considérécomme nouveau par rapport au docu-ment allemand cité.

* Übersetzung. * Official text. * Traduction.

Page 4: 22. November 1984 22 November 1984 22 novembre 1984 ...archive.epo.org/epo/pubs/oj1984/p549_604.pdf · tout Etat contractant doit, pour ce qui le concerne, excuser pour des motifs

6. Dies gilt erst recht für den Steckkon-takt nach dem geänderten Anspruch, derzusammen mit der Beschwerdeschriftund der Begründung eingereicht wurde;in diesem Anspruch heißt es nunmehr,daß "Kontaktvorsprünge" vorhandensind, die bei dem Steckkontakt nach derDE-Druckschrift eindeutig nicht vor-kommen.

7. Obwohl der Steckkontakt nach demgeänderten Anspruch neu ist, bleibt zuprüfen, ob der Anspruch die übrigenErfordernisse des EPÜ erfüllt und ob erinsbesondere eine erfinderische Tätigkeitaufweist. Im Verfahren vor der Prüfungs-abteilung ist dieser Aspekt im Zusam-menhang mit dem Gegenstand desdamals gültigen Anspruchs überhauptnoch nicht geprüft worden. Um der An-melderin nicht das Recht auf eine Prü-fung durch zwei Instanzen zu nehmen,hält es die Kammer für angebracht,die Anmeldung zur Weiterbearbeitungan die Prüfungsabteilung zurückzuver-weisen.

8. Die Anmelderin behauptet auf Seite3, Absatz 4 der Beschwerdeschrift, daßArtikel 113 (1) EPÜ nicht eingehaltenworden sei und somit ein wesentlicherVerfahrensfehler der Prüfungsabteilungvorliege (R. 67 EPU). Die Kammer ist derAnsicht, daß unterschieden werden mußzwischen den Gründen, auf die eineEntscheidung gestützt ist (d. h. dieErfordernisse des EPÜ, die von derAnmeldung oder der Erfindung, die siezum Gegenstand hat, nicht erfüllt wer-den), und der Begründung, in der aus-führlich dargelegt wird, warum die Prü-fungsabteilung vom Vorliegen solcherGründe überzeugt ist (vgl. R. 51 (3)EPÜ).

9. Als Grund für die Zurückweisungwurde im vorliegenden Fall mangelndeNeuheit des Anspruchsgegenstandsangegeben; dieser Einwand wurde indem ersten Bescheid des beauftragtenPrüfers vom 19. Februar 1982 erhobenund ausführlich begründet. Die Anmel-derin äußerte sich hierzu in einemSchreiben vom 17. Mai 1982 und legteeinen geänderten einzigen Anspruch vor,der sich von dem früheren Anspruch 1dadurch unterschied, daß sie zweiweitere Merkmale hinzugefügt hatte,mit denen sie die Erfindung besser vomStand der Technik abgrenzen wollte. Inihren Ausführungen bemerkte die An-melderin lediglich, daß die vom Prüferfür möglich gehaltene Verwendung desSteckkontakts nach DE-A-1 465 461 aufeiner im nachhinein gewonnenen Er-kenntnis beruhe und vom Fachmannnicht in Betracht gezogen worden wäre,da der Steckkontakt nach Abbildung 4und 5 eindeutig nicht in dieser Weiseverwendet werden könne.

10. Die Prüfungsabteilung war der Auf-fassung, daß die beiden zusätzlichenMerkmale dem Fachmann bereits durchdie DE-Entgegenhaltung implizit offen-

6. This is true a fortiori for the contactaccording to the amended claim sub-mitted with the Notice and Grounds ofAppeal, in which it is now specifiedthat "contact projections" are provided,which are definitely not present in thecontact shown in the DE-specification.

7. Although the contact according to thisamended claim is novel, it still has to beexamined whether the claim satisfies theother requirements of the EPC, in par-ticular whether inventive step is involved.In the procedure before the ExaminingDivision this aspect has not been con-sidered at all as far as the subject-matterof the then valid claim was concerned.In order not to deprive the applicant ofhis right to an examination in twoinstances the Board deems it appropriateto remit the application to the ExaminingDivision for further prosecution.

8. The applicants contended in para-graph 4 on page 3 of the Notice ofAppeal, that the requirements of Article113 (1) EPC have not been met, sothat there had been a substantial proce-dural violation by the Examining Division(Rule 67 EPC). The Board considersthat a distinction has to be made be-tween the grounds on which a deci-sion is based (i.e. the requirements of theEPC which are not satisfied by the appli-cation or the invention to which itrelates) and the reasoned statementexplaining in greater detail why theExamining Division is of the opinion thatsuch grounds exist (cf. Rule 51 (3) EPC).

9. The ground for rejection in the presentcase being lack of novelty in the subject-matter of the claim it is to be noted thatthis objection was raised and amplyargued in the first communication by theprimary examiner of 19 February 1982.The applicant commented thereon in aletter of 17 May 1982 and submitted anamended single claim differing fromthe previous claim 1 by the additionof two further features in order, as theapplicant stated, to distinguish the in-vention more clearly from the prior art.In his comments the applicant merelyobserved that the Examiner´s suggesteduse of the contact according to DE-A-1 465 461 was based on hindsight andthat this use would not have beenconsidered since the contact of Fig. 4and 5 clearly cannot be used as sug-gested.

10. The Examining Division consideredthat the two added features were alreadyimplicitly disclosed to the person skilledin the art by the DE specification. The

6. Les conditions de la nouveauté sontremplies a fortiori pour le contact selonla revendication modifiée qui a été sou-mise à la Chambre simultanément àl´acte de recours et au mémoire expo-sant les motifs du recours, revendica-tion qui spécifie maintenant l´existencede "surfaces convexes de contact", dontl´absence sur le contact représenté dansle document allemand est incontestable.

7. Bien que le contact selon cette reven-dication modifiée soit nouveau, il reste àexaminer si la revendication satisfait auxautres exigences de la CBE, en particuliercelle de l´implication d´une activité in-ventive. Dans la procédure devant laDivision d´examen, cet aspect n´a pas dutout été envisagé, en ce qui concernaitl´objet de la revendication alors valable.Afin de ne pas priver la demanderesse deson droit à l´examen devant deuxinstances, la Chambre estime appropriéde renvoyer la demande à la Divisiond´examen pour la poursuite de la procé-dure.

8. La requérante conclut à la page 3, 4e

alinéa de l´acte de recours que les exi-gences de l´article 113 (1) CBE n´ontpas été satisfaites, si bien qu´il yaurait eu un vice substantiel dans la pro-cédure devant la Division d´examen(règle 67 CBE). La Chambre estime qu´ilfaut faire la différence entre les motifsservant de fondement à une décision(c´est-à-dire le non-respect, par la de-mande ou par l´invention qui en faitl´objet, des conditions de la CBE) et l´ex-posé motivé expliquant d´une manièredétaillée pourquoi la Division d´examenest d´avis que de tels motifs existent[cf. règle 51 (3) CBE].

9. Dans le cas présent, le motif du rejetrésidait dans l´absence de nouveautéde l´objet de la revendication, et il y alieu d´observer que l´examinateur ins-truisant la demande a soulevé à cetégard une objection qu´il a amplementdéveloppée dans sa première notifica-tion en date du 19 février 1982. La de-manderesse y a répondu dans une lettredatée du 17 mai 1982 et a soumis unerevendication unique modifiée, différantde la revendication précédente 1 parl´adjonction de deux nouvelles carac-téristiques, afin, comme elle l´indiquaitdans sa lettre, d´établir une distinctionplus claire entre l´invention et l´état dela technique. Dans sa prise de position,elle a fait simplement observer que l´uti-lisation, telle qu´envisagée par l´exami-nateur, du contact selon le documentDE-A-1 465 461 découlait d´une dé-marche rétrospective et que l´on n´auraitpas songé à cette utilisation étant donnéque le contact représenté aux figures 4et 5 ne pouvait manifestement pas s´yprêter.

10. La Division d´examen a considéréque les deux caractéristiques ajoutéesétaient, pour l´homme du métier, déjàimplicitement divulguées dans le docu-

* Übersetzung. * Official text. * Traduction.

Page 5: 22. November 1984 22 November 1984 22 novembre 1984 ...archive.epo.org/epo/pubs/oj1984/p549_604.pdf · tout Etat contractant doit, pour ce qui le concerne, excuser pour des motifs

bart worden seien. Da die Prüfungs-abteilung die Argumente der Anmel-derin nicht überzeugend fand und zuder Feststellung gelangte, daß gegenden geänderten Anspruch noch immerderselbe Einwand bestand, traf sie dieangefochtene Entscheidung. Die Ent-scheidung enthielt natürlich einige zu-sätzliche Argumente zu den dem An-spruch hinzugefügten Merkmalen.

11. Die Beschwerdekammer vertritt denStandpunkt, daß Artikel 113 (1) nichtvorschreibt, daß dem Anmelder wieder-holt Gelegenheit gegeben werden muß,sich zu dem Vorbringen der Prüfungs-abteilung zu äußern, wenn die entschei-denden Einwände gegen die Erteilungdes europäischen Patents bestehen-bleiben.

12. Unter den gegebenen Umständen istdie Kammer daher der Auffassung, daßkein Verstoß gegen Artikel 113 (1)EPÜ vorliegt.

Aus diesen Gründen

wird wie folgt entschieden:1. Die angefochtene Entscheidung wirdaufgehoben.

2. Die Sache wird an die Prüfungsab-teilung zur Weiterbearbeitung auf derGrundlage der Beschreibung und desgeänderten Anspruchs zurückverwiesen,der am 21. September 1982 zusammenmit der Beschwerdeschrift und -begrün-dung eingereicht worden ist.

3. Der Antrag der Anmelderin auf Rück-zahlung der Beschwerdegebühr wirdzurückgewiesen.

* Übersetzung.

Entscheidung der TechnischenBeschwerdekammer 3.3.1. vom30. Juli 1984T 188/83*

Zusammensetzung der Kammer:Vorsitzender: D. CadmanMitglieder: K. Jahn

O. Bossung

Anmelder: FernholzStichwort: "Vinylacetat/FERNHOLZ"

EPÜ Artikel 54, 123

"Neuheit (verneint)" "punktförmiger Disclaimer" "unzulässige Abänderung"

Leitsatz

I. Wird bei einem vorbeschriebenenchemischen Herstellungsverfahren einbestimmtes als Bereich definiertes Ver-hältnis von Reaktionspartnern ausge-wählt, das von der bekannten Lehrezwar umfaßt, aber nicht genannt ist, sokann hierin eine neue Erfindung liegen.

Division not being convinced by the appli-cants´ arguments and having found thatthe same objection as before stillexisted against the amended claim there-upon issued the decision under appeal.This decision contained as a matter ofcourse some supplementary argumentsconcerning the features added to theclaim.

11. The Board of Appeal takes the viewthat Article 113 (1) does not requirethat the applicant be given a repeatedopportunity to comment on the argu-mentation of the Examining Division solong as the decisive objections againstthe grant of the European patentremain the same.

12. In the circumstances of the presentcase, therefore, the Board considers thatthere was no contravention of Article113 (1) EPC.

For these reasons

it is decided that:1. The decision under appeal is set aside.

2. The case is remitted to the ExaminingDivision for further prosecution on thebasis of the description and claim asamended in accordance with the appli-cants´ Notice and Grounds of the Appealfiled on 21 September 1982.

3. The applicants´ request for refund ofthe appeal fee is dismissed.

* Official text.

Decision of the Technical Boardof Appeal 3.3.1. dated30 July 1984T 188/83*

Composition of the Board:

Chairman: D. CadmanMembers: K. Jahn

0. Bossung

Applicant: Fernholz

Headword: "Vinyl acetate/FERNHOLZ"

EPC Articles 54, 123

"Novelty (denied)" "Disclaimer ofindividual values" "Non-permissibleamendments"

Headnote

I. If for the purposes of a chemical pro-duction process previously described acertain ratio of reactants, defined interms of a range, is chosen, the saidratio being covered by the conventionalteaching but not mentioned in it, thismay involve a new invention. The novelty

ment allemand. C´est alors que, nonconvaincue par les arguments de la de-manderesse et constatant que la mêmeobjection que précédemment existaitencore à l´encontre de la revendicationmodifiée, elle a rendu la décision, objetdu recours. Cette décision contenait toutnaturellement un supplément d´argu-mentation concernant la question descaractéristiques ajoutées à la revendica-tion.

11. La Chambre de recours est d´avisque l´article 113 (1) n´exige pas que soitdonnée aux demandeurs la possibilitéde prendre à nouveau position sur l´argu-mentation de la Division d´examen, tantque les objections essentielles s´oppo-sant à la délivrance du brevet euro-péen restent les mêmes.

12. Vu les faits de la cause, la Chambreconsidère qu´il n´y a pas eu infractionà l´article 113 (1) CBE.

Par ces motifs,

il est statué comme suit:

1. La décision attaquée est annulée.

2. L´affaire est renvoyée à la Divisiond´examen pour la poursuite de la pro-cédure sur la base de la description etde la revendication comportant la modi-fication soumise à la Chambre en mêmetemps que l´acte de recours et lemémoire en exposant les motifs, dé-posés le 21 septembre 1982 par larequérante.

3. La requête en remboursement de lataxe de recours est rejetée.

* Traduction.

Décision de la Chambre derecours technique 3.3.1 du30 juillet 1984T 188/83*

Composition de la Chambre:

Président: D. CadmanMembres: K. Jahn

O. Bossung

Requérant: FernholzRéférence: "Acétate devinyle/FERNHOLZ"

Articles 54 et 123 de la CBE"Nouveauté (absence de)" "Disclaimer ponctuel" "Modification inadmissible"

SommaireI. Il peut y avoir une invention nouvelledans le fait de sélectionner dans un pro-cédé de préparation chimique précé-demment décrit un rapport déterminéentre des partenaires réactionnels, définisous la forme d´un domaine, lorsque cerapport, bien qu´englobé par l´enseigne-

* Amtlicher Text. * Translation. * Traduction.

Page 6: 22. November 1984 22 November 1984 22 novembre 1984 ...archive.epo.org/epo/pubs/oj1984/p549_604.pdf · tout Etat contractant doit, pour ce qui le concerne, excuser pour des motifs

Die Neuheit dieses Bereichs wird zer-stört, wenn die Vorbeschreibung Bei-spiele enthält, die in diesem Bereichliegen. Der Bereich wird nicht dadurchneu, daß die aus den Beispielen errech-neten Werte durch Disclaimer ausge-nommen werden, jedenfalls dann nicht,wenn diese Werte aufgrund der brei-teren vorbekannten Lehre im Lichtedes allgemeinen Fachwissens nichtpunktförmig zu sehen sind.

II. Ein solches Herstellungsverfahrenwird auch nicht dadurch neu, daß aufeinen damit verbundenen, aber von derFachwelt bislang nicht erkannten Ver-fahrensvorteil hingewiesen wird, der beiunveränderter Ausführung des Verfah-rens eintritt.

Sachverhalt und AnträgeI. Die am 17. Dezember 1979 ange-meldete und am 9. Juli 1980 veröffent-lichte Patentanmeldung 79 105 213.7mit der Veröffentlichungsnummer0012 968, für welche die Priorität derVoranmeldung in der BundesrepublikDeutschland vom 21. Dezember 1978in Anspruch genommen wird, wurdedurch die Entscheidung der Prüfungs-abteilung des Europäischen Patentamtsvom 24. Juni 1983 zurückgewiesen. DerEntscheidung liegen die mit der Eingabevom 14. Januar 1983 eingereichten 4Patentansprüche zugrunde, von denendie Ansprüche 1 und 3 folgenden Wort-laut haben:"1. Verfahren zur Herstellung von Vinyl-acetat in der Gasphase durch Umsetzungvon Ethylen, Essigsäure und moleku-larem Sauerstoff an Trägerkatalysatoren,die eine Palladiumverbindung, eine Cad-miumverbindung und ein Alkaliacetatenthalten, dadurch gekennzeichnet, daßdas molare Verhältnis von Essigsäure zuSauerstoff im eingesetzten Gasgemischzwischen 2,2:1 und 3,5:1 liegt, wobeidie molaren Verhältnisse 2,762:1,2,68:1 und 2,34:1 ausgenommen sind.

3. Verfahren zur Herstellung von Vinyl-acetat in der Gasphase durch Umsetzungvon Ethylen, Essigsäure und moleku-larem Sauerstoff an Trägerkatalysatoren,die elementares Palladium, elementaresGold und ein Alkaliacetat enthalten,dadurch gekennzeichnet, daß das molareVerhältnis von Essigsäure zu Sauerstoffim eingesetzten Gasgemisch zwischen2,2:1 und 3,5:1 liegt, wobei die molarenVerhältnisse 3,30:1, 3,28:1, 3,19:1,3,11:1, 2,98:1, 2,762:1 ausgenommensind."

II. Die Zurückweisung wird mit man-gelnder erfinderischer Tätigkeit be-gründet. Aufgabe des anmeldungs-gemäßen Verfahrens sei es, das be-kannte Verfahren zur Herstellung vonVinylacetat so zu gestalten, daß Vinyl-acetat in relativ hoher und gleichblei-bender Raumzeitausbeute hergestelltwird, ohne daß exotherme Nebenreak-

of this range is destroyed if the previousdescription contains examples whichfall within it. The range is not renderednovel by the fact that the values cal-culated from the examples are exceptedby means of a disclaimer, at least not ifthese values cannot be regarded as indi-vidual on the basis of the broader pre-viously known teaching in the light ofgeneral knowledge of the art.

II. Moreover a production process of thiskind is not rendered novel by the factthat reference is made to an advantageassociated therewith which has hithertonot been recognised by those skilled inthe art and which takes effect withoutthe process having been modified.

Summary of Facts and Submissions

I. Patent application No. 79 105 213.7filed on 17 December 1979 and pub-lished on 9 July 1980 (publication No.0012 968) claiming priority of the earlierapplication in the Federal Republic ofGermany of 21 December 1978 wasrefused by a decision of the ExaminingDivision of the European Patent Office of24 June 1983. The decision was basedon the four claims as filed on 14 Jan-uary 1983, claims 1 and 3 of whichread as follows:

"1. A method of producing vinyl acetatein the gaseous phase by reacting ethylene,acetic acid and molecular oxygen in thepresence of supported catalysts con-taining a palladium compound, a cad-mium compound and an alkali acetate,characterised in that the molar ratio ofacetic acid to oxygen in the gas mixtureused is between 2.2:1 and 3.5:1, themolar ratios 2.762:1, 2.68:1 and 2.34:1being excepted.

3. A method of producing vinyl acetatein the gaseous phase by reacting ethyl-ene, acetic acid and molecular oxygenin the presence of supported catalystscontaining elemental palladium, elemen-tal gold and an alkali acetate, charac-terised in that the molar ratio of aceticacid to oxygen in the gas mixture used isbetween 2.2:1 and 3.5:1, the molarratios 3.30:1, 3.28:1, 3.19:1, 3.11:1,2.98:1 and 2.762:1 being excepted."

II. The reason given for refusal was lackof inventive step. The object of themethod according to the application wasto design the conventional method forproducing vinyl acetate so as to enablea relatively high and constant space-time yield of vinyl acetate to be pro-duced without the occurrence of secon-dary exothermal reactions leading to

ment connu, n´y est pas expressémentmentionné. La nouveauté de ce domaineest toutefois détruite dès lors que ladescription antérieure contient desexemples qui se situent dans ce mêmedomaine. Celui-ci n´est pas rendu nou-veau par le fait que les valeurs calculéesà partir desdits exemples sont excluespar disclaimer, en tous cas pas lorsqueces valeurs ne peuvent être considéréescomme ponctuelles, compte tenu del´enseignement plus large déjà divulguéet de l´interprétation que peut normale-

II. Ne contribue pas non plus à la nou-veauté d´un tel procédé de fabricationle fait de mettre en relief un avantageopératoire qui y est lié mais qui n´avaitencore jamais auparavant été reconnupar les spécialistes, et qui se manifestelors de la réalisation du procédé nonmodifié.

Exposé des faits et conclusions

I. La demande de brevet n° 79 105 213.7,déposée le 17 décembre 1979 et pu-bliée le 9 juillet 1980 sous le numéro0012 968, pour laquelle est revendiquéeune priorité du 21 décembre 1978 fon-dée sur un dépôt antérieur en Répub-lique fédérale d´Allemagne, a été rejetéepar décision du 24 juin 1983 de la Divi-sion d´examen de l´Office européen desbrevets. Cette décision a été rendue surla base des 4 revendications déposéesavec la correspondance du 14 janvier1983. Les revendications 1 et 3 s´énon-cent comme suit:

"1. Procédé pour la préparation d´acé-tate de vinyle en phase gazeuse parréaction d´éthylène, d´acide acétique etd´oxygène moléculaire sur des cataly-seurs déposés sur un support et conte-nant un dérivé de palladium, un dérivéde cadmium et un acétate alcalin, carac-térisé en ce que le rapport molaire del´acide acétique à l´oxygène dans lemélange gazeux introduit se situe entre2,2:1 et 3,5:1, les rapports molaires2,762:1,2,68:1 et 2,34:1 étant exclus.""3. Procédé pour la préparation d´acé-tate de vinyle en phase gazeuse parréaction d´éthylène, d´acide acétique etd´oxygène moléculaire sur des cataly-seurs déposés sur un support et conte-nant du palladium élémentaire, de l´orélémentaire et un acétate alcalin, carac-térisé en ce que le rapport molaire del´acide acétique à l´oxygène dans lemélange gazeux introduit se situe entre2,2:1 et 3,5:1, les rapports molaires3,30:1, 3,28:1, 3,19:1, 3,11:1, 2,98:1,2,762:1 étant exclus."

II. Le rejet a été motivé par l´absenced´activité inventive. L´objet du procédéselon la demande consiste à adapter leprocédé connu de préparation de l´acé-tate de vinyle de telle sorte qu´on ob-tienne l´acétate de vinyle avec desrendements par unité de volume et detemps relativement élevés et constants,sans que se produisent des réactions

* Amtlicher Text. * Translation. * Traduction.

ment lui donner l homme du m tier.é

´

´

Page 7: 22. November 1984 22 November 1984 22 novembre 1984 ...archive.epo.org/epo/pubs/oj1984/p549_604.pdf · tout Etat contractant doit, pour ce qui le concerne, excuser pour des motifs

tionen stattfinden, die zu höheremAbbrand, Leistungsabfall und Schädi-gung des Katalysators führen. DieseAufgabe müsse bereits als grundsätz-lich gelöst angesehen werden; hierzuwird auf US A-3 759 839 (B), DE-C-2 315 037 (D), US A-3 743 607 (E),GB-A-1 189 091 (F), US-A-3 775 342 (G)und DE-C 2 509 251 (J), irrtümlich als(I) bezeichnet, verwiesen.

Die in diesen Dokumenten beschriebe-nen Verfahren seien zwar vom An-spruchsbegehren durch Disclaimer-formulierungen ausgenommen worden,unstreitig erbrächten aber die jetztnoch beanspruchten Verfahren imwesentlichen die gleichen Ergebnissewie diese vorbekannten, anspruchs-gemäß ausgenommenen. Die Entgegen-haltungen hätten den Fachmann dazuangeregt, die dort exemplifizierten Ver-fahren in einem ähnlichen, d.h. da-zwischenliegenden und angrenzendenMolverhältnisbereich, zu variieren. Dabeisei es ohne Bedeutung, wenn im Stand derTechnik das "molare Verhältnis vonEssigsäure zu Sauerstoff" als solchesnicht angesprochen, sondern erst ausanderen Verfahrensparametern zu er-rechnen ist.

Die Frage, ob durch die Disclaimer-formulierung eine ausreichende Neu-heitsabgrenzung gewährleistet sei,wurde offengelassen.

III. Gegen diese Entscheidung wurde am25. August 1983 Beschwerde eingelegt.Die Beschwerdegebühr wurde am 30.August 1983 entrichtet und die Be-schwerde am 15. Oktober 1983 etwawie folgt begründet:Das beanspruchte Verfahren sei er-finderisch, weil der Befund, daß die Ein-haltung des beanspruchten molarenEssigsäure:Sauerstoff -Verhältnisses zurVermeidung von Leistungsabfall beson-ders kritisch ist, nicht nur neu, sondernauch überraschend sei. Bei den Ver-fahren nach den angezogenen Druck-schriften sei dieses Molverhältnis inweiten Grenzen variierbar und könnegleich, größer oder kleiner als 2:1 sein.Die Prüfungsabteilung habe daher denAuswahlcharakter der Erfindungverkannt.

IV. Die Kammer hat von sich aus dieFrage der Neuheit untersucht und dieAuffassung vertreten, daß die in die An-sprüche aufgenommenen punktförmigenDisclaimer zur Abgrenzung des bean-spruchten Verfahrens gegenüber demStand der Technik, wie es sich aus (B),(D), (E), (F), (G), (J) and EPA-4 079 (K)ergibt, nicht ausreichen. Der Be-schwerdeführer hat daraufhin den ur-sprünglichen Patentanspruch, der keinenDisclaimer enthält, aufgegriffen.

V. Die Kammer hat in einer weiterenMitteilung darauf aufmerksam gemacht,daß der Einwand mangelnder Neuheitauch für einen Patentanspruch weiter-besteht, der überhaupt keine Disclaimer-formulierung enthält.

more rapid consumption, a decline inefficiency and damage to the catalyst.This problem must already be regardedas solved in principle and reference ismade in this connection to US-A-3 759 839 (B), DE-C-2 315 037 (D) US-A-3 743 607 (E), GB-A-1 1 89 091 (F),US-A-3 775 342 (G) and DE-C-2 509 251(J), the latter erroneously designatedas (I).

Although the processes described inthese documents had been excludedfrom the object of the claims by dis-claimers, those processes still claimedindisputably produced essentially thesame results as those now excluded asconventional. The citations would havegiven a person skilled in the art the ideaof varying the processes quoted in theexamples in a range of molar ratios thatwas similar in the sense of lying withinthe range indicated and close to it. It wasirrelevant that the "molar ratio of aceticacid to oxygen" was not referred to inthe state of the art but could only be cal-culated from other process parameters.

The question of whether the disclaimersguaranteed an adequate definition ofnovelty was left open.

III. The applicant lodged an appealagainst this decision on 25 August 1983and paid the relevant fee on 30 August1983. The Grounds for Appeal given on15 October 1983 were approximately asfollows:The process claimed was inventive be-cause the finding that observance of themolar ratio of acetic acid to oxygen asclaimed was especially important inavoiding a decrease in efficiency was notonly new but unexpected. In the pro-cesses according to the cited publica-tions this molar ratio was said to bevariable within wide limits and could beequal to, greater than or smaller than2:1. The Examining Division had there-fore failed to recognise the selectivenature of the invention.

IV. The Board of Appeal considered thequestion of novelty of its own motionand came to the conclusion that the dis-claimers of individual values containedin the claims were not sufficient to de-fine the process claimed vis-a-vis thestate of the art as resulting from (B),(D), (E), (F), (G), (J), and EPA-4 079 (K).The appellant then returned to the ori-ginal claim which did not contain adisclaimer.

V. The Board drew attention in a furthercommunication to the fact that the ob-jection of lack of novelty continues toexist even for a claim without a disclaimer.

secondaires exothermiques, conduisantà une consommation plus élevée, à unediminution de puissance et à une altéra-tion du catalyseur. Or, ce problème doitêtre considéré comme déjà résolu pourl´essentiel; à cet égard, il convient dese reporter aux documents US-A-3 759 839 (B), DE-C-2 315 037 (D),US-A-3 743 607 (E), GB-A-1 189 091 (F),US-A-3 775 342 (G) et DE-C-2 509 251 (J) (et non pas (I) commeindiqué par erreur).

Les procédés décrits dans ces anté-riorités sont certes exclus du champ desrevendications par une formulationvalant disclaimer, mais les procédésrevendiqués dans la présente demandeaboutissent incontestablement pourl´essentiel aux mêmes résultats que cesprocédès déjà connus et exclus duchamp des revendications. Les anté-riorités citées auraient conduit l´hommedu métier à faire varier les procédés quiy sont illustrés dans un domaine derapports molaires semblable, c´est-à-dire intermédiaire et avoisinant. Il im-porte peu, dès lors, que le "rapportmolaire de l´acide acétique à l´oxygène"ne soit pas explicitement mentionnédans l´état de la technique, mais doivese calculer à partir d´autres paramètresopératoires.

La question de savoir si une délimita-tion suffisante de la nouveauté estassurée par la formulation valant dis-claimer n´a pas été tranchée.III. Un recours a été formé contre cettedécision le 25 août 1983. La taxe derecours a été acquittée le 30 août 1983et le recours motivé comme suit le 15octobre 1983:

Le procéde revendiqué serait inventif,car la découverte de l´importance cru-ciale que revêt, si l´on veut éviter uneperte de rendement, le respect durapport molaire acide acétique:oxygènerevendiqué, serait non seulement nou-velle, mais également inattendue. Eneffet, dans les procédés selon les anté-riorités citées, il serait possible de fairevarier ce rapport molaire dans de largeslimites, et celui-ci pourrait être égal,supérieur ou inférieur à 2:1. La Divi-sion d´examen aurait par conséquentméconnu le caractère sélectif del´invention.IV. La Chambre s´est penchée à sontour sur la question de la nouveauté.Elle a estimé que les disclaimers ponc-tuels qui figurent dans les revendicationsen vue de délimiter le procédé reven-diqué par rapport à l´état de la tech-nique, tel qu´il résulte des documents(B), (D), (E), (F), (J) et du document EP-A-4 079 (K), ne suffisaient pas. Le re-quérant a alors repris la revendicationd´origine, qui ne contient aucundisclaimer.

V. Dans une autre notification, laChambre a fait observer au requérantque l´objection d´absence de nouveautésubsistait pour une revendication necomportant pas de disclaimer.

* Amtlicher Text. * Translation. * Traduction.

Page 8: 22. November 1984 22 November 1984 22 novembre 1984 ...archive.epo.org/epo/pubs/oj1984/p549_604.pdf · tout Etat contractant doit, pour ce qui le concerne, excuser pour des motifs

VI. Trotzdem hat der Beschwerdeführerletztlich einen solchen Anspruch vorge-legt und die Patenterteilung auf dessenGrundlage beantragt; dieser lautet:"Verfahren zur Herstellung von Vinyl-acetat in der Gasphase durch Umsetzungvon Ethylen, Essigsäure und moleku-larem Sauerstoff an palladiumhaltigenTrägerkatalysatoren, die zum Beispieleine Palladiumverbindung wie z.B. Palla-diumacetat, eine Cadmiumverbindungwie z.B. Cadmiumacetat, ein Alkaliacetatwie z.B. Kaliumacetat und gegebenen-falls Manganacetat oder elementaresPalladium, elementares Gold und einAlkaliacetat wie z.B. Kaliumacetat ent-halten, dadurch gekennzeichnet, daß dasmolare Verhältnis von Essigsäure zuSauerstoff im eingesetzten Gasgemischoberhalb von 2:1 und unterhalb von4:1, zum Beispiel zwischen 2,2:1 und3,5:1 liegt."

Es wird weiter der Standpunkt vertreten,daß es sich bei dem so formuliertenVerfahren um eine Auswahlerfindunghandelt. Diese sei neu, weil das Ab-sinken der Katalysatoraktivität währendder Vinylacetatsynthese mehrfachbeschrieben wurde, aber bisher nichtvermieden werden konnte.

Entscheidungsgründe

1. Die Beschwerde entspricht den Er-fordernissen von Artikel 106 bis 108und der Regel 64 EPÜ, sie ist daherzulässig.

2. Der geltende Patentanspruch mußbereits aus formellen Gründen scheitern,weil der obere Grenzwert für das ein-zuhaltende Molverhältnis von Essig-säure zu Sauerstoff von unterhalb 4:1 inder Erstoffenbarung keine Stütze findet(vgl. Beschreibung, Seite 2 und diePatentansprüche 1 und 2), worauf dieKammer eigens hingewiesen hatte (vgl.Mitteilung vom 4.5.84).Nach Artikel 123 (2) EPÜ darf einePatentanmeldung nicht in der Weisegeändert werden, daß ihr Gegenstandüber den Inhalt der Anmeldung in ihrerursprünglich eingereichten Fassunghinausgeht. Im vorliegenden Fall wirdeine solche Erweiterung darin gesehen,daß der ursprünglich nur durch seineUntergrenze (oberhalb von 2:1) mar-kierte allgemeine Bereich für das o.g.Molverhältnis willkürlich auf "unter-halb von 4:1" eingegrenzt worden ist.Auch in der Einschränkung einesBereichs kann eine unzulässige Er-weiterung liegen, nämlich dann, wennsie der Fachmann der Erstoffenbarungnicht entnommen hätte.Hierzu wurde vorgebracht, dem Fach-mann sei bekannt, daß bei großtech-nischen Anlagen Essigsäureschwankungenim Reaktoreingangsgas von 3% absolutnicht ausgeschlossen werden könnten,so daß unter Berücksichtigung dieserSchwankung der nummehr bean-spruchte Bereich praktisch dem erstoffen-barten Vorzugsbereich von 2,2:1 bis3,5:1 entspreche.

VI. All the same the appellant ultimatelysubmitted such a claim, on the basis ofwhich he asked that a patent be granted.This claim read as follows:"A process for producing vinyl acetatein the gaseous phase by reacting ethyl-ene, acetic acid and molecular oxygenin the presence of supported catalystscontaining palladium which may, forexample, contain a palladium compoundsuch as palladium acetate, a cadmiumcompound such as cadmium acetate, analkali acetate such as potassium acetateand, if appropriate, manganese acetateor elemental palladium, elemental goldand an alkali acetate such as potassiumacetate, characterised in that the molarratio of acetic acid to oxygen in thereacted gas mixture is greater than 2:1and below 4:1, for example between2.2:1 and 3.5:1."

It was still maintained that the processso described was a selective invention.The latter was new because the loweringof catalyst activity during vinyl acetatesynthesis had been described on variousoccasions, but it had not hitherto beenpossible to avoid it.

Reasons for the Decision1. The appeal complies with the require-ments of Articles 106 to 108 and Rule64 EPC and is therefore admissible.

2. The present claim must be disallowedeven on purely formal grounds becausethe upper limit for the required molarratio of acetic acid to oxygen, namelyless than 4:1, is not supported in thefirst disclosure (see page 2 of the speci-fication and claims 1 and 2) a pointthe Board of Appeal made on its ownaccount (see communication of 4 May1984).

Article 123 (2) EPC states that a patentapplication may not be amended in sucha way that it contains subject-matterwhich extends beyond the content of theapplication as filed. In the present casesuch an extension is considered to havebeen made because the general rangefor the molar ratio referred to above,which was originally defined only interms of a lower limit (above 2:1), hasbeen arbitrarily limited by the insertionof the words "below 4:1". The limitingof a range may also be regarded as anon-allowable extension where a personskilled in the art would not have under-stood this from the first disclosure.

The applicant answered this point bystating that a person skilled in the artwould be aware that in industrial plantsit was absolutely impossible to rule out3% fluctuations in the amount of aceticacid in the reactor input gas and thathaving regard to this fluctuation therange now claimed was in practice equi-valent to the preferred range of 2.2:1 to3.5:1 as first disclosed.

VI. Passant outre, le requérant a finale-ment présenté une telle revendication,sur la base de laquelle il demande la dé-livrance du brevet. Elle se lit comme suit:"Procédé pour la préparation d´acétatede vinyle en phase gazeuse par réactiond´éthylène, d´acide acétique et d´oxy-gène moléculaire sur des catalyseursdéposés sur un support et contenant dupalladium, par exemple un dérivé dupalladium comme l´acétate de palla-dium, un dérivé du cadmium commel´acétate de cadmium, un acétate alcalincomme l´acétate de potassium, et éven-tuellement de l´acétate de manganèseou du palladium élémentaire, de l´orélémentaire et un acétate alcalin commel´acétate de potassium, caractérisé ence que le rapport molaire de l´acide acé-tique à l´oxygène dans le mélangegazeux introduit est supérieur à 2:1 etinférieur à 4:1, par exemple entre 2,2:1et 3,5:1."Le requérant soutient également que leprocédé ainsi formulé constituerait uneinvention de sélection. Celle-ci seraitnouvelle parce que si la chute d´activitédu catalyseur au cours de la synthèse del´acétate de vinyle a été maintes foisdécrite, elle n´a jamais pu être évitéeauparavant.

Motifs de la décision1. Le recours répond aux conditionsénoncées aux articles 106, 107 et 108et à la règle 64 de la CBE; il est doncrecevable.

2. La revendication actuelle doit êtrerejetée d´emblée pour des raisons for-melles, car la limite supérieure de moinsde 4:1 pour le rapport molaire qu´il fautrespecter entre l´acide acétique et l´oxy-gène n´est pas étayée par la descriptioninitiale (voir la description page 2 et lesrevendications 1 et 2), ce que laChambre avait tout spécialement faitobserver (voir notification du 4.5.84).

Selon l´article 123 (2) de la CBE, unedemande de brevet ne peut pas êtremodifiée de manière que son objets´étende au-delà du contenu de la de-mande telle qu´elle a été déposée. Dansle cas d´espèce, on peut voir une telleextension dans le fait que le domainegénéral, marqué initialement seulementpar sa limite inférieure (supérieur à 2:1)pour le rapport molaire mentionné plushaut, a été limité arbitrairement à "in-férieur à 4:1". La limitation d´un do-maine peut également constituer uneextension inadmissible précisémentlorsque l´homme du métier ne l´auraitpas déduite de la description initiale.

A cet égard, le requérant a alléguéque l´homme du métier n´ignore pas que,dans le cas d´installations à l´échelleindustrielle, on ne peut pas absolumentexclure des incertitudes de 3% en ce quiconcerne l´acide acétique dans lemélange de départ de la réaction sibien que, compte tenu de telles varia-tions, le domaine maintenant revendiquécorrespondrait pratiquement au domaine

* Amtlicher Text. * Translation. " Traduction.

Page 9: 22. November 1984 22 November 1984 22 novembre 1984 ...archive.epo.org/epo/pubs/oj1984/p549_604.pdf · tout Etat contractant doit, pour ce qui le concerne, excuser pour des motifs

Diese Auffassung geht schon deshalbfehl, weil der geltende Patentanspruchnicht nur die großtechnische Herstellungvon Vinylacetat umfaßt, sondern auchdessen Herstellung in kleineren An-lagen, für welche größere Essigsäure-schwankungen eingeräumt werden (vgl.Eingabe vom 25.6.84, 5.3 Abs.1). Jeden-falls ohne gleichzeitige Festlegung derGröße der Anlage ist die o.g. Einfügungder Bereichsobergrenze unzulässig, weilwillkürlich.

Indes erscheint es der Kammer nichtangemessen, die Zurückweisung derBeschwerde auf den Verstoß gegen Form-erfordernisse zu stützen, weil imBeschwerdeverfahren die Frage der Neu-heit im Vordergrund stand.

3. Das vorliegende Verfahren betrifftnach dem geltenden Patentanspruch einVerfahren zur Herstellung von Vinyl-acetat durch Umsetzung von Ethylen,Essigsäure und molekularem Sauerstoffin der Gasphase an palladiumhaltigenTrägerkatalysatoren. Dabei liegt dasPalladium entweder in Form einer Verbin-dung in Kombination mit einer Cad-miumverbindung und Alkaliacetat(Katalysator a) oder in elementarer Formin Kombination mit elementarem Goldund Alkaliacetat (Katalysator b) vor. DieAnmelderin hat festgestellt, daß einderartiges Verfahren nicht immer ohneexotherme Nebenreaktionen abläuft, dieihrerseits zu hohem Abbrand, Leistungs-abfall und Katalysatorschädigung führen.

Die anmeldungsgemäße Aufgabe be-steht darin, diese Nachteile zu vermeiden.Zur Lösung wird hierfür vorgeschlagen,das molare Verhältnis von Essigsäure zuSauerstoff im eingesetzten Gasgemischzwischen oberhalb 2:1 und unterhalb4:1 einzustellen.

4. Nun sind bereits im Stand der TechnikVerfahren zur Herstellung von Vinyl-acetat nach der gleichen Verfahrens-weise unter Einsatz der gleichen Kataly-satoren und Anwendung des im bean-spruchten Bereichs liegenden Essig-säure:Sauerstoff -Molverhältnisses be-schrieben. Nach der Verfahrensvariante,die vom "Salz"- Katalysator a Gebrauchmacht, wird Essigsäure und Sauerstoffin den folgenden Molverhältnissen ein-gesetzt: 2,762:1 (vgl. K, sämtliche Bei-spiele), 2,68:1 (vgl. B, Beispiele 1-7) und2,34:1 (vgl. D, Beispiele la-li). Nach deranderen Variante ("Metall" -Katalysatorb) liegt das Molverhältnis Essig-säure:Sauerstoff bei 3,3:1 (vgl. E, Bei-spiele 4-11), 3,19:1 (vgl. E, Spalte 3Zeilen 25/26), 2,98:1 (vgl. F, Beispiel 1),3,11:1 (vgl. G, Beispiele 1-3), 3,28:1(vgl. G, Beispiel 5) und 2,762:1 (vgl. J,Beispiel 5). Die genannten Verhältnis-zahlen sind zwar in den Druckschriftennicht in dieser Form angegeben, sielassen sich aber mühelos aus den Anga-

This argument is already undermined bythe fact that the present claim covers notonly industrial-scale production of vinylacetate but also its production on asmaller scale where greater fluctuationsin acetic acid are allowed for (seecommunication of 25 June 1984,5.3 (1)). In any case the insertion of anupper limit to the range, being arbitrary,is not permissible unless the size of theplant is specified at the same time.

However, the Board does not consider itreasonable to reject the appeal on thegrounds of failure to comply with formalrequirements, because in appeal pro-ceedings it is the question of noveltywhich is at the forefront.

3. The process as claimed relates to theproduction of vinyl acetate by reactingethylene, acetic acid and molecularoxygen in the gaseous phase in the pre-sence of supported catalysts containingpalladium. The palladium is either in theform of a compound in combination witha cadmium compound and alkali acetate(catalyst a) or in elemental form in com-bination with elemental gold and alkaliacetate (catalyst b). The applicant foundthat this kind of process did not alwaystake its course without secondary exo-thermal reactions which in their turnlead to higher consumption, reducedefficiency and damage to the catalyst.

The problem of the invention as claimedin the application was to obviate thesedisadvantages. The solution proposedwas that the molar ratio of acetic acidto oxygen in the gaseous mixture usedshould be adjusted between a lowerlimit of 2:1 and an upper limit of 4:1.

4. However, processes for the produc-tion of vinyl acetate involving the sameprocedure and the same catalysts andusing acetic acid in a molar ratio to oxy-gen within the range claimed are alreadyknown in the art. According to a variantof the process which utilises the "salt"catalyst a, acetic acid and oxygen aretaken in the following molar ratios:2.762:1 (see K, all examples), 2.68:1(see B, examples 1-7) and 2.34:1 (seeD, examples la-li). According to thesecond variant with the "metal" catalystb, the molar ratio of acetic acid to oxy-gen is 3.3:1 (see E, examples 4-11),3.19:1 (see E, column 3, lines 25/26),2.98:1 (see F, example 1), 3.11:1 (see G,examples 1-3), 3.28:1 (see G, example5) and 2.762:1 (see J, example 5). Al-though the figures given for the ratiosare not stated in this form in the variouspublications, they can be derived with-out difficulty from the information givenconcerning the quantities of the tworeactants, a point which the appellant

préféré, divulgué initialement, de 2,2:1à 3,5:1.

Ce point de vue se révèle erroné dufait que la revendication actuelle nerecouvre pas seulement la préparationd´acétate de vinyle à l´échelle indus-trielle, mais également sa préparationdans des installations plus petites, pourlesquelles on admet des variations plusgrandes de l´acide acétique (voir corres-pondance du 25.6.84, 5.3 1er alinéa).En tout cas, si la dimension de l´installa-tion n´est pas simultanément précisée,l´insertion de la limite supérieure dudomaine, telle que mentionnée plushaut, est inadmissible car arbitraire.Toutefois, la Chambre ne croit pasopportun de fonder le rejet du recourssur un manquement aux exigences for-melles, étant donné que la question dela nouveauté se place au premier plande la procédure de recours.

3. Le procédé selon la revendicationactuelle a pour objet la préparationd´acétate de vinyle par réaction d´éthy-lène, d´acide acétique et d´oxygènemoléculaire en phase gazeuse sur descatalyseurs déposés sur un support etcontenant du palladium. Le palladium seprésente alors soit sous forme d´undérivé en combinaison avec un dérivédu cadmium et un acétate alcalin(catalyseur a), soit sous forme élémen-taire en combinaison avec de l´or élé-mentaire et un acétate alcalin (cataly-seur b). Le demandeur a constaté qu´untel procédé ne se déroule pas toujourssans des réactions secondaires exother-miques qui conduisent à leur tour à uneconsommation élevée, à une diminutionde puissance et à une altération ducatalyseur.Le problème posé selon la demandeconsiste à éviter ces inconvénients.Pour le résoudre, il est proposé de réglerle rapport molaire de l´acide acétique àl´oxygène dans le mélange gazeux intro-duit entre une valeur supérieure à 2:1et une valeur inférieure à 4:1.

4. Or, des procédés de préparationd´acétate de vinyle suivant la mêmetechnique opératoire, avec additiondes mêmes catalyseurs et utilisation durapport molaire acide acétique:oxygènesitué dans le domaine revendiqué, sontdéjà décrits dans l´état de la technique.Conformément à la variante du procédéqui fait usage du catalyseur sous formede "sel" a, l´acide acétique et l´oxygènesont introduits dans les rapportsmolaires suivants: 2,762:1 (voir docu-ment K, tous les exemples), 2,68:1 (voirdocument B, exemples 1-7) et 2,34:1(voir document D, exemples la-li). Selonl´autre variante (catalyseur sous formede "métal" b), le rapport molaire acideacétique:oxygène est d´environ 3,3:1(voir document E, exemples 4-11),3,19:1 (voir document E, colonne 3,lignes 25, 26), 2,98:1 (voir document F,exemple 1), 3,11:1 (voir document G,exemples 1-3), 3,28:1 (voir document G,exemple 5) et 2,762:1 (voir document J,exemple 5). Certes, les valeurs indiquées

* Amtlicher Text * Translation. * Traduction.

Page 10: 22. November 1984 22 November 1984 22 novembre 1984 ...archive.epo.org/epo/pubs/oj1984/p549_604.pdf · tout Etat contractant doit, pour ce qui le concerne, excuser pour des motifs

ben über die Mengen dieser beidenReaktanten entnehmen, was auch vomBeschwerdeführer nicht bestritten wird(vgl. Eingabe vom 17.4.84, Seite 4Mitte).

Alle diese im Zusammenhang mit derVinylacetatsynthese beschriebenen neunVerhältniswerte liegen verstreut inner-halb des Bereichs, dessen Einhaltunganmeldungsgemäß als kritisch angese-hen und als erfindungswesentlich be-ansprucht wird. Dies hat zur Folge, daßdie Neuheit des beanspruchten Bereichsbzgl. des einzuhaltenden Molverhält-nisses der genannten Reaktionsteilnen-mer zerstört wird. Damit stimmt dasbeanspruchte Verfahren mit denen deso.g. Standes der Technik in allen Ver-fahrensdetails überein, so daß es inso-weit bereits zum Stand der Technikgehört und wegen mangelnder Neuheitnicht mehr patentiert werden kann (Art.52 (1) und 54 (1) EPÜ).

5. Diese Feststellung gilt nicht nur fürdie zumeist aus Beispielen errechen-baren "punktförmigen" Verhältniswerte;denn Beispiele sind im allgemeinen nurspezielle Ausführungsformen einer brei-teren Lehre und müssen im Zusammen-hang damit im Lichte des allgemeinenFachwissens betrachtet werden.

Wenngleich in den oben genanntenEntgegenhaltungen keine allgemeinenund bevorzugten Essigsäure:Sauerstoff-Bereiche zahlenmäßig definiert sind, sogehört es doch zum Grundwissen desFachmanns, daß chemische Verfahrenwie die Vinylacetatsynthese, bei denenzündfähige Gasgemische umgesetztwerden, in der Praxis außerhalb derbekannten Explosionsgrenzen durchzu-führen sind, vgl. auch (K) Seite 9, Zeilen20-23, (E) Spalte 3, Zeilen 23/24 und(J) Spalte 5, Zeilen 12-15. Gemäß denbeiden Entgegenhaltungen (K) und (J)hält man zu diesem Zweck die Sauer-stoffkonzentration im Gasgemisch nied-rig, beispielsweise unter 8 Vol.-%, be-zogen auf das essigsäurefreieGasgemisch.

Es ist ebenfalls allgemeines Fachwissen,daß sich die Zündgrenze des Gas-gemisches mit steigendem Druck zu ge-ringeren Sauerstoffkonzentrationen hinverschiebt (vgl. (D) Spalte 1, Zeilen 64-66). Da das bekannte Verfahren bei 1-20Atm. (1 Atm.= 1,01325 bar), besonders1-10 Atm. (vgl. (E) Spalte 3, Zeile 29),insbesondere 5-10 Atm. (vgl. (D) Seite 2,Zeilen 1-2) durchgeführt wird, sind auchsämtliche "punktförmigen" Zahlenwerteaus Beispielen in diesem Lichte zu se-hen. Das hat zur Folge, daß sich dasjeweils aus dem Stand der Technik er-rechnete, lediglich beispielhafte Essig-säure:Sauerstoff-Verhältnis beim sach-gerechten Lesen auf die Werte erstreckt,die unter Berücksichtigung desDruckbereichs, wenigstens des Vorzugs-bereichs den oben genannten An-

does not contest (see communication of17 April 1984, middle of page four).

All nine ratios referred to in connectionwith the synthesis of vinyl acetate arescattered within the range, adherence towhich is stated in the application to becritical and essential to the invention.This has the effect of destroying thenovelty of the range claimed and of themolar ratio to be observed between thereactants referred to. The processclaimed is therefore identical in everydetail with those of the state of the artdescribed above, of which it thus formsa part, and is not patentable on groundsof lack of novelty (Article 52 (1) and54 (1) EPC).

5. This statement applies not only to theindividual ratio values which may usuallybe calculated from examples, sinceexamples are in general only specificembodiments of a broader teaching andmust therefore be considered in conjunc-tion therewith and in the light of thegeneral knowledge of the art.

Although the citations referred to abovedo not define in numerical terms anygeneral acetic acid: oxygen ratios to bepreferred, it is part of the basic knowledgeof a person skilled in the art that chemi-cal processes such as the production ofvinyl acetate by synthesis involving thereaction of ignitable gas mixtures mustin practice be effected outside theknown explosion limits (see also (K)page 9, lines 20-23, (E) column 3, lines23/24 and (J) column 5, lines 12-15).According to the two citations (K) and(J), therefore, the concentration of oxy-gen in the gaseous mixture is kept low,for example below 8% by volume, rela-tive to the gaseous mixture containingno acetic acid.

It is also general knowledge in the artthat the ignition limit of the gaseousmixture shifts in the direction of loweroxygen concentrations as pressure in-creases (see (D) column 1, lines 64-66).Since the conventional process is carriedout at 1-20 atm (1 atm = 1.01325 bar),especially between 1 and 10 atm (see(E) column 3, line 29), and in particular5-10 atm (see (D) page 2, lines 1-2) anyindividual values taken from the exam-ples must also be viewed in this light.Consequently, the ratio of acetic acid tooxygen calculated in each case from thestate of the art and given only by way ofexample extends, if read as intended, tothose values which having regard tothe pressure range or at least the pre-ferred pressure range meet the aboverequirements for non-ignitable mixtures.

pour le rapport en cause ne figurent passous cette forme dans les documentspubliés, mais elles se déduisent sanspeine des indications relatives aux quan-tités des deux réactifs, ce qui n´est d´ail-leurs pas contesté par le requérant (voircorrespondance du 17.4.84, milieu de lapage 4).

Toutes les valeurs de rapport aunombre de neuf dont il est fait étatà propos de la synthèse d´acétate devinyle, sont dispersées à l´intérieur dudomaine dont le respect est considérécomme critique et qui est revendiquécomme l´essence de l´invention selon lademande. I! s´ensuit que la nouveautédu domaine revendiqué, en ce qui con-cerne le rapport molaire à maintenirentre les partenaires réactionnels susditsest détruite. Le procédé revendiquécoïncide alors en tous points avec ceuxde l´état de la technique pertinent, sibien qu´il en fait déjà partie et ne sauraitêtre breveté, en raison de l´absence denouveauté (art. 52 (1) et 54 (1) de laCBE).

5. Cette constatation ne vaut pas seule-ment pour les valeurs de rapport "ponc-tuelles" qu´on peut calculer le plus sou-vent à partir d´exemples; en effet, lesexemples ne représentent en généralque des formes de réalisation particu-lières d´un enseignement plus large etdoivent de ce fait être considérés à lalumière des connaissances normales del´homme du métier.

Même si les antériorités citées nedéfinissent pas numériquement de rap-ports acide acétique:oxygène, générauxet préférés, l´homme du métier saitbien qu´en fait, des procédés chimiques,tels que la synthèse d´acétate de vinyle,qui font intervenir des mélanges gazeuxinflammables doivent être mis en oeuvreen dehors des limites d´explosion con-nues; voir également les documents (K)page 9, lignes 20-23, (E) colonne 3,lignes 23, 24 et (J) colonne 5, lignes 12-15. Selon les antériorités (K) et (J) quiconcordent sur ce point, on maintientdans ce but la concentration en oxygènedans le mélange gazeux à un bas niveau,par exemple au dessous de 8% envolume, par rapport au mélange gazeuxexempt d´acide acétique.

De même, l´homme du métier n´ignorepas que la limite d´inflammabilité dumélange gazeux se déplace, lorsque lapression augmente, vers des concentra-tions plus faibles en oxygène (voir ledocument (D) colonne 1, lignes 64-66).Comme le procédé connu est mis enoeuvre à 1-20 atmosphères (1 atmos-phère = 1,01325 bar), en particulier à1-10 atmosphères (voir document (E)colonne 3, ligne 29), et plus particulière-ment à 5-10 atmosphères (voir docu-ment (D) page 2, lignes 1, 2), il fautégalement considérer sous cet anglel´ensemble des valeurs "ponctuelles"tirées des exemples. Il s´ensuit que lerapport acide acétique:oxygène ayantsimplement valeur d´exemple et calculéà chaque fois à partir de l´état de latechnique s´étend, lorsqu´on le lit con-

* Amtlicher Text. * Translation. * Traduction.

Page 11: 22. November 1984 22 November 1984 22 novembre 1984 ...archive.epo.org/epo/pubs/oj1984/p549_604.pdf · tout Etat contractant doit, pour ce qui le concerne, excuser pour des motifs

forderungen an nicht zündfähige Ge-mische entsprechen.

So zeigt die Berechnung der Anmelderinzu (D) (vgl. Eingabe vom 14.1.83, Seite2), daß sich das gemäß den BeispielenI a bis i bei einem Druck von 9 Atm.angewendete Essigsäure:Sauerstoff -Ver-hältnis von 2,34:1 bei niedrigeremDruck, also 5-8 Atm., der die Wahl eineshöheren Sauerstoffgehalts von 8% ange-raten erscheinen läßt, bis zum Verhält-nis von 1,88:1 verschiebt, d.h. derpunktförmige Wert 2,34:1 hinsichtlichdes oben genannten Verhältnisses ausden Beispielen wird der Lehre dieserEntgegenhaltung nicht gerecht, weil sieden beanspruchten Bereich zumindestvon 2,34 bis zur Untergrenze hin ein-schließt. Erhöht man im Rahmen derbekannten Lehre umgekehrt den Druckvon 9 Atm. auf 10 Atm., so steigt dasoben genannte Verhältnis über 2,34:1hinaus an.Gleiches gilt natürlich für die übrigen,vormals durch Disclaimer ausgenom-menen Verhältniszahlen, die sich dem-nach zu Verhältnisbereichen erweitern.Im übrigen liefert der Beschwerde-führer selbst ein weiteres Beispiel dafür,daß die in den Beispielen der o.g. Ent-gegenhaltungen enthaltenen Essig-säure:Sauerstoff-Werte vom Fachmannnicht punktförmig, sondern im Sinneeines engeren oder weiteren Bereichsverstanden werden. Danach weiß derFachmann aus Erfahrung, daß bei dergroßtechnischen Herstellung von Vinyl-acetat nach der bekannten Verfahrens-weise Essigsäureschwankungen im Aus-gangsgasgemisch von 3% nicht ausge-schlossen werden können und daß beikleineren Anlagen diese Schwankungennoch größer sind.

Berücksichtigt man eine Schwankungvon 3%, so ergibt sich nach der Berech-nung des Beschwerdeführers aus dembeispielhaften Molverhältnis Essig-säure:Sauerstoff von 2,67:1 ein Ver-hältnisbereich von 2,59:1 bis 2,75:1(vgl. Eingabe vom 25.6.84, S.3 Abs. 1).Aufgrund dieses auf Erfahrung gegrün-deten Wissens erweitert sich jeder inden Druckschriften enthaltene punkt-förmige Essigsäure:Sauerstoff-Wert fürden Fachmann beim Lesen zu einemBereich.

6. Ein mit allen technischen Merkmalenvorbeschriebenes chemisches Her-stellungsverfahren wird nicht dadurchneu, daß auf einen damit verbundenen,aber von der Fachwelt bislang nichterkannten Verfahrensvorteil hingewiesenwird, der bei unveränderter Ausführungdes Verfahrens eintritt; denn einchemisches Herstellungsverfahren wirdpatentrechtlich durch die Bezeichnungder Ausgangsstoffe, der Verfahrspara-meter und des Endprodukts eindeutigfestgelegt und nacharbeitbar beschrie-ben. Die anmeldungsgemäße zusätz-liche Information, daß bei Ausführung

Thus the applicant´s computation for (D)(see communication of 14 January1983, page 2) shows that according toexamples la to i and with a pressure of9 atm the ratio of acetic acid to oxygenof 2.34:1 at low pressure, i.e. 5-8 atm,which would make the choice of a higheroxygen content of 8% appear advisable,shifts to the ratio 1.88:1. In other wordsthe individual value 2.34:1 does not con-form to the teaching of this citationwith regard to the above ratio as re-flected in the examples because it in-cludes the range claimed at least from2.34 down to the lower limit. Con-versely, if in the context of the conven-tional teaching the pressure is increasedfrom 9 to 10 atm, the ratio referred torises beyond 2.34:1.

The same applies of course to the otherratio values previously excluded from theapplication by the disclaimer, which areaccordingly extended into ratio ranges.Moreover, the applicant himself providesanother example of how a person skilledin the art understands the acetic acid tooxygen ratios contained in the examplesin the above-mentioned citations not asindividual values but in the sense of anarrower or wider range. According tothe applicant, the person skilled in theart knows from experience that, whenvinyl acetate is produced on an industrialscale using the conventional process,fluctuations of 3% in the acetic acidcontent of the initial gas mixture cannotbe ruled out, and that in smaller plantsthese fluctuations are of an even higherorder.

If a fluctuation of 3% is taken intoaccount, and on the basis of the appli-cant´s computation from the example ofa molar ratio of acetic acid to oxygen of2.67:1, there results a ratio range of2.59:1 to 2.75:1 (see communication of25 June 1984, page 3, para. 1). In thelight of this empirical knowledge, eachof the specific individual values given forthe ratio of acetic acid to oxygen con-tained in the citations will be understoodby a person skilled in the art as being arange.

6. A chemical production process pre-viously described with all its technicalfeatures is not rendered novel by thefact that attention is drawn to an advan-tage associated with the process but nothitherto acknowledged by the art andwhich occurs when the process iscarried out without any change. Fromthe point of view of patent law a chemi-cal production process is clearly definedby a statement of the initial substances,the process parameters and the end pro-ducts, and can be subsequently modi-fied. The additional information con-tained in the application to the effect

venablement, aux valeurs qui comptetenu du domaine de pression, ou aumoins du domaine préféré corres-pondent aux conditions susmentionnéespour des mélanges non inflammables.C´est ainsi que le calcul du demandeureffectué d´après le document (D) (voircorrespondance du 14.1.83, page 2)montre que le rapport acide acé-tique:oxygène de 2,34:1 utilisé dans lesexemples la à li associé à une pressionde 9 atmosphères se décale, lorsqu´onabaisse la pression à 5-8 atmos-phères le choix d´une teneur en oxy-gène de 8%, c´est-à-dire plus élevéeapparaissant alors comme indiqué pour s´établir à 1,88:1, Cela signifie quela valeur ponctuelle 2,34:1 tirée desexemples ne reflète pas l´enseignementde cette antériorité, car celle-ci inclut ledomaine revendiqué, au moins de 2,34jusqu´à la limite inférieure. Si, dans lecadre de l´enseignement connu, on élèveà l´inverse la pression de 9 atmosphèresà 10 atmosphères, le rapport susditaugmente au-delà de 2,34:1.

Il en va de même naturellement desautres valeurs du rapport auparavantexclues par disclaimer, qui s´étendentpar conséquent à des domaines entiers.

Du reste, le requérant fournit lui-mêmeun autre exemple de ce que les valeursdu rapport acide acétique:oxygène con-tenues dans les exemples des antérioritéscitées ne sont pas considérées parl´homme du métier comme ponctuellesmais bien comme englobées dans undomaine plus ou moins large. En effet,l´homme du métier sait par expérienceque, dans la préparation d´acétate devinyle à l´échelle industrielle selon lemode opératoire connu, on ne peut pasexclure des incertitudes de l´ordre de 3%en ce qui concerne l´acide acétique dansle mélange de départ, et que ces varia-tions sont encore plus grandes pour desinstallations plus petites.Si l´on considère une variation de 3%, onobtient, d´après le calcul du requérant, àpartir du rapport molaire acide acé-tique:oxygène de 2,67:1 servantd´exemple, un domaine de rapportsmolaires de 2,59:1 à 2,75:1 (voir cor-respondance du 25.6.84, p. 3, 1 eralinéa). En raison de cette connaissancefondée sur l´expérience, chaque valeurponctuelle du rapport acide acé-tique:oxygène contenue dans les anté-riorités s´étend automatiquement à undomaine entier pour le lecteur averti.

6. Un procédé de préparation chimiqueprécédemment décrit avec toutes sescaractéristiques techniques n´est pasrendu nouveau par le fait que l´on in-dique un avantage opératoire qui y estlié mais qui n´avait pas été jusqu´alorsreconnu par les spécialistes, et qui semanifeste lors de la réalisation du pro-cédé non modifié; en effet, en droit desbrevets, un procédé de préparationchimique est clairement défini et décritde façon à pouvoir être reproduit,lorsque sont indiqués les substances dedépart, les paramètres opératoires etle produit final. L´information supplé-

* Amtlicher Text. * Translation. * Traduction.

Page 12: 22. November 1984 22 November 1984 22 novembre 1984 ...archive.epo.org/epo/pubs/oj1984/p549_604.pdf · tout Etat contractant doit, pour ce qui le concerne, excuser pour des motifs

der vorbekannten Verfahrensweisenohne jegliche technische Änderung dieKatalysatoraktivität erhalten bleibt,mag für die Praxis von großem Wertsein; gleichwohl eröffnet dieser Hin-weis mangels eines Vorschlags über denEinsatz zusätzlicher technischer Mittelkeine neue technische Lehre. Dembereits Bekannten wird also nichts hin-zugefügt, weil sich die vermeintlicheErfindung darin erschöpft, eben so zuverfahren, wie bereits beschrieben.

7. Aus dem gleichen Grunde kann derBeschwerdeführer auch nicht mit demArgument gehört werden, es handlesich beim beanspruchten Verfahren umeine Auswahlerfindung, weil es mit sei-nen Vorteilen dem Stand der Techniknicht expressis verbis zu entnehmen sei(vgl. Eingabe vom 14.4.84, Seite 4 Abs.2 Ende). Auch für Auswahlerfindungengelten die gleichen Patentierungsvor-aussetzungen wie für andere Erfin-dungen, nämlich zunächst die, daß sieneu sein, d. h. sich vom Bekannten durchtechnische Merkmale unterscheidenmüssen. Daran fehlte es wie ausge-führt im vorliegenden Fall. Auf den An-meldungsgegenstand kann daher man-gels Neuheit kein europäisches Patenterteilt werden.

Aus diesen Gründenwird wie folgt entschieden:

Die Beschwerde wird zurückgewiesen.

that when the conventional processesare used without any technical modifi-cation the catalyst activity is maintainedmay be of great value in practical terms;but at the same time, because of the lackof any suggestion as to additional tech-nical means that might be employed,this information incorporates no newtechnical teaching. Nothing, in otherwords, is added to what is alreadyknown because the alleged inventionconsists solely in following the proce-dure already described.

7. For the same reason the appellant´sargument that the process claimed in-volves a selection because it cannot withits advantages be derived directly fromthe state of the art (see communicationof 14 April 1984, page 4, end of para-graph 2) must be discounted. Even inthe case of inventions involving selec-tion, the same prerequisites apply forpatenting as for other inventions, namelyfirst of all that they must be new andthus differ from the conventional art byvirtue of specific technical features. This,as has been explained, was not true inthe present case. The lack of noveltytherefore means that no Europeanpatent can be granted for the subject-matter of the application.

For these reasons,it is decided that:

The appeal is rejected.

mentaire selon la demande, suivantlaquelle lorsqu´on met en oeuvre le pro-cédé précédemment connu sans aucunemodification technique l´activité ducatalyseur demeure inchangée, peutavoir une grande importance pratique;mais faute d´une suggestion sur l´intro-duction de moyens techniques addi-tionnels, elle n´ouvre aucune perspec-tive nouvelle au plan de l´enseignement.Il n´y a donc aucun apport à l´état de latechnique, car l´invention présumée selimite à la reproduction d´opérationsdéjà décrites.

7. Pour la même raison, la Chambre nepeut pas davantage suivre le requérantlorsque celui-ci affirme que le procédérevendiqué constituerait une inventionde sélection parce qu´il n´est pas ex-pressément décrit, avec ses avantages,dans l´état de la technique (voir cor-respondance du 14.4.84, page 4, fin du2 ème alinéa). Des conditions de breveta-bilité identiques s´appliquent à toutesles inventions, y compris aux inventionsde sélection: elles doivent en premierlieu être nouvelles, c´est-à-dire qu´ellesdoivent se distinguer de l´art antérieurpar des caractéristiques techniques nou-velles, dont on ne trouve pas trace en laprésente espèce, comme cela a étéexposé plus haut. Il n´y a donc pas lieude délivrer un brevet européen, en raisonde l´absence de nouveauté.

Par ces motifs,

il est statué comme suit:Le recours est rejeté.

* Amtlicher Text.

Entscheidung der TechnischenBeschwerdekammer 3.2.1 vom3. Juli 1984T 89/84*

Zusammensetzung der Kammer:Vorsitzender: G. AnderssonMitglieder: P. Ford

M. Huttner

Patentinhaberin/Beschwerdeführerin:The Torrington Company

Einsprechende/Beschwerdegegnerin:SKF Kugellagerfabriken GmbHStichwort: Rückzahlung derBeschwerdegebühr/TORRINGTON

EPÜ Artikel 108, 110(2), 113(1),114 (1); Regeln 65 (1), 67

"Beschwerdegebühr" "keineRückzahlung bei Nichteinreichung derBegründung" "keine Notwendigkeitfür eine parallele Beschwerde"

LeitsätzeI. Wird absichtlich keine Beschwerde-begründung eingereicht, um die Be-schwerde unzulässig zu machen, sokann die Beschwerdegebühr nicht zu-rückgezahlt werden.

* Translation.

Decision of the Technical Boardof Appeal 3.2.1 dated3 July 1984T 89/84*

Composition of the Board:Chairman: G. AnderssonMembers: P. Ford

M. Huttner

Patent Proprietor/Appellant: TheTorrington Company

Opponent/Respondent: SKFKugellagerfabriken GmbHHeadword: "Reimbursement ofappeal fees/TORRINGTON"EPC Articles 108, 110 (2), 113 (1),114 (1); Rules 65 (1), 67"Fee for appeal" "no refund ifStatement of Grounds not filed" "Cross-appeal" "no necessity tofile"

HeadnoteI. If a Statement of Grounds of appealis deliberately not filed so as to renderthe appeal inadmissible, the appeal feecannot be reimbursed.

* Traduction.

Décision de la Chambre derecours technique 3.2.1 du3 juillet 1984T 89/84*

Composition de la Chambre:Président: G. AnderssonMembres: P. Ford

M. Huttner

Titulaire du brevet/Requérante: TheTorrington CompanyOpposante/Intimée: SKFKugellagerfabriken GmbHRéférence: "Remboursement de lataxe de recours/TORRINGTON"Articles 108, 110 (2), 113(1), 114(1);règles 65 (1) et 67 de la CBE"Taxe de recours" "remboursementnon accordé en cas de non-présentation du mémoire exposant lesmotifs du recours" "Recoursincident" "ne doit pasnécessairement être formé"

Sommaire

I. Si le requérant a omis à dessein dedéposer le mémoire exposant les motifsdu recours, dans l´intention de rendre lerecours irrecevable, la taxe de recours nepeut être remboursée.

* Übersetzung. * Official text. * Traduction.

Page 13: 22. November 1984 22 November 1984 22 novembre 1984 ...archive.epo.org/epo/pubs/oj1984/p549_604.pdf · tout Etat contractant doit, pour ce qui le concerne, excuser pour des motifs

II. Da die Beschwerdekammern gehaltensind, den Sachverhalt von sich aus zuprüfen, und sie damit jede von derVorinstanz entschiedene Frage neu auf-rollen können, besteht für einen Be-schwerdegegner oder eine Partei, die beieiner von einer anderen Partei einge-legten Beschwerde Beschwerdegegnersein könnte, keine verfahrensbedingteNotwendigkeit für eine "parallele Be-schwerde", selbst wenn das Beschwer-deverfahren nach dem EPÜ die Einle-gung einer "parallelen Beschwerde"zuließe, was noch zu klären ist.

Sachverhalt und AnträgeI. Die Beschwerdeführerin ist Inhaberindes europäischen Patents Nr. 00 16 880,gegen das am 4. Februar 1983 von derSKF Kugellagerfabriken GmbH, Schwein-furt (DE), Einspruch eingelegt wordenwar. Die Einsprechende hatte den Wider-ruf des Patents mit der Begründung bean-tragt, daß sein Gegenstand im Hinblickauf den Stand der Technik nicht patent-fähig sei. Die Beschwerdeführerin hattedie Zulässigkeit des Einspruchs bestrittenund geltend gemacht, daß der Einspruch,selbst wenn er zulässig sei, als unbe-gründet zurückgewiesen werden müsse.

II. In der angefochtenen Entscheidungvom 31. Januar 1984 hatte die Ein-spruchsabteilung die Auffassung ver-treten, daß der Einspruch zwar zulässig,jedoch nicht begründet sei. Sie hattedeshalb beschlossen, den Einspruchzurückzuweisen und das Patent in un-veränderter Form aufrechtzuerhalten.

III. Am 6. April 1984 legte die Beschwer-deführerin fernschriftlich Beschwerdegegen den Teil der Entscheidung ein,der die Zulässigkeit des Einspruchs be-traf. Das Fernschreiben wurde rechtzeitigschriftlich bestätigt und die Beschwerde-gebühr ordnungsgemäß entrichtet.

IV. In der Beschwerdeschrift erklärtedie Beschwerdeführerin, wenn die Ein-sprechende nicht ebenfalls fristgerechtBeschwerde einlege, werde sie keine Be-gründung zu ihrer Beschwerde ein-reichen, sondern bei der Beschwerde-kammer eine Entscheidung auf Unzu-lässigkeit der Beschwerde beantragen."so daß die Beschwerdegebühr zurück-gezahlt werden kann".

V. Die Einsprechende legte innerhalb derFrist keine Beschwerde ein, und der Ver-treter der Beschwerdeführerin führte ineinem am 26. Juni 1984 eingegangenenSchreiben aus, daß er deshalb auchkeine Beschwerdebegründung vorgelegthabe. Er räumte ein, daß die Beschwerdenicht zulässig sei, behauptete jedoch,daß ihre Einreichung "de facto eine ver-fahrensbedingte Notwendigkeit" gewesensei, da nur Fragen, die Gegenstand einerBeschwerde seien, von der Kammer be-handelt werden könnten und die Be-schwerdeführerin die Zulässigkeit desEinspruchs nicht hätte in Frage stellenkönnen, wenn nur die EinsprechendeBeschwerde eingelegt hätte. Der Ver-treter hielt deshalb eine Rückzahlungder Beschwerdegebühr für angebracht.

II. Since a Board of Appeal is required toexamine the facts of its own motion andmay consequently re-open any matterdecided by the department of firstinstance, there seems to be no pro-cedural necessity for the filing of a"cross-appeal" by a respondent or by a

party, who might be a respondent to anappeal filed by another party, even if,which is an open question, the appealprocedure under the EPC permits thefiling of a "cross-appeal".

Summary of Facts and SubmissionsI. The appellant is the proprietor of Euro-pean patent No. 00 16 880 againstwhich an opposition was filed on 4February 1983 by SKF Kugellagerfabri-ken GmbH, Schweinfurt (DE). The oppo-nent requested revocation of the patenton the ground that its subject matterwas not patentable having regard to thestate of the art. The appellant contestedthe admissibility of the opposition andargued that even if it were admissible itshould be rejected as unsubstantiated.

II. By the decision under appeal, dated31 January 1982, the Opposition Divi-sion held that the opposition was admis-sible but that it had not been substan-tiated. Accordingly, it was decided thatthe opposition was rejected and that thepatent should be maintained unamended.

III. On 6 April 1984, by telex, the appel-lant filed a Notice of Appeal against thatpart of the decision which related to theissue of the admissibility of the opposi-tion. The telex was duly confirmed inwriting and the appeal fee was duly paid.

IV. The notice of appeal included a state-ment to ´the effect that if the opponentdid not file a Notice of Appeal in duetime the appellant would not file a State-ment of Grounds in support of its appealbut would request the Board of Appealto issue a decision that the appeal wasinadmissible "thereby enabling theappeal fee to be refunded".

V. The opponent did not file a Notice ofAppeal in due time and by letter re-ceived on 26 June 1984 the appellant´srepresentative stated that accordingly hehad not filed a Statement of Grounds. Heconceded that the appeal was not ad-missible but argued that the filing of theappeal was "in effect a proceduralnecessity" having regard to the fact thatonly matters under appeal can be con-sidered by the Board and had theopponent appealed and the appellantnot appealed, the appellant would nothave been able to argue about theadmissibility of the opposition. Accord-ingly, the representative submitted thatthere was a good case for reimburse-ment of the appeal fee.

II. Etant donné qu une chambre de re-cours est tenue de procéder à l´examend´office des faits, et qu´elle peut en con-séquence réexaminer toute questiontranchée en première instance, il nesemble pas nécessaire pour le bondéroulement de la procédure qu´un"recours incident" soit formé par unintimé ou une partie qui pourrait êtrel´intimé dans un recours formé par uneautre partie, même si ce qui est uneautre question la procédure derecours prévue par la CBE permet laformation d´un "recours incident".

Exposé des faits et conclusionsI. La requérante est titulaire du breveteuropéen n° 00 16 880 ayant fait l´ob-jet d´une opposition formée le 4 février1983 par SKF Kugellagerfabriken GmbH,Schweinfurt (Allemagne). L´opposante ademandé la révocation du brevet aumotif que son objet n´était pas bre-vetable, compte tenu de l´état de latechnique. La requérante a contesté larecevabilité de l´opposition et elle aallégué que même si elle était déclaréerecevable, cette opposition devait êtrerejetée comme non fondée.

II. Par une décision que conteste larequérante, en date du 31 janvier 1982,la Division d´opposition a estimé quel´opposition était recevable, mais qu´ellen´était pas fondée. En conséquence, ellea décidé de rejeter l´opposition et demaintenir le brevet sans modification.

III. Par télex du 6 avril 1984, larequérante a formé un recours contre lapartie de la décision relative à la ques-tion de la recevabilité de l´opposition. Letélex a été dûment confirmé par écrit etla taxe de recours a été dûmentacquittée.

IV. Dans l´acte de recours, il était dé-claré que si l´opposante ne formait pasun recours en temps voulu, la requérantene produirait pas de mémoire exposantles motifs du recours, mais demanderaità la Chambre de recours de rendre unedécision constatant l´irrecevabilité durecours, "de manière à permettre le rem-boursement de la taxe de recours".

V. L´opposante n´a pas formé de recoursen temps voulu et, dans une lettre reçuele 26 juin 1984, le mandataire de larequérante a déclaré qu´il n´avait pas, enconséquence, déposé de mémoire ex-posant les motifs du recours. Tout enreconnaissant que son recours n´étaitpas recevable, il a soutenu que la forma-tion de ce recours était "réellementnécessaire pour le bon déroulement dela procédure", du fait que la Chambrene peut examiner que les questions dontelle est saisie, et que si l´opposante avaitformé un recours alors que la requéranten´en formait pas, la requérante n´auraitpas été en mesure de contester la receva-bilité de l´opposition. Le mandataire adonc allégué qu´il y avait lieu en l´occur-rence de rembourser la taxe de recours.

* Übersetzung. * Official text. * Traduction.

Page 14: 22. November 1984 22 November 1984 22 novembre 1984 ...archive.epo.org/epo/pubs/oj1984/p549_604.pdf · tout Etat contractant doit, pour ce qui le concerne, excuser pour des motifs

Entscheidungsgründe1. Die Beschwerdeführerin hat mit Rechteingeräumt, daß die Beschwerde un-zulässig ist, da sie nicht rechtzeitigbegründet wurde, wie in Artikel 108EPÜ gefordert. Da die Beschwerde nichtzulässig ist, kann die Beschwerde-kammer nicht prüfen, ob sie begründetist (s. Art. 110 (1) EPÜ), sondern mußsie verwerfen (R. 65 (1) EPÜ).

2. Regel 67 EPÜ, in der die Vor-aussetzungen festgelegt sind, unterdenen eine Beschwerdekammer dieRückzahlung der Beschwerdegebühranordnet, nennt als erste Bedingung,daß die Kammer der Beschwerde statt-gibt. Daraus folgt, daß Regel 67 EPÜ ungeachtet anderer Überlegungen imvorliegenden Fall nicht angewandt wer-den kann.

3. In einem früheren Fall hat eine Be-schwerdekammer die Auffassung ver-treten, daß die einschränkende Formu-lierung der Regel 67 EPÜ eindeutig un-vereinbar mit der Ansicht ist, es liegeweitgehend im Ermessen der Beschwer-dekammer, die Rückzahlung der Be-schwerdegebühr anzuordnen (vgl. Ent-scheidung der Technischen Beschwer-dekammer 3.3.1 vom 30. März 1982 inder Sache T 41/82, ABI. EPA 7/1982,256). An diese Entscheidung hat sichdie Technische Beschwerdekammer3.5.1 in ihrer Entscheidung vom 3. März1983 in der Sache T 13/82 (ABI. EPA10/1983, 411), auf die die Beschwerde-führerin in ihrem schriftlichen Vorbrin-gen Bezug nimmt, ausdrücklich ange-lehnt.

4. Wenn ein Beschwerdeführer seineBeschwerde nicht rechtzeitig begründet

sei es wegen eines echten Versäum-nisses (wie in der Sache T 13/82), seies, weil die Beschwerde rein vorsorg-lich eingelegt wurde (wie auf derMünchner Diplomatischen Konferenzerörtert: vgl. Nr. 7 der Entscheidungs-gründe in der Sache T 13/82), oder seies unter Berufung auf eine "verfahrens-bedingte Notwendigkeit" wie im vor-liegenden Fall , dann ist nach Ansichtder Kammer eine Rückzahlung der Be-schwerdegebühr ausgeschlossen. Zwi-schen solchen Fällen kann kein grund-sätzlicher Unterschied gemacht werden.Es wäre nämlich unlogisch, Beschwer-deführer, die absichtlich keine Begrün-dung eingereicht haben, großzügiger zubehandeln als solche, die dies ausUnachtsamkeit versäumt haben.

5. Im vorliegenden Fall sollte aucherwähnt werden, daß die Kammer nichtdavon überzeugt ist, daß für die Be-schwerdeführerin, wie von ihr behauptet,eine "verfahrensbedingte Notwendig-keit" bestand, Beschwerde gegen einefür sie ungünstige Feststellung in einerinsgesamt zu ihren Gunsten ausgefallenenEntscheidung einzulegen. Hätte die Ein-

Reasons for the Decision

1. The appellant has correctly concededthat the appeal is inadmissible becauseno Statement of Grounds in support ofthe appeal was filed in due time as re-quired by Article 108 EPC. Since theappeal is not admissible the Board ofAppeal cannot examine whether it isallowable (cf. Article 110 (1) EPC) andmust reject the appeal (Rule 65 (1) EPC).

2. Rule 67 EPC, in laying down the con-ditions under which a Board of Appealshall order reimbursement of appealfees, requires as its first condition thatthe Board shall deem the appeal inquestion to be allowable. It follows that,irrespective of any other considerations,Rule 67 EPC cannot be applied in thepresent case.

3. It has previously been held by a Boardof Appeal that the restrictive language ofRule 67 EPC is plainly inconsistent withthe idea that a Board of Appeal has awide discretion to order reimbursementof appeal fees: Decision of TechnicalBoard 3.3.1 dated 30 March 1982 inCase T 41/82 (Official Journal EPO,7/1982, 256). That Decision was ex-pressly followed by Technical Board ofAppeal 3.5.1 in its Decision of 3 March1983 in Case T 13/82 (Official JournalEPO, 10/1983, 411), which is referred toin the appellant´s written submissions.

4. If an appellant does not file a State-ment of Grounds in support of his appealin due time, whether as a result of agenuine omission (as in Case T 13/82),or because the appeal was filed as a pre-cautionary measure (as discussed by theMunich Diplomatic Conference: cf. para.7 of the Reasons for the Decision inCase T 13/82), or as an alleged "pro-cedural necessity", as in the presentcase, in the view of the Board, the resultmust be that the appeal fee cannot berefunded. No distinction of principle canbe made between such cases. Indeed, itwould be illogical to treat appellantswho deliberately refrained from filingStatements of Grounds more generouslythan those who failed to file by inad-vertence.

5. In the present case, it should also besaid that the Board is not satisfied thatthere was any "procedural necessity", asalleged, for the appellant to file anappeal against an adverse finding in adecision which, in its overall result, wasfavourable. If the opponent had appealed,the Board would have had to considerthe admissibility question. When con-

Motifs de la décision

1. La requérante a eu raison de recon-naître que son recours était irrecevable,puisqu´aucun mémoire en exposant lesmotifs n´avait été déposé dans le délaiprescrit par l´article 108 de la CBE. Lerecours n´étant pas recevable, laChambre de recours n´est pas en mesured´examiner s´il peut y être fait droit(article 110 (1) de la CBE), et elledoit le rejeter (règle 65 (1) de la CBE).

2. La première des conditions requisespar la règle 67 de la CBE pour le rem-boursement de la taxe de recours estque la chambre de recours fasse droit aurecours. Il s´ensuit que, indépen-damment de toute autre considération,la règle 67 de la CBE n´est pas appli-cable en l´espèce.

3. Dans une affaire antérieure, unechambre de recours avait estimé que laformulation restrictive de la règle 67 dela CBE était en contradiction flagranteavec la conception selon laquelle leschambres de recours ont un large pou-voir d´appréciation en ce qui concernele remboursement de la taxe de recours(cf. la décision T 41/82 rendue le 30mars 1982 par la chambre de recourstechnique 3.3.1, Journal officiel del´OEB n° 7/1982, p. 256). La chambrede recours technique 3.5.1 a expressé-ment suivi la chambre de recourstechnique 3.3.1 dans la décisionT 13/82, citée par écrit par la re-quérante, qu´elle a rendue le 3 mars1983 (cf. Journal officiel de l´OEB n°10/1983, p. 411).

4. Si un requérant ne présente pas entemps voulu le mémoire exposant lesmotifs de son recours, que ce soit enraison d´une omission véritable (commedans l´affaire n° T 13/82), ou parce quele recours n´a été formé que par mesurede précaution (problème discuté lors dela Conférence diplomatique de Munich:cf. point 7 des motifs de la décisionT 13/82), ou parce qu´il était présentécomme "nécessaire pour le bon déroule-ment de la procédure" comme dans laprésente espèce, il s´ensuit nécessaire-ment, de l´avis de la Chambre, que lataxe de recours ne peut être remboursée.Il est impossible d´établir une distinctionfondamentale entre de tels cas. En effet,il serait illogique que des requérantsqui se sont délibérément abstenus deprésenter un mémoire exposant lesmotifs du recours soient traités plusfavorablement que ceux qui ont omispar inadvertance d´accomplir cette for-malité.

5. Il convient également de préciser que,dans la présente espèce, la Chambren´est pas convaincue, contrairement àce qui avait été allégué par la requérante,qu´il était "nécessaire pour le bondéroulement de la procédure" que laditerequérante forme un recours contre unélément défavorable d´une décision quilui était favorable dans l´ensemble. Si

* Übersetzung. * Official text. * Traduction.

Page 15: 22. November 1984 22 November 1984 22 novembre 1984 ...archive.epo.org/epo/pubs/oj1984/p549_604.pdf · tout Etat contractant doit, pour ce qui le concerne, excuser pour des motifs

sprechende Beschwerde eingelegt, sohätte die Kammer die Frage der Zu-lässigkeit prüfen müssen. Bei der Be-handlung einer Beschwerde ist die Be-schwerdekammer immer gehalten, denSachverhalt von sich aus zu prüfen (s.Art. 114 (1) EPÜ); die Kammer kann alsojede von der Vorinstanz entschiedeneFrage, so auch die Frage der Zulässigkeiteines Einspruchs, wieder aufrollen, auchwenn diese von keiner Partei ange-sprochen wird. Natürlich erhalten dannalle betroffenen Parteien entsprechenddem in den Artikeln 110 (2) und 113 (1)EPÜ festgelegten Grundsatz Gelegenheit,sich dazu zu äußern. Für einen Be-schwerdegegner oder eine Partei, die beieiner von einer anderen Partei eingeleg-ten Beschwerde Beschwerdegegner seinkönnte, besteht deshalb bei Verfahrenvor den Beschwerdekammern des EPAkeine Notwendigkeit für eine "paral-lele Beschwerde", selbst wenn das Be-schwerdeverfahren nach dem EPÜ dieEinlegung einer "parallelen Beschwerde"zuließe, was noch zu klären ist.

Aus diesen Gründen

wird wie folgt entschieden:

1. Die Beschwerde wird als unzulässigverworfen.

2. Der Antrag auf Rückzahlung der Be-schwerdegebühr wird zurückgewiesen.

sidering an appeal, a Board of Appeal isalways required to examine the facts ofits own motion (cf. Article 114 (1) EPC)and the Board may, consequently, re-open any matter which was decided bythe department of first instance, includ-ing the admissibility of an opposition,without the matter being raised by anyparty. If it does so, of course, all partiesconcerned will be given an opportunityto present their comments on thematter, in accordance with the principleset out in Articles 110 (2) and 113 (1)EPC. Therefore, there seems to be nonecessity for the filing of a "cross-appeal" by a respondent or by a partywho might be a respondent to an appealfiled by another party, in proceedingsbefore the Boards of Appeal of the EPO,even if, which is an open question, theappeal procedure under the EPC permitsthe filing of a "cross-appeal".

For these reasons,

it is decided that:

1. The appeal is rejected as inadmissible.

2. The request for reimbursement of theappeal fee is rejected.

l´opposante avait formé un recours, laChambre aurait été tenue d´examiner laquestion de la recevabilité. Lorsqu´unechambre de recours examine un recours,elle doit toujours procéder à l´examend´office des faits (article 114 (1) de laCBE), et elle peut en conséquence réexa-miner toute question tranchée en pre-mière instance, y compris celle de larecevabilité d´une opposition, mêmesi cette question n´a pas été soulevéepar l´une des parties. Si elle procèdeà ce réexamen, il va de soi que toutesles parties concernées auront la possi-bilité de prendre position à ce sujet, con-formément au principe posé par lesarticles 110 (2) et 113 (1) de la CBE.Il ne semble donc pas nécessaire pourle bon déroulement de la procéduredevant les chambres de recours de l´OEBqu´un "recours incident" soit formé parun intimé ou par une partie qui pourraitêtre l´intimé dans un recours formé parune autre partie, même si ce qui estune autre question la procédure derecours prévue par la CBE permet laformation d´un "recours incident".

Par ces motifs,

il est statué comme suit:

1. Le recours est rejeté comme irreceva-ble.

2. La requête en remboursement de lataxe de recours est rejetée.

* Übersetzung. * Official text. * Traduction.

ENTSCHEIDUNGEN DERPRÜFUNGS- UND

EINSPRUCHSABTEILUNGEN

Entscheidung der Prüfungs-abteilung 125 vom5. Juni 1984* **

PCT Artikel 24, 25, 48EPÜ Artikel 121

LeitsatzI. Das EPA als Bestimmungsamt kann dieVersäumung von Fristen entschuldigen,die das Anmeldeamt gemäß Artikel14 (1) (b) i. V. m. Regel 26.2 PCT zurBehebung von Formmängeln gesetzt hat(Artikel 24 (2), 48 (2) (a) PCT).

DECISIONS OF THEEXAMINING AND OPPOSITION

DIVISIONS

Decision of the ExaminingDivision 125 of5 June 1984* **

PCT Articles 24, 25, 48EPC Article 121

Headnote

I. The EPO acting as designated Officecan excuse the non-observance of timelimits which have been set by theReceiving Office for the correction offormal deficiencies in accordance withArticle 14 (1) (b) in conjunction withRule 26.2 PCT (Articles 24 (2) and48 (2) (a) PCT).

DECISIONS DES DIVISIONSD´EXAMEN ET D´OPPOSITION

Décision de la divisiond´examen 125, en date du5 juin 1984* **

Articles 24, 25, et 48 du PCTArticle 121 de la CBE

SommaireI. L´OEB agissant en tant qu´officedésigné peut excuser la non-observa-tion de délais qui ont été impartis parl´Office récepteur conformément àl´article 14 (1) (b) ensemble larègle 26.2 du PCT pour la correction d´irré-gularités formelles (articles 24 (2) et48 (2) (a) du PCT).

* Übersetzung.

** Entscheidungen der Prüfungs- und Einspruchs-abteilungen werden veröffentlicht. wenn sieProbleme von allgemeinem Interesse behan-deln. rechtskräftig sind und entsprechendeEntscheidungen von Beschwerdekammern nochnicht vorliegen (vgl. ABI. 3/1984, S. 117).

* Official text.

** Decisions of the Examining and OppositionDivisions are reported where they deal withproblems of general interest, are final and thereare as yet no corresponding decisions byBoards of Appeal (cf. OJ 3/1984, p. 117).

* Traduction.

** Les décisions des divisions d´examen et d´oppo-sition sont publiés au JO OEB, lorsqu´elles por-tent sur des questions d´intérêt général,qu´elles sont passées en force de chose jugéeet que les chambres de recours n´ont pasencore eu à statuer en la matière (JO de l´OEB3/1984, p. 117).

Page 16: 22. November 1984 22 November 1984 22 novembre 1984 ...archive.epo.org/epo/pubs/oj1984/p549_604.pdf · tout Etat contractant doit, pour ce qui le concerne, excuser pour des motifs

II. Der Anmelder hat innerhalb der Zwei-monatsfrist nach Regel 51 PCT folgendeHandlungen vorzunehmen:a) Beim Internationalen Büro ist dieÜbersendung von Kopien aller Schrift-stücke aus den Akten der internationalenAnmeldung zu beantragen (Artikel 25(1) (a) PCT);

b) beim EPA ist die nationale Gebühr (imweiteren Sinn, d. h. die nationaleGebühr, die Benennungsgebühren unddie Recherchengebühr für eine euro-päische Recherche, Regel 104b (1) EPÜ)zu entrichten und ggf. eine Übersetzungder internationalen Anmeldung einzu-reichen (Artikel 25 (2) (a) PCT);

c) beim EPA ist gemäß Artikel 121 EPÜAntrag auf Weiterbehandlung der An-meldung einzureichen, die Weiterbe-handlungsgebühr zu entrichten und dieversäumte Handlung nachzuholen.

EntscheidungDen von der Anmelderin gestellten An-trägen vom 24. November 1983 aufNachprüfung der Entscheidung desjapanischen Patentamts als Anmeldeamt,nach der die internationale Anmeldungals zurückgenommen gilt, und vom 29.Februar 1984 auf Wiedereinsetzung indie Frist für die Stellung des Weiterbe-handlungsantrags wird stattgegeben.

Die Wirkung der internationalen Anmel-dung PCT/JP83/00250 als europäischePatentanmeldung (Nr. 83 902 449.4)für Frankreich wird gemäß Artikel 24 (2)PCT in Verbindung mit Artikel 25, Regel51.1 und 51.3 und Artikel 48 (2) a)PCT sowie den Artikeln 121 und 122EPÜ aufrechterhalten. Die Anmeldungwird der Eingangsstelle zur Weiterbe-handlung übermittelt.

SachverhaltI. Die internationale Anmeldung wurdeam 2. August 1983 beim japanischenPatentamt als Anmeldeamt eingereicht.Neben anderen Staaten war Frankreichin der Anmeldung bestimmt. Für die An-meldung wurde die Priorität einer japa-nischen Erstanmeldung vom 27. Juli1983 beansprucht. Das Antragsform-blatt war vom Vertreter unterzeichnet.Das Anmeldeamt forderte die Anmel-derin am 8. August 1983 auf, eine Voll-macht für den Vertreter vorzulegen. DieFrist hierfür lief am 8. September 1983ab (R. 80.2 PCT). Die Vollmacht wurdebeim japanischen Amt am 9. September1983, also einen Tag zu spät, ein-gereicht.

Das Anmeldeamt erklärte am 3. Oktober1983 die internationale Anmeldung fürzurückgenommen, weil innerhalb derFrist keine Vollmacht eingegangen ist(Art. 14 (1) a) und b), R. 26.2 PCT).Die normale Bearbeitung der interna-tionalen Anmeldung in der internatio-nalen Phase wurde eingestellt.

II. The applicant has to fulfil the follow-ing acts within the two-month timelimit under Rule 51 PCT:

(a) Request the International Bureau tosend copies of any document in the fileto the EPO as designated Office (Article25 (1) (a) PCT).

(b) Pay the EPO the national fee (i.e. in awider sense: the national fee, the desig-nation fees and the European searchfee), Rule 104b (1) EPC and, whereappropriate, file a translation of the inter-national application (Article 25 (2) (a)PCT).

(c) Request the EPO to allow furtherprocessing of the application, pay the feeand complete the omitted act in accord-ance with Article 121 EPC.

DecisionThe applicant´s request of 24 November1983 for review of the Decision of theJapanese Patent Office, acting asreceiving Office, which considered theinternational application withdrawn andthe request of 29 February 1984 forrestitutio in integrum into the time limitfor further processing are allowed.

The effect of international applicationPCT/JP83/00250 as a European patentapplication (No. 83 902 449.4) forFrance shall be maintained pursuant toArticle 24(2) PCT in conjunction withArticle 25, Rule 51.1 and 51.3, Article48 (2) (a) PCT, Article 121 EPC andArticle 122 EPC. The application shallbe transmitted to the Receiving Sectionfor further processing.

FactsI. The international application was filedon 2 August 1983 with the JapanesePatent Office acting as receiving Office.Among other countries, France wasdesignated in the application. The appli-cation claimed priority from a Japanesefirst filing from 27 July 1983. The re-quest form was signed by the represen-tative. The receiving Office invited theapplicant on 8 August 1983 to submitan authorisation for the representative.The time limit expired on 8 September1983 (Rule 80.2 PCT). The authorisa-tion was filed with the Japanese Officeon 9 September 1983, i.e. one day toolate.

The receiving Office declared the inter-national application withdrawn on 3October 1983 as no authorisation wasreceived within the time limit (Article14 (1) (a) and (b), Rule 26.2 PCT). Thenormal treatment of the internationalapplication in the international phasewas terminated.

II. Le déposant doit accomplir lesactes suivants dans un délai de deuxmois, tel que prévu à la règle 51 du PCT:

a) demander au Bureau internationald´adresser des copies de tout documentcontenu dans le dossier à l´OEB agissanten tant qu´office désigné (article 25 (1)(a) du PCT);

b) payer à l´OEB la taxe nationale (ausens large, à savoir la taxe nationale,les taxes de désignation et la taxe derecherche pour l´établissement du rap-port de recherche européenne; règle104ter (1) de la CBE) et, le caséchéant, déposer une traduction de lademande internationale (article 25 (2) (a)du PCT);

c) présenter à l´OEB conformément àl´article 121 de la CBE une requête enpoursuite de la procédure de la de-mande, payer la taxe et accomplirl´acte omis.

DécisionIl est fait droit à la requête de la de-manderesse en date du 24 novembre1983 visant à la revision de la décisionrendue par l´Office japonais des brevets,Office récepteur, qui avait considéré lademande internationale comme retirée,et à la requête en date du 29 février1984 visant à obtenir la restitutio inintegrum quant au délai fixé pourla présentation de la requête en pour-suite de la procédure.

Les effets en tant que demande debrevet européen (n° 83 902 449.4)produits pour la France par la de-mande internationale PCT/JP 83/00250sont maintenus en vertu des dispositionscombinées des articles 24.2) et 25 duPCT, de la règle 51, paragraphes 1 et 3et de l´article 48.2) a) du PCT ainsique des articles 121 et 122 de la CBE.La demande est transmise à la Sectionde dépôt aux fins de poursuite de laprocédure.

Exposé des faits

I. La demande internationale a étédéposée le 2 août 1983 auprès del´Office japonais des brevets agissanten qualité d´Office récepteur. Elle reven-diquait la priorité d´un premier dépôtjaponais en date du 27 juillet 1983.Entre autres pays, la France étaitdésignée dans la demande. Le formu-laire de requête avait été signé par lemandataire. Le 8 août 1983, l´Officerécepteur a invité la demanderesse àdéposer un pouvoir pour le mandatairedans un délai qui expirait le 8 septembre1983 (règle 80.2 du PCT). Le pouvoir aété déposé auprès de l´Office japonaisle 9 septembre 1983, autrement ditavec un jour de retard.

Le 3 octobre 1983, l´Office récepteura déclaré que la demande inter-nationale était considérée comme retiréeau motif que le pouvoir n´avait pas étéreçu dans le délai prescrit (article 14.1)a) et b), règle 26.2 du PCT), ce qui a misfin au traitement normal de la demandeinternationale dans la phase interna-tionale.

* Übersetzung. * Official text. * Traduction.

Page 17: 22. November 1984 22 November 1984 22 novembre 1984 ...archive.epo.org/epo/pubs/oj1984/p549_604.pdf · tout Etat contractant doit, pour ce qui le concerne, excuser pour des motifs

II. Die Anmelderin versuchte nun, dieWirkung der internationalen Anmeldungu. a. für Frankreich, für das das EPA Be-stimmungsamt war, aufrechtzuerhalten.Am 17. Oktober stellte die Anmelderinbeim Internationalen Büro in Genf denAntrag, dem EPA als Bestimmungsamtgemäß Artikel 25 (1) PCT Kopien derinternationalen Anmeldung und der dazu-gehörenden Unterlagen zu übersenden,was am selben Tag beschah.

Am 28. November 1983 reichte die An-melderin eine Übersetzung der Anmel-dung und der Vollmacht beim EPA einund entrichtete die nationale Gebühr(Anmeldegebühr) und die Benennungs-gebühr für Frankreich.Die Recherchengebühr wurde zu-nächst nicht in voller Höhe entrichtet:die Anmelderin zahlte jedoch später dienoch ausstehenden 20% der Recherchen-gebür sowie die Zuschlagsgebühr inHöhe von 50% gemäß Regel 85a EPÜnach.

III. Mit Schreiben vom 29. Februar 1984(Eingang 2. März 1984) stellte die An-melderin einen Antrag auf Weiterbe-handlung der internationalen Anmeldungund ersuchte darum, die durch das Frist-versäumnis bei der Einreichung der Voll-macht beim japanischen Anmeldeamteingetretenen Rechtsfolgen rückgängigzu machen. Gleichzeitig beantragte siedie Wiedereinsetzung in die zwei-monatige Frist für die Weiterbehandlungder Anmeldung (Art. 121 EPÜ) und ent-richtete am 2. März 1984 die ent-sprechenden Gebühren.

Gründe

1. Laut Artikel 24 (1) ii) PCT endet die inArtikel 11 (3) PCT vorgesehene Wirkungder internationalen Anmeldung in einemBestimmungsstaat mit den gleichenFolgen wie die Zurücknahme einernationalen Anmeldung, unter anderemwenn die internationale Anmeldung auf-grund des Artikels 14 (1) b) PCT als zu-rückgenommen gilt. Diese Rechtsfolgeist mit der Mitteilung des japanischenPatentamts als Anmeldeamt vom 3.Oktober 1983 eingetreten. Das EPA alsBestimmungsamt muß jedoch prüfen, obnicht Bestimmungen des PCT, die nachArtikel 150 (2) EPÜ anzuwenden sind,eine Fortsetzung des Verfahrens vor-schreiben.

Wenn die Bedingungen nach Artikel25 (2) PCT erfüllt sind, muß jedesBestimmungsamt die Wirkung der inter-nationalen Anmeldung als nationale(regionale) Patentanmeldung gemäßArtikel 11 (3) PCT aufrechterhalten.Darüber hinaus kann das Bestimmungs-amt diese Wirkung auch aus anderenGründen aufrechterhalten (Art. 24 (2)PCT). Den Bestimmungsämtern stehtes frei, dem Anmelder Vorteile zuzuge-stehen, die über seine im PCT fest-gelegten Rechte hinausgehen.

II. The applicant now tried to maintainthe effect of the international applicationi.a. for France for which the EPO wasdesignated Office. On 17 October theapplicant filed a request with the Inter-national Bureau in Geneva asking thelatter to send copies of documentsmaking up the international applicationto the EPO as designated Office pur-suant to Article 25 (1) PCT, which itaccordingly did on the same date.

On 28 November 1983 the applicantfiled a translation of the application andthe authorisation with the EPO and paidthe national fee (filing fee) and thedesignation fee for France.

The search fee was not paid at thecorrect amount, but the applicant laterpaid the missing 20% of the search feeand the surcharge of 50% in accordancewith Rule 85a.

III. By letter dated 29 February 1984(received 2 March 1984) the applicantfiled a request for further processingof the international patent applicationand asked that the legal consequencesof missing the time limit for filing theauthorisation before the Japanese Re-ceiving Office be retracted. At the sametime he requested restitutio in integruminto the time limit of two months forfurther processing (Article 121 EPC) andpaid the fees on 2 March 1984.

Reasons1. Under Article 24 (1) (ii) PCT, the effectof the international application providedfor in Article 11 (3) PCT ceases in anydesignated State with the same conse-quences as the withdrawal of anynational application in that State interalia if the international application isconsidered withdrawn by virtue ofArticle 14 (1) (b) PCT. This legal conse-quence ensued with the communicationdated 3 October 1983 from the Japa-nese Patent Office acting as receivingOffice. The EPO as designated Officemust, however, check whether provi-sions of the PCT, applicable underArticle 150 (2) EPC, do not prescribethat the proceedings be continued.

Where the conditions under Article25 (2) PCT are met, every designatedOffice must maintain the effect of theinternational application as a national(regional) patent application pursuant toArticle 11 (3) PCT. Beyond this obliga-tion, the designated Office may main-tain this effect for other reasons (Article24 (2) PCT). The designated Offices arenot prohibited from giving the appli-cant favourable treatment which goesbeyond his rights laid down in the PCT.

II. La demanderesse a alors tenté de fairemaintenir les effets de la demande inter-nationale, notamment pour la France,Etat pour lequel l´OEB était l´Officedésigné. Le 17 octobre, elle a déposéauprès du Bureau international àGenève une requête dans laquelle elledemandait à celui-ci de faire parvenirà l´OEB, Office désigné, des copies despièces de la demande internationaleconformément à l´article 25.1) du PCT,ce qui fut fait à cette même date.Le 28 novembre 1983, la demanderessea déposé une traduction de la demandeet le pouvoir auprès de l´OEB et aacquitté la taxe nationale (taxe dedépôt) ainsi que la taxe de désigna-tion pour la France.

La taxe de recherche n´a pas été inté-gralement acquittée, mais la deman-deresse a versé ultérieurement les 20%manquants de cette taxe, ainsi que lasurtaxe de 50%, conformément à larègle 85bis de la CBE.

III. Par une lettre datée du 29 février1984 (reçue le 2 mars 1984), lademanderesse a présenté une requêteen poursuite de la procédure relative àla demande internationale et a demandél´annulation de l´effet juridique pro-duit par le non-respect du délai prescritpour le dépôt du pouvoir auprès del´Office récepteur, à savoir l´Office japo-nais. Elle a présenté en même tempsune requête en restitutio in integrumquant au délai de deux mois fixé pourla présentation de la requête en pour-suite de la procédure (article 121 de laCBE) et a acquitté les taxes correspon-dantes le 2 mars 1984.

Motifs1. Aux termes de l´article 24.1) ii) duPCT, les effets de la demande interna-tionale prévus à l´article 11.3) du PCTcessent dans tout Etat désigné et cettecessation a les mêmes conséquencesque le retrait d´une demande nationaledans cet Etat, notamment si la demandeinternationale est considérée commeretirée en raison de l´article 14.1) b)du PCT. Cet effet juridique a été produitpar la notification établie le 3 octobre1983 par l´Office japonais des brevetsagissant en qualité d´Office récepteur.Toutefois l´OEB, étant l´Office désigné,doit vérifier s´il n´existe pas dans le PCTdes dispositions, applicables au titre del´article 150 (2) de la CBE, qui prescri-vent la poursuite de la procédure.

Lorsque les conditions prévues àl´article 25.2) du PCT sont remplies,tout office désigné est tenu de main-tenir les effets de la demande interna-tionale en tant que demande de brevetnationale (régionale) visés à l´article11.3) du PCT. Outre cette obligation,l´office désigné peut maintenir ces effetspour d´autres motifs (article 24.2) duPCT). Il n´est pas interdit aux officesdésignés d´accorder au demandeur untraitement plus favorable que celuirésultant des droits conférés par lePCT.

* Übersetzung. * Official text. * Traduction.

Page 18: 22. November 1984 22 November 1984 22 novembre 1984 ...archive.epo.org/epo/pubs/oj1984/p549_604.pdf · tout Etat contractant doit, pour ce qui le concerne, excuser pour des motifs

2. Artikel 25 (2) PCT verpflichtet dasBestimmungsamt, die Entscheidung desAnmeldeamts nachzuprüfen und dieWirkung der Anmeldung aufrechtzu-erhalten, wenn die Entscheidung "aufeine versehentliche Maßnahme oderUnterlassung ... zurückzuführen" ist.Dies trifft im vorliegenden Fall nicht zu,da das japanische Patentamt als Anmel-deamt zu Recht festgestellt hat, daß dieVollmacht erst nach Ablauf der fest-gesetzten Frist eingereicht worden ist.Die Erklärung, daß die Anmeldung alszurückgenommen gilt, war deshalbrichtig.

3. Zu prüfen bleibt, ob die Bedingungenfür die Anwendung des Artikels 24 (2)PCT erfüllt sind. Diese Vorschrift ermächtigtdie Bestimmungsämter, die Wirkung derinternationalen Anmeldung als nationaleAnmeldung aufrechtzuerhalten. Bei derAusübung des Ermessens wird in ersterLinie das vor dem jeweiligen Bestim-mungsamt geltende nationale Rechtmaßgebend sein. Denkbar ist aber auch,daß andere Bestimmungen des PCT inihrem jeweiligen Zusammenhang einepräzise Richtschnur für die Ausübungdes in Artikel 24 (2) PCT vorgesehenenErmessens vorgeben. Diese Vorgabekann so eng sein, daß sie den Ermessens-spielraum einschränkt und das Bestim-mungsamt verpflichtet, die Wirkung derinternationalen Anmeldung unter be-stimmten Umständen aufrechtzuerhalten.

Aus Artikel 48 (2) a) PCT ergibt sich fürjeden Vertragsstaat, soweit er betroffenist, die Verpflichtung, eine Fristüber-schreitung als entschuldigt anzusehen,wenn dafür Gründe vorliegen, die nachseinem nationalen Recht zugelassen sind.Ein Vertragsstaat ist "betroffen" undsein Bestimmungsamt zur entsprechen-den Behandlung der internationalen An-meldung verpflichtet, wenn für diesenStaat ein Verlust der Wirkung einernationalen (regionalen) Anmeldunggemäß Artikel 11 (3) PCT eintritt. Dieterritoriale Wirkung der internationalenAnmeldung ist der Schlüssel zur Aus-legung des Begriffs "betroffen" in Artikel48 (2) a) PCT. Das von der Behörde desVertragsstaats anzulegende Kriterium,nämlich, ob "Gründe vorliegen, die nachseinem nationalen Recht zugelassensind", ist präzise genug und kann auchohne Durchführungsvorschriften aufnationaler Ebene unmittelbar angewandtwerden. Wenn solche nach dem natio-nalen (regionalen) Recht zugelassenenGründe gemäß Artikel 48 (2) a) PCTvorliegen, muß das Bestimmungsamteine Fristüberschreitung "als entschul-digt ansehen".Unter den Begriff "Frist" fallen alle vomAnmelder nach dem PCT einzuhaltendenFristen. Dazu gehören auch die Fristenvor dem Anmeldeamt. Da umfassendeRechtsbehelfe in der internationalenPhase fehlen, ist Artikel 48 (2) a) PCT fürdie Rechtssicherheit auf dem PCT-Wegvon grundlegender Bedeutung und mußdeshalb weit ausgelegt werden (vgl. Ent-scheidung der Juristischen Beschwer-dekammer J 05/80, ABI. EPA 9/1981,

2. Article 25 (2) PCT obliges the desig-nated Office to review the decision of thereceiving Office and to maintain theeffect of the application if the decision"was the result of an error or omis-sion". This does not apply in the presentcase, because the Japanese PatentOffice as receiving Office has rightlyfound that the authorisation was filedafter expiration of the set time limit. Thedeclaration that the application isdeemed withdrawn was thereforecorrect.

3. It still remains to be examinedwhether the conditions for the applica-tion of Article 24 (2) PCT are met. Thisprovision contains an authorisation onthe part of the designated Offices tomaintain the effect of the internationalapplication as a national application. Itwill be primarily national law before thedesignated Office which governs theexercise of discretion. But it is also con-ceivable that other provisions of thePCT in their context give specificguidance for exercising discretion underArticle 24 (2) PCT. This guidance maybe specific to such an extent that itlimits the discretion and imposes anobligation on the designated Office tomaintain the effect of the internationalapplication under certain circumstances.

From Article 48 (2) (a) PCT stems the obli-gation for any Contracting State, as faras that State is concerned, to excuse, forreasons admitted under its national law,any delay in meeting any time limit. AContracting State is to this extent"concerned" and its designated Officeobliged to treat the international appli-cation accordingly when the effect of anational (regional) application pur-suant to Article 11 (3) PCT is lost for thatState. The territorial effect of the inter-national application is the key in inter-preting the notion of "concerned" inArticle 48 (2) (a) PCT. The criterion to beapplied by the authority of the Contract-ing State, namely whether there are"reasons admitted under its nationallaw", is sufficiently specific and can begiven effect directly even without imple-menting regulations on the nationallevel. If there are such reasons pursuantto Article 48 (2) (a) PCT under national(regional) law, the designated Officemust "excuse" any delay in meeting anytime limit.

The notion "any time limit" encompassesall time limits to be monitored by theapplicant according to the PCT. Itincludes time limits before the receivingOffice. In the absence of comprehensivelegal remedies in the internationalphase, Article 48 (2) (a) PCT is of funda-mental importance for the legalsecurity on the PCT-route and musttherefore be given a wide interpretation(cf. Legal Board of Appeal J 05/80, OJ

2. L´article 25.2) du PCT dispose quel´office désigné doit réviser la décisionde l´office récepteur et maintenir leseffets de la demande si la décision "estle résultat d´une erreur ou d´une omis-sion". Cette disposition n´est pas appli-cable en l´espèce, étant donné quel´Office japonais des brevets, agissanten qualité d´Office récepteur, a constatéavec raison que le pouvoir avait étédéposé après l´expiration du délaiprescrit. C´est donc à juste titre qu´il adéclaré que la demande était réputéeretirée.

3. Reste à examiner si les conditionsrequises pour l´application de l´article24.2) du PCT sont remplies. Aux termesde cet article, les offices désignés peu-vent maintenir les effets de la demandeinternationale en tant que demandenationale. L´exercice de leur pouvoird´appréciation est régi en premier lieupar la législation nationale de l´Etat del´office désigné. Mais il n´est pas im-possible non plus que, considérées dansleur contexte, d´autres dispositions duPCT donnent des indications particu-lières pour l´exercice du pouvoir d´appré-ciation prévu à l´article 24.2) du PCT.Dans certains cas, ces indications peu-vent être précises au point de limiter cepouvoir d´appréciation et de faire obli-gation à l´office désigné de maintenirles effets de la demande internationale.

Aux termes de l´article 48.2) a) du PCT,tout Etat contractant doit, pour ce qui leconcerne, excuser pour des motifsadmis par sa législation nationale toutretard dans l´observation d´un délai. Acet égard, un Etat contractant est réputé"concerné" et son office est tenu, enqualité d´office désigné, de traiter lademande internationale en conséquence,lorsque les effets d´un dépôt national(régional) conférés à cette demande envertu de l´article 11.3) du PCT cessentpour cet Etat. La portée territoriale de lademande internationale constitue unélément clé pour l´interprétation de l´ex-pression "pour ce qui le concerne" figur-ant à l´article 48.2) a) du PCT. Le critère surlequel doit se fonder l´administration del´Etat contractant, à savoir l´existence de"motifs admis par sa législation natio-nale", est suffisamment précis pour pou-voir être directement appliqué, mêmeen l´absence de règlement d´exécutionau niveau national. S´il existe des motifsadmis par sa législation nationale(régionale) visés à l´article 48.2) a) duPCT, l´office désigné doit "excuser" toutretard dans l´observation d´un quel-conque délai.La notion de "délai" englobe tous lesdélais que le demandeur doit veiller àrespecter, conformément aux disposi-tions du PCT, y compris les délais àobserver vis-à-vis de l´office récep-teur. En l´absence de remèdes juridiquessuffisants dans la phase internationale,l´article 48.2) a) du PCT revêt une im-portance fondamentale pour la sécuritéjuridique de la voie du PCT et doit parconséquent être interprété au sens large

* Übersetzung. * Official text. * Traduction.

Page 19: 22. November 1984 22 November 1984 22 novembre 1984 ...archive.epo.org/epo/pubs/oj1984/p549_604.pdf · tout Etat contractant doit, pour ce qui le concerne, excuser pour des motifs

343). Um der Verpflichtung zur "Ent-schuldigung" der Fristüberschreitung vordem Anmeldeamt nachzukommen, mußdas Bestimmungsamt von den in Artikel24 (2) PCT vorgesehenen MöglichkeitenGebrauch machen. Artikel 24 (2) PCTräumt hier kein Ermessen ein, sonderndient als Grundlage und Verfahrens-mechanismus für die Wahrnehmung derVerpflichtung aus Artikel 48 (2) a)PCT.

4. Welche allgemeinen verfahrensrecht-lichen Voraussetzungen im Rahmen desArtikels 24 (2) PCT erfüllt sein müssen,geht indirekt aus dem Verweis aufArtikel 25 PCT ("... auch wenn dieseWirkung auf Grund des Artikels 25 Ab-satz 2 nicht aufrechterhalten werdenmuß") hervor. Artikel 24 (2) PCT enthältkeine näheren Bestimmungen darüber,wie der Eintritt in die nationale (regio-nale) Phase vor dem EPA als Bestim-mungsamt erfolgen soll. Deshalb ist dieseBestimmung als Unterfall zu Artikel25 (2) PCT anzusehen: die darin er-wähnte Feststellung, daß die angefoch-tene Entscheidung über die Zurücknahmeder internationalen Anmeldung auf eineversehentliche Maßnahme oder eineUnterlassung zurückzuführen ist, kannim vorliegenden Fall nicht getroffenwerden.

Die verfahrensrechtlichen Erfordernisse,nämlich (a) die fristgerechte Antrag-stellung durch den Anmelder nach Artikel25 (1) c) und Regel 51.1 PCT und (b)die Einhaltung der in Artikel 25 (2) a)PCT genannten nationalen Erfordernissefür den Eintritt in die nationale (regionale)Phase (d. h. Entrichtung der nationalenGebühr innerhalb der vorgeschriebenenFrist R. 51.3 PCT und, wie indiesem Fall, Einreichung einer Über-setzung) müssen also erfüllt sein.

5. Mit ihrem an das InternationaleBüro in Genf gerichteten Antrag vom 17.Oktober 1983 hat die Anmelderin dieErfordernisse des Artikels 25 (1) undder Regel 51.1 PCT erfüllt. Sie hat demEPA als Bestimmungsamt auch die Über-setzung der internationalen Anmeldunginnerhalb der vorgeschriebenen Fristübermittelt (Art. 25 (2) a) und R. 51.3PCT). Die Anmelderin hat ferner dienationale Gebühr entrichtet, und zwarzum Teil innerhalb dieser Frist und zumTeil zusammen mit einer Zuschlags-gebühr innerhalb einer nach demEPÜ in vergleichbaren Verfahrenssitua-tionen zugelassenen Nachfrist (R. 85aEPÜ).

6. Wenn beurteilt werden soll, ob Gründegemäß Artikel 48 (2) PCT vorliegen, dienach dem EPÜ das gemäß Artikel2 x) PCT das "nationale Recht" vor demEPA ist eine Fristüberschreitung "ent-schuldigen", müssen die Verfahrens-schritte nach dem PCT und dem EPÜverglichen werden. Sind Ähnlichkeitenmit dem EPÜ festzustellen, so muß dem

9/1981, 343). In order to give effect to theobligation to "excuse" the delay in meet-ing a time limit before the receivingOffice, the designated Office mustavail itself of the power given to it underArticle 24 (2) PCT. In this area Article24 (2) is not a matter for discretion buta basis and procedural mechanism forcarrying out the obligation stemmingfrom Article 48 (2) (a) PCT.

4. What general procedural conditionsmust be fulfilled under Article 24 (2) PCTemerges indirectly from thereference to Article 25 PCT in thisprovision (by the words "... even wheresuch effect is not required to be main-tained by virtue of Article 25 (2)").Article 24 (2) PCT contains no detailedprovisions on how the national (regional)phase is to be entered before the EPOas designated Office. Therefore thisprovision must be seen as a sub-category of Article 25 (2) PCT whereinthe finding that the contested decisionto withdraw the international applicationwas the result of an error or omission isnot germane to the case.

The procedural requirements, namely(a) the request in due time by the appli-cant under Article 25 (1) (c) and Rule51.1 PCT and (b) compliance with thenational requirements specified in Article25 (2) (a) PCT for the entry of thenational (regional) phase (i.e. paymentof the national fee within the prescribedtime limit Rule 51.3 PCT and, asin this case, filing of a translation) musttherefore be met.

5. With his request of 17 October 1983addressed to the International Bureau inGeneva. the applicant complied withArticle 25 (1) and Rule 51.1 PCT. Theapplicant furthermore furnished thetranslation of the international applica-tion to the EPO as designated Officewithin the prescribed period (Article25 (2) (a) and Rule 51.3 PCT). He alsopaid the national fee partly within thistime limit, partly together with a sur-charge within a period of graceallowed under the EPC under com-parable procedural situations (Rule 85aEPC).

6. In assessing whether there arereasons pursuant to Article 48 (2) PCT"excusing" under the .EPC which isthe "national law" before the EPOaccording to Article 2 (x) PCT anydelay in meeting a time limit, a com-parison must be made of the proceduralsteps under the PCT and EPC. If thereare found to be similarities with the

(cf. décision de la chambre de recoursjuridique J 05/80, JO n° 9/1981,p 343). Pour satisfaire à l´obligationd´ "excuser" le retard dans l´observa-tion d´un délai vis-à-vis de l´office récep-teur, l´office désigné doit, quant à lui,faire usage de la faculté qui lui estdonnée aux termes de l´article 24.2) duPCT. A cet égard, l´article 24.2) ne relèvepas du pouvoir d´appréciation; il sertde fondement et règle le mécanisme dela procédure à suivre pour s´acquitter del´obligation prévue à l´article 48.2) a) duPCT.

4. La référence faite à l´article 24.2) duPCT aux dispositions de l´article 25 duPCT (cf. les termes "... même lorsqu´iln´est pas exigé que de tels effets soientmaintenus en raison de l´article 25.2)")permet indirectement de savoir quellessont les conditions générales de procé-dure à remplir pour l´application de cetarticle 24.2). L´article 24.2) du PCT neprécise pas comment doit s´effectuerl´entrée dans la phase nationale (régio-nale) auprès de l´OEB agissant en tantqu´Office désigné. Il convient donc deconsidérer cette disposition comme undes cas d´application de l´article 25.2) duPCT, distinct du cas dans lequel il estconstaté que la décision contestée pro-noncant le retrait de la demande inter-nationale est le résultat d´une erreurou d´une omission.

Les conditions relatives à la procédure,à savoir a) la présentation de la requêtepar le demandeur dans le délai prescritpar l´article 25.1) c) et par la règle 51.1du PCT et b) le respect des conditionsénoncées à l´article 25.2) a) du PCTqui sont requises au niveau national pourl´entrée dans la phase nationale (régio-nale) (à savoir paiement de la taxenationale dans le délai prescrit règle51.3 du PCT et, comme c´est le casen l´espèce, dépôt d´une traduction)doivent donc être remplies.

5. Par sa requête en date du 17 octobre1983 adressée au Bureau internationalà Genève, la demanderesse s´est con-formée aux dispositions de l´article25.1) et de la règle 51.1 du PCT. Enoutre, elle a, dans le délai prescrit(article 25.2) a) et règle 51.3 du PCT),remis la traduction de la demande inter-nationale à l´OEB, Office désigné. Elle apar ailleurs acquitté dans ce délai unepartie de la taxe nationale et acquittél´autre partie assortie d´une surtaxe

dans le délai supplémentaire accordépar la CBE dans des situations com-parables (règle 85bis de la CBE).

6. Pour déterminer si, comme il estrequis à l´article 48.2) du PCT, il existedes motifs admis par la CBE qui estla "législation nationale" applicabledevant l´OEB, en vertu de l´article 2.x) duPCT "excusant" tout retard dans l´ob-servation d´un délai, il convient de com-parer les étapes de la procédure prévuespar le PCT et celles prévues par la CBE.

* Übersetzung. * Official text. * Traduction.

Page 20: 22. November 1984 22 November 1984 22 novembre 1984 ...archive.epo.org/epo/pubs/oj1984/p549_604.pdf · tout Etat contractant doit, pour ce qui le concerne, excuser pour des motifs

PCT-Anmelder ein gleichwertiger Schutzzugestanden und unter ähnlichenUmständen die Möglichkeit zur Be-seitigung der eingetretenen Rechtsfolgengeboten werden. Die Worte "Gründe ...,die nach seinem nationalen Recht zu-gelassen sind" sind als allgemeiner Hin-weis auf eine vergleichbare Situation imnationalen (regionalen) Recht (im vor-liegenden Fall also dem EPÜ) auszu-legen.

7. Als Rechtsbehelfe bei Fristüberschrei-tungen sind im EPÜ insbesondere dieWeiterbehandlung der Anmeldung (Art.121 EPÜ) und die Wiedereinsetzung inden vorigen Stand (Art. 122 EPÜ) vor-gesehen.

Im vorliegenden Fall hat die Anmelderinin der internationalen Phase die Fristnach Regel 26.2 PCT versäumt. DieseFrist wird von der zuständigen Behörde(dem Anmeldeamt) entsprechend denUmständen des Einzelfalls auf minde-stens einen und normalerweise höchstenszwei Monate ab dem Zeitpunkt der Auf-forderung zur Mängelbeseitigung fest-gesetzt (R. 26.2 PCT).

Sie ist deshalb mit den vom Euro-päischen Patentamt festgesetzten Fristen,für die Artikel 121 EPÜ gilt, zu ver-gleichen. Die in Artikel 14 (1) b) PCTvorgesehene Rechtsfolge der Zurück-nahme der internationalen Anmeldungbei Überschreitung der Frist ist deshalbdurch entsprechende Anwendung desArtikels 121 EPÜ rückgängig zu machen.

8. Eine weitere Voraussetzung für dieAnwendung des Artikels 121 EPÜ ist,daß der Anmelder innerhalb von zweiMonaten nach dem Tag, an dem die Ent-scheidung über die Zurückweisung derAnmeldung oder an dem die Mitteilung,daß die Anmeldung als zurückgenommengilt (R. 69 (1) EPÜ), zugestellt wordenist, den Weiterbehandlungsantrag stellt,die entsprechende Gebühr entrichtetund die versäumte Handlung nachholt.

Die Mitteilung des japanischen Patent-amts als Anmeldeamt vom 3. Oktober1983, daß die Anmeldung nach Artikel14 (1) b) PCT als zurückgenommen gilt,ist als Mitteilung gemäß Regel 69 (1)EPÜ zu behandeln.

Ausgehend vom Datum der Mitteilungdes japanischen Patentamts als "maß-gebliches Ereignis" berechnet sich dieFrist für die Stellung des Antrags aufWeiterbehandlung der Anmeldung gemäßArtikel 121 EPÜ nach Regel 83 (4) inVerbindung mit Regel 85 EPÜ. Im vor-liegenden Fall lief daher die zweimonatigeFrist am 5. Dezember 1983 ab (der 3.Dezember 1983 war ein Samstag).

EPC, the PCT applicant must be allowedequivalent protection and, on similarconditions, offered the possibility ofhaving the legal consequences which haveensued retracted. The words "reasonsadmitted under ... national law" are tobe interpreted as a general referenceto a comparable situation in national(regional) law (the EPC in the presentcase).

7. Legal remedies in the EPC in respectof delays in meeting time limits are inparticular further processing of the patentapplication (Article 121 EPC) and re-establishment of rights (Article 122EPC).

In the case in question the applicant hasmissed, in the international phase, thetime limit under Rule 26.2 PCT. Thistime limit is fixed by the competentauthority (the receiving Office), accord-ing to the circumstances of the particularcase, at between one month and normallynot more than two months from thedate of invitation to correct the defect(Rule 26.2 PCT).

It is therefore to be compared withtime limits set by the European PatentOffice to which Article 121 EPC applies.The legal consequence in Article 14(1) (b) PCT of withdrawal of the inter-national application flowing from non-compliance with the time limit musttherefore be retracted by applyingArticle 121 EPC mutatis mutandis.

8. The application of Article 121 EPCfurther presupposes that the applicantfiles the request for further processing,pays the relevant fee and completes theomitted act within two months of thedate on which either the decision torefuse the application, or the communi-cation that the application is deemed tobe withdrawn, was notified (Rule 69 (1)EPC).

The communication dated 3 October1983 from the Japanese Patent Officeas receiving Office concerning theeffect of withdrawal pursuant to Article14 (1) (b) PCT is to be treated as acommunication pursuant to Rule 69 (1)EPC.

Taking as the "relevant event" the dateof the communication from the JapanesePatent Office as receiving Office, furthercomputation of the time limit for filingthe request for further processing pur-suant to Article 121 EPC is determinedby Rule 83 (4) EPC in conjunction withRule 85 EPC. In the present case thetwo-month time limit therefore expiredon 5 December 1983 (3 December1983 was a Saturday).

Si l´on constate qu´il existe des simili-tudes avec la CBE, le déposant autitre du PCT doit bénéficier d´une pro-tection équivalente et, dans des cir-constances similaires, avoir la possi-bilité de faire annuler les effets juridiquesqui ont été produits. Les termes"motifs admis par ... sa législationnationale" doivent être interprétéscomme une référence de portée généraleà une situation comparable prévue dansune législation nationale (régionale)(la CBE, en l´occurrence).

7. Les remèdes juridiques prévus par laCBE en ce qui concerne les retards dansl´observation des délais sont notammentla poursuite de la procédure de la de-mande de brevet (article 121 de la CBE)et la restitutio in integrum (article 122de la CBE).

Dans la présente espèce, la deman-deresse n´a pas observé, au cours dela phase internationale, le délai prévupar la règle 26.2 du PCT. Ce délai estfixé par l´autorité compétente (l´officerécepteur) compte tenu des circons-tances du cas d´espèce; il est d´un moisau moins et, normalement, de deuxmois au plus, à compter de la date del´invitation à corriger l´irrégularité(règle 26.2 du PCT).

Ce délai est donc comparable auxdélais fixés par l´Office européen desbrevets auxquels l´article 121 de la CBEest applicable. En conséquence, le retraitde la demande internationale, effet juri-dique prévu à l´article 14.1) b) du PCTen cas de non respect du délai prescrit,doit être annulé par application del´article 121 de la CBE.

8. L´application de l´article 121 de laCBE suppose en outre que le demandeurprésente la requête en poursuite de laprocédure, acquitte la taxe correspon-dante et exécute l´acte non accomplidans un délai de deux mois à compterde la date à laquelle la décision derejet de la demande de brevet européena été signifiée, ou à compter dela date à laquelle la notification que lademande est réputée retirée a été signi-fiée (règle 69 (1) de la CBE).

La notification par laquelle l´Office japo-nais des brevets agissant en qualitéd´Office récepteur a constaté le 3octobre 1983 que la demande étaitconsidérée comme retirée en vertu del´article 14.1) b) du PCT doit être traitéecomme une notification établie en vertude la règle 69 (1) de la CBE.

Le délai de présentation de la requêteen poursuite de la procédure visée àl´article 121 de la CBE se calcule parapplication de la règle 83 (4) ensemblela règle 85 de la CBE en prenant comme"point de départ" la date de la notifica-tion établie par l´Office japonais desbrevets agissant en tant qu´Office récep-teur. Dans la présente espèce, le délaide deux mois expirait donc le 5 décem-bre 1983 (le 3 décembre 1983 étantun samedi).

* Übersetzung. * Official text. * Traduction.

Page 21: 22. November 1984 22 November 1984 22 novembre 1984 ...archive.epo.org/epo/pubs/oj1984/p549_604.pdf · tout Etat contractant doit, pour ce qui le concerne, excuser pour des motifs

9. Es stellt sich nun die Frage, wie sichdie Verfahrensfrist für den Rechtsbehelfgemäß Artikel 121 EPÜ (Weiterbehand-lung der Anmeldung) in den Rahmendes PCT einfügt. Das Verfahren beimnormalen Übergang in die nationalePhase (Art. 22 und 23 PCT) muß mitFällen verglichen werden, in denen dasinternationale Verfahren als Folge einerErklärung, daß die Anmeldung alszurückgenommen gilt, vorzeitig abge-brochen wird. Im "Normalfall" bildet dasEnde der Frist für die Erfüllung dernationalen Erfordernisse (Art. 22 und39 (1) PCT) die Trennlinie. Bis zu diesemZeitpunkt dürfen die Bestimmungsämternach Art. 23 (1) und 40 (1) PCT die in-ternationale Anmeldung nicht bearbeitenoder prüfen. Diese Bestimmungen sindauch so auszulegen, daß keine nationale(regionale) Frist vor der Frist nachArtikel 22 PCT ablaufen darf. Das EPÜschreibt ausdrücklich vor, daß im Kon-fliktfall die Bestimmungen des PCT maß-gebend sind, und nennt als Beispiel dieverlängerte Frist für die Stellung desPrüfungsantrags nach Artikel 94 (2)EPÜ (Art. 150 (2) EPÜ). Ein weiteres, imEPÜ nicht ausdrücklich erwähntes Bei-spiel ist die Jahresgebühr (vgl. Hinweisefür PCT-Anmelder, ABI. EPA 3/1979,110).

Aus diesen Bestimmungen in ihremjeweiligen Zusammenhang läßt sich derfolgende allgemeine Grundsatz ableiten:Keine Frist nach dem EPÜ darf vor derdurch den PCT festgelegten Frist für dieErfüllung der nationalen Erfordernissebei den Bestimmungsämtern ablaufen.

Im Falle eines vorzeitigen Abbruchs desinternationalen Verfahrens endet dieinternationale Phase mit dem Ablaufder Frist nach Regel 51.1 und 51.3 PCT.Die Frist für die Stellung des Weiterbe-handlungsantrags darf also nicht vorder durch diese Bestimmung fest-gesetzten Frist ablaufen. Zufällig decktsich die Frist nach Artikel 121 mit dernach Regel 51.1 und 51.3 PCT. DieFristen werden durch dasselbe Ereignis(Mitteilung des Rechtsverlusts) in Ganggesetzt und sind gleich lang (zweiMonate). In Übereinstimmung mit Arti-kel 150 (2) EPÜ und Regel 51.1 und51. 3 PCT kann deshalb verlangt werden,daß der Antrag innerhalb von zweiMonaten nach Zustellung der Erklärungdes Anmeldeamts, daß die internationaleAnmeldung als zurückgenommen gilt,gestellt wird.

10. Die Anmelderin hat die zwei-monatige Frist für die Stellung des Wei-terbehandlungsantrags nach Artikel 121EPÜ und Regel 51.1 PCT nicht einge-halten. Eine Überschreitung dieser Fristist nach Artikel 48 (2) a) PCT entschuld-

9. Now the question arises how the pro-cedural time limit for the remedy underArticle 121 EPC (further processing)fits in the procedural framework of thePCT. The procedural situation of normalentries into the national phase (Articles22 and 23 PCT) must be compared withcases of an early break-up of the inter-national application ensuing from adeclaration that the application is deemedwithdrawn. In the "normal" situation itis the end of the time limit for fulfillingthe national requirements (Articles 22and 39 (1) PCT) which marks the demar-cation. Up to this point in time desig-nated Offices must not process orexamine the international application(Articles 23 (1) and 40 (1) PCT). Theseprovisions are construed as implyingthat no national (regional) time limitmay expire prior to the time limit underArticle 22 PCT. The EPC specificallystates that in cases of conflict the pro-visions of the PCT prevail and quotes theprolonged period for filing the request forexamination under Article 94 (2) EPC asan example (Article 150 (2) EPC).Another example, not expressly men-tioned in the EPC, is the annual fee (cf.Notice for PCT-applicants OJ 3/1979,110).

From this set of provisions in their con-text the following general principle canbe deduced: no time limit set under theEPC may expire prior to the time limit setby the PCT for the fulfillment of nationalrequirements before the designatedOffices.

In case of early break-up of the interna-tional procedure the expiry of the timelimit under Rule 51.1 and 51.3 PCTmarks the end of the internationalphase. The time limit for filing therequest for further processing maytherefore not expire prior to thetime limit set by this provision. Bychance, the time limits under Article 121on the one hand and Rule 51.1 and 51.3PCT coincide. They start from the sameevent (notification of loss of right) andhave the same length (two months).It is therefore in accordance withArticle 150 (2) EPC and Rule 51.1 and51.3 to require the request to be filedwithin two months from the notificationof the receiving Office declaring theinternational application to be deemedwithdrawn.

10. The applicant did not meet the twomonths´ time limit for the request forfurther processing under Article 121EPC and Rule 51.1 PCT. This time limitis also amenable to an "excuse" underArticle 48 (2) (a) PCT. Here the com-

9. La question se pose maintenant desavoir si dans la procédure le délai fixépour la mise en oeuvre du remède juri-dique prévu à l´article 121 de la CBE(poursuite de la procédure) cadre avecla procédure du PCT. Il convient decomparer la procédure normale d´en-trée des demandes internationales dansla phase nationale (articles 22 et 23 duPCT) avec les cas où la procédure inter-nationale prend fin prématurément à lasuite d´une notification déclarant queladite demande est réputée retirée. Dans laprocédure "normale", c´est l´expirationdu délai requis pour remplir les condi-tions d´entrée dans la phase nationale(articles 22 et 39.1) du PCT) qui marquela limite entre la phase internationale etla phase nationale. Jusqu´à cette date,les offices désignés ne doivent nitraiter ni examiner la demande interna-tionale (articles 23.1) et 40.1) du PCT).Ces dispositions sont interprétées commesignifiant qu´un délai national (régional)ne peut venir à expiration avant l´expira-tion du délai prévu à l´article 22 du PCT.La CBE stipule expressément que lesdispositions du PCT prévalent en casde divergence et cite à titre d´exemple laprolongation du délai fixé par l´article94 (2) de la CBE pour la formulation dela requête en examen (article 150 (2)de la CBE). La taxe annuelle constitueun autre exemple, bien qu´il ne soit pasmentionné expressément dans la CBE(cf. Avis aux déposants PCT, JO n°3/1979, p. 110).A partir de cet ensemble de dispositionsconsidérées dans leur contexte, on peutposer le principe général suivant: undélai fixé par la CBE ne peut venir àexpiration avant le délai imparti par lePCT pour remplir les conditions d´entréedans la phase nationale auprès desoffices désignés.Dans le cas où la procédure relative àune demande internationale prend fin pré-maturément, l´expiration du délai fixépar la règle 51, paragraphes 1 et 3 duPCT marque la fin de la phase interna-tionale. Le délai prescrit pour présenterla requête en poursuite de la procédurene peut par conséquent venir à expira-tion avant le délai fixé par cette règle. Il setrouve que le délai fixé par l´article 121 dela CBE coïncide avec celui prévu par larègle 51, paragraphes 1 et 3 du PCT.Ils ont pour point de départ le mêmefait (notification de la perte d´un droit)et ont la même durée (deux mois). Con-formément à l´article 150 (2) de la CBEet à la règle 51, paragraphes 1 et 3 duPCT, il convient donc d´exiger que cetterequête soit présentée dans un délai dedeux mois à compter de la notificationpar laquelle l´Office récepteur déclareque la demande internationale est ré-putée retirée.

10. La demanderesse n´a pas observé ledélai de deux mois prescrit pour pré-senter la requête en poursuite de laprocédure, aux termes de l´article 121de la CBE et de la règle 51.1 du PCT. Envertu de l´article 48.2) a) du PCT, le non-

* Übersetzung. * Official text. * Traduction.

Page 22: 22. November 1984 22 November 1984 22 novembre 1984 ...archive.epo.org/epo/pubs/oj1984/p549_604.pdf · tout Etat contractant doit, pour ce qui le concerne, excuser pour des motifs

bar. Hier ist der vergleichbare Rechtsbe-helf nach dem EPÜ die Wiedereinsetzungin den vorigen Stand.

11. Die Anmelderin hat mit Schreibenvom 29. Februar 1984 (Eingang 2.März 1984) die Wiedereinsetzung in diezweimonatige Frist für die Weiterbe-handlung der Anmeldung beantragt undam 2. März 1984 die entsprechendenGebühren entrichtet. Der Wegfall desHindernisses, das der Einhaltung derFrist entgegenstand, war die Mitteilungdes EPA vom 28. Februar 1984, daß dieWeiterbehandlung der Anmeldung be-antragt werden muß und die Frist nachArtikel 121 (2) EPÜ abgelaufen ist. DieAnmelderin hat auch die versäumteHandlung nachgeholt, d. h. den Antragauf Weiterbehandlung der Anmeldunggestellt und die Weiterbehandlungsge-bühr entrichtet. Der Antrag auf Wieder-einsetzung in den vorigen Stand istdamit rechtzeitig gestellt worden (Art.122 (2) EPÜ).

In Anbetracht der komplizierten ver-fahrensrechtlichen Situation (vgl. Ent-scheidung der Juristischen Beschwerde-kammer J 06/79, ABI. EPA 1980, 225)belegen die vom Vertreter vorge-brachten Gründe und die zur Begrün-dung dienenden Tatsachen, daß dienach den gegebenen Umständen ge-botene Sorgfalt beachtet worden ist.Da dem Antrag auf Wiedereinsetzung indie Frist nach Artikel 121 (2) EPÜ statt-gegeben wird und die Bedingungen fürdie Anwendung dieser verfahrensrecht-lichen Schutzklausel erfüllt sind, wirddie Anmeldung vom EPA gemäß Artikel24 (2) PCT in Verbindung mit den Arti-keln 48 (2) a) und 25 PCT sowie Artikel121 EPÜ weiterbehandelt und die Wir-kung der internationalen Anmeldung alseuropäische Patentanmeldung gemäßArtikel 11 (3) PCT aufrechterhalten.

12. Die Zuständigkeit der Prüfungs-abteilung für die Entscheidung über dieAufrechterhaltung der Wirkung der inter-nationalen Anmeldung ergibt sich ausArtikel 153 (2) EPÜ in Verbindung mitden Artikeln 25 und 24 (2) PCT. Zustän-dig für die Weiterbehandlung der An-meldung ist die Eingangsstelle, der dieAnmeldung übermittelt werden muß(Art. 16 EPÜ).

parable remedy under the EPC is resti-tutio in integrum.

11. By letter dated 29 February 1984(received 2 March 1984) the applicantapplied for restitutio in integrum intothe two-month time limit for furtherprocessing and paid the appropriate feeson 2 March 1984. The removal of thecause of non-compliance was the informa-tion from the EPO on 28 February 1984that further processing had to be requestedand that the time limit under Article121 (2) EPC had expired. The omittedact has also been completed, i.e. therequest for further processing has beensubmitted and the fee for furtherprocessing has been paid. The requestfor restitutio in integrum was thereforefiled in time (Article 122 (2) EPC).

Taking into account the complex pro-cedural situation (cf. Decision of theLegal Board of Appeal, J 06/79, OfficialJournal 1980, 225) the grounds putforward by the representative and thefacts on which he relies demonstratethat all due care required by the circum-stances has been taken.

As the application for restitutio inintegrum into the time limit under Article121 (2) EPC is allowed and the condi-tions for the application of this proce-dural safeguard are fulfilled, it followsthat the application shall be further pro-cessed by the EPO pursuant to Article24 (2) in conjunction with Article 48 (2)(a) PCT, Article 25 PCT and Article 121EPC and the effect of the internationalapplication as a European applicationpursuant to Article 1 1 (3) PCT has to bemaintained.

12. The competence of the ExaminingDivision for the decision to maintain theeffect of the international applicationemerges from Article 153 (2) in con-junction with Article 25 and Article 24(2) PCT. The Receiving Section is re-sponsible for the further processingof the application, and it is to thisSection that the application must beremitted (Article 16).

respect de ce délai peut également être"excusé". Dans ce cas, le remède juri-dique comparable prévu par la CBE estla restitutio in integrum.

11. Par une lettre datée du 29 février1984 (reçue le 2 mars 1984), la deman-deresse a présenté une requête en resti-tutio in integrum quant au délai de deuxmois fixé pour la présentation de larequête en poursuite de la procédure eta acquitté les taxes correspondantes le2 mars 1984. C´est la notification del´OEB en date du 28 février 1984 si-gnalant qu´il y avait lieu de présenterune requête en poursuite de la procé-dure et que le délai prévu à l´article121 (2) de la CBE était venu à expira-tion qui a fait cesser l´empêchement.Par ailleurs, l´acte non accompli a étéexécuté, autrement dit la requête enpoursuite de la procédure a été présentéeet la taxe de poursuite de la procédure aété acquittée. La requête en restitutio inintegrum a donc été présentée dans ledélai prescrit (article 122 (2) de la CBE).

Compte tenu de la complexité de lasituation (cf. décision de la chambre derecours juridique J 06/79, JO n°7/1980, p. 225), les motifs invoqués parle mandataire et les faits sur lesquels ils´appuie montrent qu´il a fait preuve detoute la vigilance nécessitée par lescirconstances.

Etant donné qu´il est fait droit à la re-quête en restitutio in integrum quant audélai visé à l´article 121 (2) de la CBEet que les conditions requises pourl´application de cette clause de sauve-garde sont remplies, la procédure rela-tive à la demande doit être poursuiviepar l´OEB en vertu de l´article 24,2) en-semble l´article 48.2) a) du PCT, l´article25 du PCT et l´article 121 de la CBE, etles effets en tant que demande euro-péenne de la demande internationaleprévus à l´article 11.3) du PCT doiventêtre maintenus.

12. C´est à la Division d´examen qu´il in-combe de prendre la décision de main-tenir les effets de la demande interna-tionale, cette compétence découlant del´article 153 (2) de la CBE ensemble lesarticles 25 et 24.2) du PCT. La Sectionde dépôt est compétente pour la pour-suite de la procédure relative à la de-mande et c´est à elle que la demandedoit être transmise (article 16 de la CBE).

* Übersetzung. * Official text. * Traduction.

Page 23: 22. November 1984 22 November 1984 22 novembre 1984 ...archive.epo.org/epo/pubs/oj1984/p549_604.pdf · tout Etat contractant doit, pour ce qui le concerne, excuser pour des motifs

MITTEILUNGEN DESEUROPÄISCHEN PATENTAMTS

Vierte Ausgabe derInternationalenPatentklassifikation (IPK)

1. Die vierte Ausgabe der InternationalenPatentklassifikation wurde am 1. Juni1984 veröffentlicht; sie gilt ab 1. Januar1985. Die neue Ausgabe unterscheidetsich von der bisherigen im wesentlichendadurch, daß die Einführung gründlichüberarbeitet und vereinfacht und dasKonzept der Hybridsysteme das einIndizieren zusätzlicher Informationen(nichtobligatorische Klassifikation) er-möglicht eingeführt worden ist; fer-ner sind alle Anmerkungen nach einemgemeinsamen Muster überarbeitet undlogischer und einheitlicher aufgebautworden.Darüber hinaus sind eine ganze Reihevon Änderungen der Klassifikation be-schlossen worden. Diese Änderungenbetreffen 289 Unterklassen, also etwa45% aller Unterklassen; in 25 Fällensind die Klassentitel oder die dazuge-hörigen Querverweise geändert worden.

2. Im folgenden ist eine Auswahl der alsbesonders wichtig erachteten Än-derungen wiedergegeben.

2.1 Die Klasse B 29, die sowohl dasVerarbeiten von Kunststoffen als auchdas Verarbeiten von Massen in pla-stischem Zustand allgemein umfaßt, undihre sieben Unterklassen wurden voll-ständig neu gestaltet.

Drei der bisherigen Unterklassen, näm-lich B 29 B, C und D, wurden beibehal-ten. Das Klassifikationsschema vonB 29 B und C wurde jedoch von Grundauf geändert und der Zuständig-keitsbereich von B 29 D sehr starkeingeschränkt.Die anderen vier Unterklassen unter B 29wurden alle gestrichen. Ihr Inhalt ist nungrößtenteils in B 29 C erfaßt.

Ferner wurden im Rahmen der Über-arbeitung drei neue Unterklassen auf-genommen, nämlich:

B 27 N Herstellung von Gegenständenim Trockenverfahren aus Spänen oderFasern, die aus Holz oder anderem ligno-zellulosehaltigem oder ähnlichem or-ganischen Material bestehen; mit oderohne Zusatz von organischem Bindemittel

B 29 K Index-Schema für Formmassenin Verbindung mit den UnterklassenB 29 B, C oder D

B 29 L Index-Schema für besondereGegenstände in Verbindung mit Unter-klasse B 29 C

Wie aus den Titeln der Unterklassenersichtlich, sind die Schemata der beidenletztgenannten Unterklassen nur zumIndizieren zu verwenden.

INFORMATION FROM THEEUROPEAN PATENT OFFICE

Fourth edition of theInternational PatentClassification (IPC)

1. The fourth edition of the Inter-national Patent Classification was pub-lished on 1 June, 1984, and will enterinto force on 1 January, 1985. Whatdistinguishes the new edition is mainlythat the Guide has been radically re-drafted and simplified, that the conceptof hybrid systems which provides forthe indexing of additional information(non-obligatory classification) hasbeen introduced, and that all notes havebeen redrafted in a standardised mannerand given a more logical and uniformpresentation.

Furthermore, a considerable number ofamendments to the Classification havebeen agreed upon. The amendmentsrelate to 289 subclasses, i.e. about 45%of the total number of subclasses and in25 cases class titles or theiraccompanying references have beenamended.

2. A selection of what is considered tobe major changes is indicated in the nextparagraphs.

2.1 Class B 29, covering both workingof plastics and working of substances ina plastic state in general, and its sevensubclasses have been completelyrevised.

Three of the existing subclasses havebeen retained, namely B 29 B,C and D.The classification schemes of B 29 B andC, however, have been completelyamended, whereas the scope of B 29 Dhas been very much restricted.

The other four existing subclasses underB 29 have all been deleted. Most of theircontent now is included in B 29 C.

The revision has also led to theintroduction of three new subclasses,namely:

B 27 N Manufacture by dry processes ofarticles, with or without organic bindingagents, made from particles or fibresconsisting of wood or otherlignocellulosic or like organic material

B 29 K Indexing scheme associated withsubclasses B 29 B,C or D, relating tomoulding materials

B 29 L Indexing scheme associated withsubclass B 29 C, relating to particulararticles.

As can be seen from the subclass titles,the schemes of the latter two sub-classes are to be used for indexingpurposes only.

COMMUNICATIONS DEL´OFFICE EUROPEEN DES

BREVETS

Quatrième édition de laClassification internationaledes brevets (CIB)

1. La quatrième édition de la Classifi-cation internationale des brevets a étépubliée le 1er juin 1984 et entrera envigueur le 1er janvier 1985. La principalecaractéristique de cette nouvelle éditionréside dans le fait qu´on a complètementremanié et simplifié le Guide, qu´on aintroduit le concept des systèmes hy-brides permettant l´indexation de l´infor-mation additionnelle (classement facul-tatif), qu´on a normalisé la rédaction detoutes les notes présentées désormaisd´une manière plus logique et uniforme.

En outre, il a été apporté à la Classi-fication un nombre considérable demodifications. Ces modifications con-cernent 289 sous-classes, soit 45%environ du nombre total des sous-classes, et dans 25 cas on a modifiéles titres des classes ou les renvoisles accompagnant.

2. Vous trouverez ci-dessous une listedes modifications jugées les plus impor-tantes.

2.1 La classe B 29, comprenant à lafois le travail des matières plastiqueset le travail des substances à l´état plas-tique en général, a été complètementremaniée, ainsi que ses sept sous-classes.Trois des anciennes sous-classes ont étéconservées, à savoir B 29 B, C et D.Toutefois les systèmes de classificationappliqués à B 29 B et C ont été com-plètement modifiés, et le domaine cou-vert par B 29 D a été considérablementrestreint.Les quatre autres sous-classes com-prises dans B 29 ont toutes été sup-primées. Leur contenu a été repris pourl´essentiel dans B 29 C.

La révision a entraîné également lacréation de trois nouvelles sous-classes,à savoir:

B 27 N Fabrication d´objets par des pro-cédés à sec, avec ou sans agents liantsorganiques, à partir de particules ou defibres de bois ou d´autres matières ligno-cellulosiques ou de substances or-ganiques analogues.

B 29 K Système d´indexation associéaux sous-classes B 29 B, C ou D, relatifaux matières à mouler.

B 29 L Système d´indexation associé àla sous-classe B 29 C, relatif aux objetsparticuliers.Comme l´indique le titre des sous-classes, les systèmes associés aux deuxdernières sous-classes ne doivent êtreutilisés qu´aux fins d´indexation.

Page 24: 22. November 1984 22 November 1984 22 novembre 1984 ...archive.epo.org/epo/pubs/oj1984/p549_604.pdf · tout Etat contractant doit, pour ce qui le concerne, excuser pour des motifs

2.2 In die vierte Ausgabe der IPK sindvier neue Unterklassen mit folgendenSymbolen und Titeln aufgenommenworden:B 23 H Metallbearbeitung durch Einwir-kung elektrischen Stromes hoher Strom-dichte auf ein Werkstück unter Benut-zung einer Elektrode an Stelle einesWerkzeugs; derartige Bearbeitung kom-biniert mit anderen Formen der Metall-bearbeitungDiese neue Unterklasse umfaßt die alsElektroerosion bezeichnete Metallbear-beitung.

C 07 K Peptide, ProteineC 10 N Index-Schema nur im Zusam-menhang mit Unterklasse C 10 M zubenutzenDiese neue Unterklasse wurde in Verbin-dung mit einer vollständigen Überar-beitung des Klassifikationsschemas derUnterklasse C 10 M eingeführt, dieSchmiermittelzusammensetzungen unddie Verwendung chemischer Substanzenals Schmiermittel oder als Schmiermittel-zusätze umfaßt.B 03 M Kodieren; Dekodieren oder Ko-deumsetzung allgemein

2.3 Zwei andere Unterklassen wurdengestrichen, nämlichC 13 L Stärke, Dextrin; ähnliche Kohlen-hydrate. Der Inhalt dieser Unterklasse istnun in der Unterklasse C 08 B erfaßt.

G 11 D Übertragung von digitalen Datenzwischen dynamischen und statischenSpeichern. Der Inhalt dieser Unter-klasse wurde in G 06 F 12/00, 13/00übernommen.

2.4 Grundlegend geänderte Unter-klassen

In mehreren Unterklassen ist das Klassi-fikationsschema im Vergleich zur drittenAusgabe vollständig oder sehr weitge-hend geändert worden. Zu erwähnensind neben den Unterklassen B 29 B undC und C 10 M (wie bereits dargelegt) fol-gende Unterklassen:

C 04 B Kalk: Magnesia; Schlacke; Ze-mente; Massen hieraus z. B. Mörtel, Be-ton oder ähnliche Baumaterialien; künst-liche Steine, keramische Massen; feuer-feste Massen; Behandlung von Natur-steinenC 07 B Allgemeine Verfahren der or-ganischen Chemie; hierfür bestimmteVorrichtungenC 23 C Beschichten metallischer Werk-stoffe; Beschichten von Werkstoffen mitmetallischen Stoffen; Oberflächenbe-handlung metallischer Werkstoffe durchDiffussion in die Oberfläche, durchchemische Umwandlung oder Substitu-tion; Beschichten allgemein durchVakuumbedampfen, durch Aufstäuben,durch lonenimplantation oder durchchemisches Abscheiden aus derDampfphase

G 10 L Analyse oder Synthese vonSprache; Spracherkennung

2.2 Four new subclasses have beenintroduced in the fourth edition of theIPC. Their symbols and titles are:

B 23 H Working of metal by the action ofa high concentration of electric currenton a work-piece using an electrodewhich takes the place of a tool; suchworking combined with other forms ofworking of metal.

This new subclass covers the so-calledelectroerosion of metal.

C 07 K Peptides, proteinsC 10 N Indexing scheme associated withC 10M.

This new subclass was introduced inconnection with a complete revision of theclassification scheme of subclass C 10 M,covering lubricating compositions and theuse of chemical substances as lubricants orlubricating ingredients.

H 03 M Coding; decoding or codeconversion, in general.

2.3 Two other subclasses have alsobeen deleted, namely:

C 13 L Starch; Dextrin; similarcarbohydrates. Its contents are nowcovered by subclass C 08 B.

G 11 D Transmission of digital databetween dynamic and static stores. Thecontent of this subclass is transferred toG 06 F 12/00, 13/00.

2.4 Substantially amended subclasses

In several subclasses completelyamended or nearly completely amendedclassification schemes have been intro-duced in comparison with the schemesin the third edition. Besides the sub-classes B 29 B and C (already mentionedin 21) and C 10 M (already mentioned in22) we draw attention to:C 04 B Lime; Magnesia; Slag; Cements;Compositions e.g. mortars, concrete orlike building materials; Artificial stone;Ceramics; Refractories, Treatment ofnatural stone

C 07 B General methods of organicchemistry; Associated apparatus

C 23 C Coating metallic material;Coating material with metallic material;Surface treatment of metallic material bydiffusion into the surface, by chemicalconversion or substitution; Coating byvacuum evaporation, by sputtering, byion implantation or by chemical vapourdeposition, in general

G 10 L Speech analysis or synthesis;Speech recognition

2.2 Dans la quatrième édition de laCIB on a ajouté quatre nouvelles sous-classes, dont voici les symboles et lestitres:B 23 H Usinage du métal par action, surune pièce, d´une forte concentration decourant électrique à l´aide d´une élec-trode tenant lieu d´outil; Usinage de cetype combiné avec d´autres formesd´usinage du métal.

Cette nouvelle sous-classe couvre l´usi-nage du métal décrit comme "électro-érosion".C 07 K Peptides; ProtéinesC 10 N Système d´indexation associé àla sous-classe C 10 M.

On a ajouté cette nouvelle sous-classedans le cadre d´une révision complètedu système de classification de la sous-classe C 10 M, couvrant les composi-tions lubrifiantes et l´utilisation de sub-stances chimiques comme lubrifiants ouingrédients lubrifiants.

H 03 M Codage, décodage ou conver-sion de code, en général.

2.3 Deux autres sous-classes ontégalement été supprimées, à savoir:C 13 L Amidon; Dextrine; Hydrates decarbone similaires. Son contenu estactuellement repris dans la sous-classeC 08 B.

G 11 D Transmission de données numé-riques entre mémoires dynamiques etstatiques. Le contenu de cette sous-classe est transféré sous G 06 F 12/00.13/00.

2.4 Sous-classes considérablementmodifiéesDans plusieurs sous-classes, on a intro-duit des systèmes de classificationmodifiés totalement ou quasi totale-ment par rapport à ceux utilisés dans latroisième édition. Outre les sous-classesB 29 B et C (déjà mentionnées sous 21)et C 10 M (déjà mentionnée sous 22)nous signalons:

C 04 B Chaux; Magnésie; Scories; Ci-ments; Leurs compositions, par exem-ple mortiers, béton ou matériaux de con-struction similaires; Pierre artificielle;Céramiques; Réfractaires; Traitement dela pierre naturelle.

C 07 B Procédés généraux de chimieorganique; Appareils à cet effet.

C 23 C Revêtement de matériaux métal-liques; Revêtement de matériaux avecdes matériaux métalliques; Traitementde matériaux métalliques par diffusiondans la surface, par conversion chimiqueou substitution; Revêtement par éva-poration sous vide; par pulvérisationcathodique, par implantation d´ions oupar dépôt chimique en phase vapeur, engénéral.

G 10 L Analyse ou synthèse de la parole;Reconnaissance de la parole.

Page 25: 22. November 1984 22 November 1984 22 novembre 1984 ...archive.epo.org/epo/pubs/oj1984/p549_604.pdf · tout Etat contractant doit, pour ce qui le concerne, excuser pour des motifs

2.5 Unterklassen mit einer Reihe vonÄnderungen des Klassifikationsschemas

Diese Änderungen bestehen häufig inder Einführung feinerer Unterteilungen.In einigen Fällen liegen die Änderungenin genaueren Definitionen oder in einemanderen Klassifizierungsansatz. Sowurde in der Unterklasse C 03 C dasSchema zu chemischen Zusammenset-zungen für Gläser, Glasuren oder Emailsvollständig überarbeitet. Der Inhalt derUnterklasse C 09 K wurde neu definiertund der Klassifizierungsansatz in ihrergrößten Hauptgruppe (lumineszierendeMaterialien) vollständig geändert. Dar-über hinaus ist eine neue Hauptgruppemit etwa 30 Untergruppen eingeführtworden, um die Entwicklung bei denchemischen Zusammensetzungen vonFlüssigkristallen zu erfassen.

In die Unterklasse F 24 J wurde eineneue Hauptgruppe mit etwa 25 Unter-gruppen betreffend die Verwendung vonSonnenwärme aufgenommen. In F 23 Gwurde das Schema für die Abfallver-brennung verfeinert.

Eine Reihe von Änderungen sind fernerin folgende Unterklassen aufgenommenworden:B 24 B Maschinen, Einrichtungen oderVerfahren zum Schleifen oder Polieren;Ab- oder Herrichten der Arbeitsflächenvon Schleifwerkzeugen; Zuführen vonSchleif-, Polier- oder Läppmitteln

B 60 C Fahrzeugreifen; Aufpumpen vonReifen; Wechseln oder Reparieren vonReifen; Ausbessern von aufblasbarenelastischen Körpern allgemein oder An-bringen von Ventilen daran; Einrich-tungen oder Anordnungen in bezug aufReifen

G 01 N Untersuchen oder Analysierenvon Stoffen durch Bestimmen ihrerchemischen oder physikalischen Eigen-schaftenG 05 F Systeme zum Regeln elektrischeroder magnetischer Veränderlicher

G 11 B Informationsspeicherung mitRelativbewegung zwischen Aufzeich-nungsträger und Wandler

H 04 N Bildübertragung, z. B. Fernsehen

2.6 Die vierte Ausgabe der IPK und dasdazugehörige Stich- und Schlagwörter-verzeichnis können beim Verlag

Carl Heymanns Verlag KGGereonstraße 18 22

D5000 Köln 1Bundesrepublik Deutschland

bestellt werden.

2.5 Subclasses in which aconsiderable number of amendmentshas been introduced in theclassification schemes.Often these amendments consist of theintroduction of a number of finersubdivisions. Sometimes the amend-ments relate to more precise definitionsor to a change in approach to classifying.For instance, in subclass C 03 C thescheme relating to chemicalcomposition of glasses, glazes orvitreous enamels has been completelyrevised. In the case of subclass C 09 K,the content of the subclass has been re-defined and the approach to classifyingin its largest main group (coveringluminescent materials) has beencompletely changed. In addition a newmain group with about thirty subgroupshas been introduced to cover develop-ments relating to the chemicalcomposition of liquid crystals.In subclass F 24 J a new main groupwith about twenty-five subgroups hasbeen introduced covering use of solarheat. And in F 23 G the scheme dealingwith consuming waste by combustionhas been made more detailed.

Other cases of subclasses in which aconsiderable number of amendmentshas been introduced are:

B 24 B Machines, devices or processesfor grinding or polishing; Dressing orconditioning of abrading surfaces;Feeding or grinding, polishing or lappingagents

B 60 C Vehicle tyres; Tyre inflation; Tyrechanging or repairing; Repairing, orconnecting valves to, inflatable elasticbodies in general; Devices orarrangements related to tyres

G 01 N Investigating or analysingmaterials by determining their chemicalor physical properties

G 05 F Systems for regulating electric ormagnetic variablesG 11 B Information storage based onrelative movement between recordcarrier and transducer

H 04 N Pictorial communication, e.g.television

2.6 The fourth edition of the IPC and itsCatchword Index may be ordered fromthe publisher:

Carl Heymanns Verlag KGGereonstrasse 1 8 22

5000 Cologne 1Federal Republic of Germany

2.5 Sous-classes dont les systèmes declassification ont subi un nombre im-portant de modifications

Les modifications consistent souvent àintroduire un certain nombre de subdivi-sions plus fines. Parfois, elles correspon-dent à des définitions d´une précisionaccrue ou à un changement de méthodede classement. Par exemple, dans lasous-classe C 03 C, le système con-cernant la composition chimique desverres, glaçures ou émaux vitreux a étécomplètement remanié. Dans le cas dela sous-classe C 09 K, le contenu a étéredéfini et la méthode de classement àl´intérieur de son groupe principal leplus vaste (couvrant les substancesluminescentes) a été totalement modi-fiée. Enfin on a ajouté un nouveaugroupe principal comportant environ 30sous-groupes pour couvrir les déve-loppements intervenus dans le domainede la composition chimique des cristauxliquides. Dans la sous-classe F 24 J ona ajouté un nouveau groupe principalregroupant environ 25 sous-groupescouvrant l´utilisation de la chaleur so-laire. Et dans F 23 G le système concer-nant l´incinération des déchets est traitéplus en détail.Nous indiquons ci-dessous d´autres casde sous-classes ayant subi un grandnombre de modifications:B 24 B Machines, dispositifs ou pro-cédés pour meuler ou pour polir; Dres-sage ou remise en état des surfacesabrasives; Alimentation des machines enmatériaux de meulage, de polissage oude rodage

B 60 C Pneumatiques pour véhicules;Gonflage des pneumatiques; Change-ment ou réparation des pneumatiques;Réparation ou fixation des valves à descorps élastiques gonflables, en général;Dispositifs ou agencements concernantles pneumatiques

G 01 N Recherche ou analyse des maté-riaux par détermination de leurs pro-priétés chimiques ou physiques

G 05 F Système de régulation des varia-bles électriques ou magnétiquesG 11 B Enregistrement de l´informa-tion basé sur un mouvement relatif entrele support d´enregistrement et le trans-ducteur

H 04 N Transmission d´images, parexemple télévision

2.6 Les commandes relatives à la qua-trième édition de la CIB et à l´index desmots-clés sont à adresser à l´éditeur:

Carl Heymanns Verlag KGGereonstrasse 18 22

5000 Cologne 1République fédérale d´Allemagne

Page 26: 22. November 1984 22 November 1984 22 novembre 1984 ...archive.epo.org/epo/pubs/oj1984/p549_604.pdf · tout Etat contractant doit, pour ce qui le concerne, excuser pour des motifs

Mitteilung des Präsidenten desEPA vom 5. November 1984über die Tage (ausgenommenSamstage und Sonntage) imJahre 1985 an denen das EPAzur Entgegennahme vonSchriftstücken nicht geöffnetistGemäß Ziffer 5.4 der Mitteilung desPräsidenten des EPA vom 5. Dezember1979 (Amtsblatt 1/1980, S. 2 und 3)werden die Kalendertage im Jahre1985, an denen das EPA zur Entgegen-nahme von Schriftstücken nicht geöff-net ist, bekanntgegeben (ausgenommenSamstage und Sonntage):

EPA München

1. Januar 1985 Neujahr5. April 1985 Karfreitag*8. April 1985 Ostermontag*1. Mai 1985 Erster Mai16. Mai 1985 Christi

Himmelfahrt*27. Mai 1985 Pfingstmontag*6. Juni 1985 Fronleichnam*17. Juni 1985 Tag der Deutschen

Einheit15. August 1985 Mariä Himmelfahrt1. November 1985 Allerheiligen20. November 1985 Buß- und Bettag*24. Dezember 1985 Heiliger Abend25. Dezember 1985 Erster

Weihnachtstag26. Dezember 1985 Zweiter

Weihnachtstag31. Dezember 1985 Silvester

EPA Den Haag

1. Januar 1985 Neujahr5. April 1985 Karfreitag*8. April 1985 Ostermontag*30: April 1985 Nationalfeiertag1. Mai 1985 Erster Mai16. Mai 1985 Christi

Himmelfahrt*27. Mai 1985 Pfingstmontag*24. Dezember 1985 Heiliger Abend25. Dezember 1985 Erster

Weihnachtstag26. Dezember 1 985 Zweiter

Weihnachtstag31. Dezember 1985 Silvester

* Anmerkung: Mit * gekennzeichneteKalendertage sind bewegliche Feiertage

Notice of the President of theEPO dated 5 November 1984concerning days (excludingSaturdays and Sundays) onwhich the EPO is not open forthe receipt of documents during1985

In accordance with point 5.4 of theNotice of the President of the EPO dated5 December 1979 (Official Journal1/1980, pp. 2 and 3), the calendar datesduring the year 1985, on which the EPOwill be closed for the receipt of docu-ments, are hereby notified (excludingSaturdays and Sundays):

EPO Munich

1 January 1985 New Year´s Day5 April 1985 Good Friday*8 April 1985 Easter Monday*1 May 1985 May Day16 May 1985 Ascension Day*27 May 1985 Whit Monday*6 June 1985 Corpus Christi*17 June 1985 Day of German

Unity15 August 1985 Assumption Day1 November 1985 All Saints´ Day20 November 1985 Day of Prayer and

Repentance*24 December 1985 Christmas Eve25 December 1985 Christmas Day26 December 1985 Boxing Day31 December 1985 New Year´s Eve

EPO The Hague

1 January 1985 New Year´s Day5 April 1985 Good Friday*8 April 1985 Easter Monday*30 April 1985 National Holiday1 May 1985 May Day16 May 1985 Ascension Day*27 May 1985 Whit Monday*24 December 1985 Christmas Eve25 December 1985 Christmas Day26 December 1985 Boxing Day31 December 1985 New Year´s Eve

* Note: Days marked with an * aremovable public holidays

Communiqué du Président del´OEB du 5 novembre 1984 rela-tif aux dates (à l´exception dessamedis et dimanches)auxquelles l´OEB n´est pasouvert pour la réception despièces, au cours de l´année1985

Conformément au point 5.4 de la com-munication du Président de l´OEB du 5décembre 1979 (Journal officiel n°1/1980, pages 2 et 3) nous indiquons ci-après les dates des jours où, au coursde l´année 1985, l´OEB n´est pas ouvertpour recevoir le dépôt des pièces (al´exception des samedis et dimanches):

OEB Munich

1 janvier 1985 Nouvel An5 avril 1985 Vendredi Saint*8 avril 1985 Lundi de Pâques*1 mai 1985 Fête du travail16 mai 1985 Ascension*27 mai 1985 Lundi de

Pentecôte*6 juin 1985 Fête-Dieu*17 juin 1985 Fête nationale1 5 août 1985 Assomption1 novembre 1985 Toussaint20 novembre 1985 Jour de pénitence

et de prière*24 décembre 1985 Veille de Noël25 décembre 1985 Noël26 décembre 1985 Lendemain de Noël31 décembre 1985 Saint-Sylvestre

OEB La Haye

1 janvier 1985 Nouvel An5 avril 1985 Vendredi Saint*8 avril 1985 Lundi de Pâques*30 avril 1985 Fête nationale1 mai 1985 Fête du travail16 mai 1985 Ascension*27 mai 1985 Lundi de

Pentecôte*24 décembre 1985 Veille de Noël25 décembre 1985 Noël26 décembre 1985 Lendemain de Noël31 décembre 1985 Saint-Sylvestre

* Note: les jours marqués d´un * sont desjours fériés mobiles

Page 27: 22. November 1984 22 November 1984 22 novembre 1984 ...archive.epo.org/epo/pubs/oj1984/p549_604.pdf · tout Etat contractant doit, pour ce qui le concerne, excuser pour des motifs

Wichtiger Hinweis: NeueGebühren

Neufestsetzung der Verwaltungs-gebühren, Auslagen und Verkaufs-preise des EPA und der Gegenwerteder Gebühren, Differenzbeträgefür Jahresgebühren, Auslagen undVerkaufspreise in den Währungender Vertragsstaaten

1. Durch Beschluß des Präsidenten desEPA vom 8. November 1984 sind dieVerwaltungsgebühren, Auslagen undVerkaufspreise des EPA und die Gegen-werte der Gebühren, Differenzbeträgefür Jahresgebühren, Auslagen und Ver-kaufspreise in den Währungen der Ver-tragsstaaten neu festgesetzt worden.

2. Dieser Beschluß und das Gebühren-verzeichnis in der ab 3. Januar 1985geltenden Fassung sind in einer Beilage(vom 23. November 1984) zum ABI.11/1984 veröffentlicht worden. DieseBeilage wird den Beziehern des Amts-blatts gesondert zugestellt. WeitereExemplare des neuen Gebührenverzeich-nisses können beim EPA, Referat 4.5.1(Unterrichtung der Öffentlichkeit), ange-fordert werden.

Important Notice: New Fees

Revised EPO administrative fees,costs and prices and new equivalentsof fees, renewal fee differences, costsand prices in the currencies of theContracting States

1. By Decision of the President of theEPO dated 8 November 1984, theamount of the administrative fees, costsand prices of the EPO and the equivalentsof fees, renewal fee differences, costsand prices in the currencies of the Con-tracting States have been revised witheffect from 3 January 1985.2. This Decision and the Schedule ofFees effective from 3 January 1985have been published in a Supplement(dated 23 November 1984) to OJ11/1984. This Supplement is being sentunder separate cover to Official Journalsubscribers. Additional copies of the newSchedule of Fees may be obtained onrequest from the EPO, Department 4.5.1(Public Information).

Avis important: Nouvelles Taxes

Révision des montants des taxesd´administration, frais et tarifs devente de l´OEB, ainsi que des contre-valeurs des taxes, différences entreles anciens et les nouveaux montantsdes taxes annuelles, frais et tarifs devente dans les monnais des Etatscontractants

1. Par décision du Président de l´OEBen date du 8 novembre 1984, les mon-tants des taxes d´administration, frais ettarifs de vente de l´OEB, ainsi que lescontre-valeurs des taxes, différencesentre les anciens et les nouveaux mon-tants des taxes annuelles, frais et tarifsde vente dans les monnaies des Etatscontractants ont été révisés avec effetau 3 janvier 1985.

2. Cette décision et le barème des taxesen vigueur à compter du 3 janvier 1985sont publiés dans un supplément (datédu 23 novembre 1984) au JO n°11/1984, qui sera expédié séparémentaux abonnés. Des copies supplémen-taires du nouveau barème des taxespeuvent être obtenues sur demandeadressée à l´OEB, bureau 4.5.1 ("Infor-mation du public").

VERTRETUNG

Prüfungskommission für dieeuropäische Eignungsprüfung

1. Bisher bekanntgemachte Mitteilungender Prüfungskommission

Siehe ABI. 1/1979, S. 27 ff.; 3/1979, S.95 ff.; 6-7/1979, S. 298 ff.; 7/1980, S.218 ff.; 5/1981, S. 147 ff.; 9/1981, S.383 ff.; 11/1981, S. 520; 8/1982, S.319 ff.; 11/1982, S. 426; 1/1983, S.24; 3/1983, S. 104; 7/1983, S. 290 ff.;11/1983, S. 460; 3/1984, S. 129 ff.;7/1984, S. 326 ff.

2. Mitglieder der PrüfungskommissionHerr G. Gall (AT) ist mit Wirkung vom30. September 1984 aus der Prüfungs-kommission vorzeitig ausgeschieden.Für seine verbleibende Amtszeit undweitere vier Jahre hat der Präsident desEuropäischen Patentamts Herrn R.Teschemacher (DE) zum Mitglied derPrüfungskommission berufen. SeineAmtszeit hat am 1. Oktober 1984 be-gonnen und endet am 30. November1988.Die Amtszeit der Herren G. H. Graf(DE) und R. C. Petersen (GB) endetenach vier Jahren am 30. November1984.Der Präsident des Europäischen Patent-amts hat nach Anhörung des Rats desInstituts der zugelassenen Vertreter die

REPRESENTATION

Examination Board for theEuropean QualifyingExamination

1. Examination Board notices publishedto dateSee OJ 1/1979, p. 27 et seq.; 3/1979, p.95 et seq.; 6-7/1979, p. 298 et seq.;7/1980, p. 218 et seq.: 5/1981, p. 147et seq.; 9/1981, p. 383 et seq.;11/1981, p. 520; 8/1982, p. 319 etseq.; 11/1982, p. 426; 1/1983, p. 24;3/1983, p. 104; 7/1983, p. 290 et seq.;11/1983, p. 460; 3/1984, p. 129 et seq.;7/1984, p. 326 et seq.

2. Members of the Examination Board

Mr. G. Gall (AT) ceased to be a memberof the Examination Board with effectfrom 30 September 1984 before theexpiry of his term of office. For theremainder of his term of office and for afurther four years, i.e. from 1 October1984 to 30 November 1988, the Presi-dent of the European Patent Office hasappointed Mr. R. Teschemacher (DE) asa member of the Board.

The terms of office of Messrs. G. H. Graf(DE) and R. C. Petersen (GB) expiredafter four years on 30 November 1984.

The President of the European PatentOffice has appointed Messrs. G. H. Grafand R. C. Petersen as members of the

REPRESENTATION

Jury d´examen pour l´exameneuropéen de qualification

1. Communications du jury publiéesjusqu´à présentVoir JO n° 1/1979, p. 27 s.; n° 3/1979,p. 95 s.; n° 6-7/1979, p. 298 s.; n°7/1980, p. 218 s.; n° 5/1981, p. 147 s.;n° 9/1981, p. 383 s.; n° 11/1981, p.520; n° 8/1982, p. 319 s.; n° 11/1982,p. 426; n° 1/1983, p. 24; n° 3/1983, p.104; n° 7/1983, p. 290 s.; n° 11/1983,p. 460; n° 3/1984, p. 129 s.; n° 7/1984,p. 326 s.

2. Membres du jury d´examen

M. G. Gall (AT) a démissionné de sesfonctions au sein du jury d´examen le30 septembre 1984, avant la fin de sonmandat. Pour le remplacer, le Présidentde l´Office européen des brevets anommé M. R. Teschemacher (DE) mem-bre du jury d´examen. Son mandat a priseffet le 1er octobre 1984 et se terminerale 30 novembre 1988.

Le mandat de MM. G. H. Graf (DE) etR. C. Petersen (GB) a expiré au bout dequatre ans, le 30 novembre 1984.

Par décision du Président de l´Officeeuropéen des brevets, MM. G. H. Graf etR. C. Petersen sont reconduits dans leurs

Page 28: 22. November 1984 22 November 1984 22 novembre 1984 ...archive.epo.org/epo/pubs/oj1984/p549_604.pdf · tout Etat contractant doit, pour ce qui le concerne, excuser pour des motifs

Herren G. H. Graf und R. C. Petersenerneut zu Mitgliedern der Prüfungs-kommission berufen. Die Amtszeit dieserMitglieder beginnt am 1. Dezember1984 und endet am 30. November1988.

3. PrüfungsergebnisseDie vom 25.-27. April 1984 durch-geführte fünfte europäische Eignungs-prüfung ist von den folgenden Be-werbern bestanden worden:

Examination Board, following consulta-tion of the Council of the Institute ofProfessional Representatives, for a fur-ther term of office. The terms of office ofthese members are for four years witheffect from 1 December 1984.

3. Examination Results

In the fifth European Qualifying Examina-tion, which took place from 25 to 27April 1984, the following candidateswere successful:

fonctions de membres du jury d´examen,après consultation du Conseil de l´Ins-titut des mandataires agréés. La duréedu mandat de ces membres du jury, quiest de quatre années, a commencé àcourir le 1er décembre 1984.

3. Résultats de l´examen

Les candidats suivants ont été reçusau cinquième examen européen dequalification qui a eu lieu du 25 au27 avril 1984:

Auckland, Jacqueline Fey, Hans-Jürgen Meyer-Roedern, GisoBassett, Richard Fisher, Adrian John Müller, Karl-ErnstBecker, Thomas Füller, Hans-Joachim Notegen, Eric-AndréBrandt, Ernst-Ulrich Glawe, Ulrich Petra, ElkeButtrick, Richard Harland, Linda Pfeifer, Hans-PeterBücken, Helmut Harrison, Ivor Stanley Pinter, RudolfCaron, Jean Jones, Pauline Possel, OkkoClark, Jane Kopacz, William James Russell, JosephineCockbain, Julian König, Beate Schreiner, SiegfriedCouchman, Jonathan Hugh Kugler, Hermann Unwin, Stephen G.Crawford, Fiona Lang, Friedrich Urner, PeterDahlkamp, Heinrich-L. March, Gary C. Valentine, Jill BarbaraDreykorn-Lindner, Werner Maury, Richard Philip Vinazzer, EdithEbert, Jutta Mayer, Ludgerus Wasmuth. RolfEngelhardt, Harald

Listeder beimEuropäischen Patentamtzugelassenen Vertreter

Listof professionalrepresentatives before theEuropean Patent Office

Listedes mandataires agréésprès l´Officeeuropéen des brevets

Bundesrepublik DeutschlandFederal Republic of Germany / République fédérale d´Allemagne

Änderungen / Amendments / ModificationsCzowalla, Ernst (DE)

PatentanwälteCzowalla, Matschkur & PartnerDr.-Kurt-Schumacher-Strasse 23Postfach 9109D 8500 Nürnberg 11

Hanewinkel, Lorenz (DE)Patentanwalt Dipl.-Phys. Lorenz HanewinkelFerrariweg 17aD4790 Paderborn

Matschkur, Peter (DE)PatentanwälteCzowalla, Matschkur & PartnerDr.-Kurt-Schumacher-Strasse 23Postfach 9109D 8500 Nürnberg 11

Prüfer, Lutz H. (DE)Harthauser Strasse 25dD 8000 München 90

Frankreich / FranceÄnderungen / Amendments / ModificationsAubertin, François (FR)

Cabinet Lepage & Aubertin4, rue de HaguenauF 67000 Strasbourg

Flavenot, Bernard (FR)ABRITT s.a.r.l.122, rue de ParisF 78470 Saint Rémy-lès-Chevreuse

Lepage, Jean-Pierre (FR)Cabinet Lepage & Aubertin23-25, rue Nicolas LeblancF 59000 Lille

Richebourg, Michel François (FR)Etudes et FabricationDowell Schlumberger (EFDS)Z. I. MolinaB.P. 90F 42003 Saint-Etienne Cedex

Lôschungen / Deletions / RadiationsCongard, Roger-Paul (FR) - R. 102(1)

Institut Français du Pétrole1 et 4, avenue de Bois-Préau, B.P. 311F 92502 Rueil-Malmaison

Italien / Italy / ItalieÄnderungen / Amendments / ModificationsDini, Roberto (IT)

Indesit S.p.A.Stabilimento No. 5Ufficio BrevettiI10060 None (Torino)

Page 29: 22. November 1984 22 November 1984 22 novembre 1984 ...archive.epo.org/epo/pubs/oj1984/p549_604.pdf · tout Etat contractant doit, pour ce qui le concerne, excuser pour des motifs

Ôsterreich / Austria / AutricheLôschungen / Deletions / RadiationsHoschtalek, Manfred (AT) - R. 102(1)

Steyr-Daimler-Puch AktiengesellschaftKärntnerring 7A 1010 Wien

Schweiz / Switzerland / SuisseÄnderungen / Amendments / ModificationsLusuardi, Werther Giovanni (CH)

Lyss-Strasse 12CH 3293 Dotzigen

Schweden / Sweden / SuèdeLôschungen / Deletions / RadiationsDu Rietz, Harald (SE) - R. 102(1)

Östergötlands Patentbyrå ABBox 5020S 600 05 Norrkôping

Vereinigtes Königreich / United Kingdom / Royaume-UniÄnderungen / Amendments / Modifications

Cresswell, Thomas Anthony (GB)J. A. Kemp & Co.14 South SquareGray´s InnGB London WC1R 5EU

Dayneswood, Trevor (GB)Beecham PharmaceuticalsPatent and Trade Mark DepartmentBiosciences Research CentreGreat BurghYew Tree Bottom RoadGB Epsom, Surrey KT18 5XQ

Dummett, Thomas lan Peter (GB)102 The ThoroughfareGBWoodbridge, Suffolk IP12 1AR

Gibson, Stewart Harry (GB)Urquhart-Dykes & Lord36 Forest RoadGB Hartwell, Northampton NN7 2HE

Lôschungen / Deletions / RadiationsRennie, lan Malcolm (GB) R. 102(1)

Boult, Wade & Tennant27 Furnival StreetGB London EC4A 1PQ

AUS DEN VERTRAGSSTAATEN

Zweites Symposiumeuropäischer Patentrichterin Straßburg

1. Auf Einladung des Centre d´EtudesInternationales de la Propriété Indus-trielle (CEIPI) fand in Straßburg in derZeit vom 5. bis 7. September 1984unter der Schirmherrschaft des General-sekretärs des Europarats das 2. Sym-posium europäischer Patentrichterstatt.* Erneut sind Richter der für Pa-tentsachen zuständigen Gerichte aus 8Vertragsstaaten des Europäischen Pa-tentübereinkommens (BundesrepublikDeutschland, Frankreich, Italien, Nieder-lande, Österreich, Schweden, Schweizund Vereinigtes Königreich) und Mit-glieder der Beschwerdekammern desEPA zusammengetroffen, um Fragen vongemeinsamem Interesse zu erörtern, diemit der Anwendung und Auslegung deseuropäischen und damit harmonisiertennationalen Patentrechts verbunden sind.

2. Themen des 2. Symposiums waren:

Die Rechtsprechung der Beschwerde-kammern des EPA zu Art. 56 EPÜ (Er-finderische Tätigkeit)

INFORMATION FROM THECONTRACTING STATES

Second Symposium ofEuropean Patent Judgesin Strasbourg

1. The Second Symposium of EuropeanPatent Judges was held in Stras-bourg from 5 to 7 September 1984under the aegis of the Secretary Generalof the Council of Europe and at the in-vitation of the Centre d´Etudes Inter-nationales de la Propriété Industrielle(CEIPI).* Once again, patent judges fromeight Contracting States to the Euro-pean Patent Convention (Austria, France,Federal Republic of Germany, Italy, theNetherlands, Sweden, Switzerland andthe United Kingdom) and members ofthe Boards of Appeal of the EPO met todiscuss matters of common interestassociated with the application andinterpretation of European patent lawand national laws harmonised there-with.

2. The following topics were discussed:

EPO Board of Appeal case law onArticle 56 EPC (inventive step)

INFORMATIONS RELATIVESAUX ETATS CONTRACTANTS

Second colloque européen desjuges de brevets, tenu àStrasbourg

1. Du 5 au 7 septembre 1984 s´est tenuà Strasbourg, à l´invitation du Centred´Etudes Internationales de la PropriétéIndustrielle (CEIPI) et sous le patronagedu Secrétaire général du Conseil del´Europe, le second colloque européendes juges de brevets*. Des juges detribunaux compétents en matière debrevets, venus de huit Etats parties àla Convention sur le brevet européen(République fédérale d´Allemagne, Au-triche, France, Italie, Pays-Bas, Royaume-Uni, Suède et Suisse) et des membresdes chambres de recours de l´OEB sesont rencontrés à nouveau pour dis-cuter de questions d´intérêt commun enrelation avec l´application et l´interpré-tation du droit européen des brevetset du droit national des brevets har-monisé avec le droit européen.

2. Les thèmes abordés au cours de cesecond colloque étaient les suivants:- jurisprudence des chambres de re-cours de l´OEB faisant intervenir l´article56 de la CBE (activité inventive)

* Ein Bericht über das 1. Symposium europäischerPatentrichter in München 1982 ist in ABI.12/1982, S. 480 veröffentlicht.

A report on the First Symposium of EuropeanPatent Judges held in Munich in 1982 was pub-lished in OJ 12/1982, p. 480.

* Un compte rendu du 1er colloque européen desjuges de brevets, tenu à Munich en 1982, a étépublié dans le JO n° 12/1982, p. 480.

Page 30: 22. November 1984 22 November 1984 22 novembre 1984 ...archive.epo.org/epo/pubs/oj1984/p549_604.pdf · tout Etat contractant doit, pour ce qui le concerne, excuser pour des motifs

Die zweite medizinische Indikationfür Arzneimittel im europäischen Patent-recht

Schutz der Mikroorganismen im euro-päischen Patentrecht

Das Anerkennungsprotokoll

funktionelle Ansprüche Die Errichtung eines Berufungsge-

richts für das Gemeinschaftspatent.

Die einführenden Referate des Sym-posiums werden zusammen mit einemBericht über Verlauf und Ergebnisse derTagung demnächst in den drei Amts-sprachen in verschiedenen Zeitschriften(PIBD, GRUR Int. und IIC) veröffentlicht.Hierauf wird zu gegebener Zeit im Amts-blatt des EPA hingewiesen.

second medical use of medicamentsunder European patent law

the protection of micro-organismsunder European patent law

the Protocol on Recognition functional claimsthe setting up of a Court of Appeal for

Community patents.

The introductory papers delivered at theSymposium and the report on the pro-ceedings and conclusions will be pub-lished shortly in the three official lan-guages in three different publications(PIBD, GRUR Int. and IIC). A notice con-cerning this will appear in the OfficialJournal of the EPO in due course.

la deuxième application théra-peutique d´un médicament en droit euro-péen des brevets

la protection des micro-organismesen droit européen des brevets

le protocole sur la reconnaissance les revendications fonctionnelles

la création d´une Cour d´appel pourle brevet communautaire

Les exposés des orateurs, ainsi qu´unrapport présentant le compte rendu dela discussion et des conclusions du col-loque vont prochainement être publiésdans les trois langues officielles dansdifférentes revues (PIBD, GRUR Int. etIIC). Cette publication sera annoncéeen temps utile dans le Journal officielde l´OEB.

ÖsterreichGesetzgebung

I. Patentrechts-Novelle 1984

Der Nationalrat der Republik Österreichhat am 23. Mai 1984 das Bundesgesetz,mit dem das Patentgesetz 1970 unddas Patentverträge-Einführungsgesetzgeändert werden (Patentrechts-Novelle1984),1) beschlossen BGBI. Nr.234/1984.Hauptinhalt der Novelle bildet dieHarmonisierung des österreichischenPatentrechts mit dem des EuropäischenPatentübereinkommens.Einzelne Bestimmungen der Novelle wiez. B. über den Ausbau der Service-und Informationsleistungen des Öster-reichischen Patentamts, die Erleichterungder Wiedereinsetzung, das Patentver-letzungsverfahren und Rechtshilfeer-suchen des Europäischen Patentamtssind am 1. August 1984 in Kraft ge-treten. Der Großteil der Bestimmungentritt am 1. Dezember 1984 in Kraft.

Inhaber europäischer Patente werdendarauf hingewiesen, daß mit Wirkungab 1. Dezember 1984 Übersetzungender Patentschrift ins Deutsche Spä-testens 3 Monate nach Veröffentlichungdes Hinweises auf die Erteilung deseuropäischen Patents bzw. auf die Ent-scheidung über den Einspruch imEuropäischen Patentblatt beim Öster-reichischen Patentamt einzureichen sindund innerhalb dieser Frist eine Veröffent-lichungsgebühr zu zahlen ist ( § 5 n.F.PatV-EG).

Im Anschluß an die in den Jahren 1979und 1980 erschienene Serie von Bei-trägen über die Auswirkungen des Euro-päischen Patentübereinkommens aufdas Recht der einzelnen Vertragsstaaten

AustriaLegislation

I. 1984 Amendment to patent legislation

On 23 May 1984 the Parliament (Na-tionalrat) of the Republic of Austriapassed a federal law, BGBI. No.234/1984, amending the Patent Law1970 and the Introductory Law on Pa-tent Treaties (Patentrechts-Novelle1984).1)

The main purpose of the Amendment isto harmonise Austrian patent law withthat of the European Patent Convention.

Some of its provisions entered into forceon 1 August 1984, for example those re-lating to the extension of the informationand other services provided by theAustrian Patent Office, the facilitation ofre-establishment, the patent infringementprocedure and letters rogatory from theEuropean Patent Office. Most of theprovisions will enter into force on 1December 1984.

Proprietors of European patents shouldnote that, with effect from 1 December1984, German translations of patentspecifications have to be filed with theAustrian Patent Office no later than threemonths after publication of the mentionof the grant of the European patent or ofthe decision relating to an opposition inthe European Patent Bulletin, and that apublication fee has to be paid within thisperiod (Sec. 5 ILPT as amended).

Following the series of articles publishedin 1979 and 1980 on the impact of theEuropean Patent Convention on the lawof the various Contracting States (cf. theannouncement in OJ 5/1980, p. 113), the

AutricheLégislation

I. Loi de 1984 portant modification dela législation sur les brevets

Le parlement (Nationalrat) de la Répub-lique d´Autriche a adopté le 23 mai1984 la loi fédérale (Patentrechts-No-velle 1984) BGBI. n° 234/1984, por-tant modification de la loi sur les brevetsde 1970 et de la loi d´introduction detraités en matière de brevets.Cette loi vise essentiellement à har-moniser le droit autrichien des brevetsavec les dispositions de la Conventionsur le brevet européen.Un certain nombre de dispositions decette loi, comme par exemple cellesconcernant l´extension des prestationsde l´Office autrichien des brevets enmatière de services et d´information,l´assouplissement des conditions de larestauration en l´état antérieur, les ac-tions en contrefacon et les commis-sions rogatoires émanant de l´Officeeuropéen des brevets, sont entrées envigueur le 1 er août 1984. La majeurepartie des dispositions entreront envigueur le 1 er décembre 1984.

L´attention des titulaires de brevetseuropéens est attirée sur le fait que, àcompter du 1er décembre 1984, les tra-ductions en langue allemande du fasci-cule de brevet doivent être déposéesauprès de l´Office autrichien des brevetsdans un délai maximum de trois mois àcompter de la publication au Bulletineuropéen des brevets de la mention dela délivrance du brevet européen ou, lecas échéant, de la décision relative àl´opposition, et qu´une taxe de publica-tion doit être acquittée avant l´expira-tion de ce délai (Art. 5 de la PatV-EGtelle que modifiée).

Suite à la série d´articles qui ont étépubliés en 1979 et 1980 en vue d´in-former le public des incidences de laConvention sur le brevet européen sur ledroit des différents Etats contractants

" Bundesgesetzblatt für die Republik Österreich Nr.234/1984, (105. Stück, S. 1607).

1) Bundesgesetzblatt für die Republik Osterreich Nr234/1984 (issue No 105 p. 1607).

1) Bundesgesetzblatt für die Republik Österreich Nr234/1984 (n° 105. p. 1607).

1)

Page 31: 22. November 1984 22 November 1984 22 novembre 1984 ...archive.epo.org/epo/pubs/oj1984/p549_604.pdf · tout Etat contractant doit, pour ce qui le concerne, excuser pour des motifs

(vgl. Übersicht in ABI. 5/1980, S. 113)wird das EPA in Kürze eine Ergänzungzum Beitrag für Österreich veröffent-lichen.

II. Fortschreibung der Informations-broschüre "Nationales Recht zumEPÜ" (3. Auflage)

Bezieher der Informationsbroschüre desEPA "Nationales Recht zum EPÜ" wer-den gebeten, die in Tabelle IV, Spalte 2für Österreich genannte Frist wie folgtzu ändern "3 Monate nach Bekannt-machung des Hinweises auf die Patent-erteilung bzw. die Entscheidung überden Einspruch im Europäischen Patent-blatt".

EPO will shortly be publishing a supple-ment to the article concerning Austria.

II. Updating of the information brochure"National law relating to the EPC" (3rdedition)

Subscribers to the EPO´s informationbrochure entitled "National law relating tothe EPC" are asked to amend the periodspecified for Austria in Table IV, column2, as follows: "3 months after the dateon which the mention of the grant or thedecision regarding opposition is pub-lished in the European Patent Bulletin".

(cf. récapitulation parue au JO n°5/1980, p. 113), l´OEB publiera prochaine-ment un complément concernantl´Autriche.

II. Mise à jour de la brochure d´infor-mation "Droit national relatif à laCBE" (3 e édition)

Les détenteurs de la brochure d´informa-tion de l´OEB "Droit national relatif à laCBE" sont invités à modifier commesuit le texte indiquant le délai pourl´Autriche, tableau IV, colonne 2 de cettebrochure: "3 mois à compter de la pu-blication au Bulletin européen des bre-vets de la mention de la délivrance dubrevet ou, le cas échéant, de la déci-sion relative à l´opposition".

Schweiz

Rechtsauskunft des Bundesamtsfür geistiges Eigentum (BAGE)vom 30. Mai 1984*

PatG Art. 2 (b) und 7 (c)

(EPÜ Art. 52 (4) und 54 (5))

Zur Frage, in welcher Form und unterwelchen Voraussetzungen nach Ansichtdes BA GE die Patentierung einer zweitenIndikation eines Stoffes zulässig ist,der sowohl als solcher als auch zurVerwendung in einem therapeu-tischen, chirurgischen oder diagno-stischen Verfahren bekannt ist.

LeitsatzEin Anspruch "Verwendung der Verbin-dung X zur Behandlung von ..... (Indi-kationsgebiet) ist nach schweizerischemRecht nicht zulässig. EingeschränkteAnsprüche, die auf die "Verwendungeiner Verbindung .... zur Herstellungvon Mitteln gegen ...." lauten, sindzulässig, auch wenn es sich um einezweite (oder weitere) Indikation einesbekannten Arzneistoffes handelt. In dieBeschreibung einer Patentanmeldungdürfen Angaben über die Herrichtungeines Arzneimittels (z.B. Etikettierung,Verpackung, Dosieranweisung) aufge-nommen werden.

A. Der Deutsche Bundesgerichtshof hatin einem Beschluß vom 20. September1983 einen Patentanspruch "Verwen-dung der Verbindung X zur Behand-lung von ... (Indikationsgebiet) ..."als nach deutschem Recht in einem Fallfür zulässig erklärt, in welchem sowohldie Verbindung X als solche als auchbereits eine Anwendung davon in einemam menschlichen oder tierischen Körperangewandten chirurgischen, therapeu-tischen oder diagnostischen Verfahrenbekannt sind (Amtsblatt EPA 1984,S. 26ff.; s. nun auch den gegenteiligenbritischen Entscheid, a.a.O., S. 233ff.).

SwitzerlandLegal Advice from the SwissFederal Intellectual PropertyOffice (FIPO), dated 30 May1984*

Federal Law on Patents for Inventions:Art. 2 (b) and 7c(EPC Art. 52 (4) and 54 (5))

In what form and under what condi-tions FIPO considers patentable thesecond medical use of a known sub-stance, the use of which in a thera-peutic, surgical or diagnostic methodis also known.

HeadnoteA claim for the "use of compound X totreat ... (indication) ..." is inadmissibleunder Swiss Law. Limited claimsdirected to the "use of a compound ...to produce agents against ..." are ad-missible even where they relate to asecond (or subsequent) medical use of aknown medicament. Details concerningthe formulation of a medicament (e.g.labelling, packaging or dosage) may beincluded in a patent application´sdescription.

A. In a decision dated 20 September1983 the German Federal Court of Jus-tice held a claim for the "use of com-pound X to treat ... (indication) ..."to be admissible under German lawwhere both compound X as such as wellas its use in a surgical, therapeutic ordiagnostic method to the human or ani-mal body are known (cf. OJ EPO 1/1984,p. 26 et seq.; see also the divergingUK decision, OJ EPO 5/1984, p. 233 etseq.).

SuisseRenseignement juridiquecommuniqué par l´Officefédéral de la propriété intellec-tuelle (OFPI), le 30 mai 1984*

Art. 2 (b) et 7c LBI

(Art. 52 (4) et 54 (5) de la CBE)

Forme et conditions de brevetabilitéde la deuxième application thérapeu-tique d´une substance connue en tantque telle et quant à son utilisationpour la mise en oeuvre d´une méthodede traitement chirurgical ou théra-peutique ou d´une méthode dediagnostic.

SommaireUne revendication ainsi formulée:"Utilisation du composé X pour le traite-ment de ... (champ d´application)" n´estpas admissible en droit suisse. Une re-vendication limitée ainsi libellée:"Utilisation d´un composé ... pour lafabrication de moyens destinés à com-battre ..." est admissible, même s´ils´agit d´une deuxième (ou d´une autre)application thérapeutique d´une sub-stance pharmaceutique déjà connue.Dans la description d´une demande debrevet peuvent figurer des indicationsrelatives au conditionnement du médica-ment (y compris: étiquetage, l´embal-lage, la posologie).

A. Dans une décision rendue le 20 sep-tembre 1983, le Bundesgerichtshof(Cour fédérale de justice de la Répu-blique fédérale d´Allemagne) a déclaréadmissible en droit allemand une reven-dication ayant pour objet "l´utilisationdu composé X pour le traitement de ...(champ d´application)" dans une espècedans laquelle étaient déjà connus aussibien le composé X en tant que tel qu´uneutilisation de ce composé dans uneméthode de traitement chirurgical outhérapeutique ou dans une méthode dediagnostic appliquée au corps humain ouanimal (Journal officiel de l´OEB n°

* Amtlicher Text (vgl. PMMBI. 1984 I 53). DerLeitsatz wurde vom EPA abgefaßt.

* Translation of the official text (see Swiss Patent,Design and Trademark Journal -PMMBI/FBDM 1984 I 53). The headnotewas drawn up by the EPO

* Traduction du texte officiel (cf. FDBM 1984 I,p. 53). Le sommaire a été rédigé à l´OEB.

Page 32: 22. November 1984 22 November 1984 22 novembre 1984 ...archive.epo.org/epo/pubs/oj1984/p549_604.pdf · tout Etat contractant doit, pour ce qui le concerne, excuser pour des motifs

B. Auf diesen Beschluß hinweisend, wur-den dem Amt folgende Fragen unter-breitet, wobei der Fragesteller durch-wegs davon ausging, "daß die zu schüt-zende Erfindung erfinderisch ist":

1. Wäre ein solcher Anspruch nachschweizerischem Recht ebenfalls zulässig?

2. Wäre nach schweizerischem Rechtein Anspruch "Verwendung der Verbin-dung X zur Herstellung von ... (z. B. blut-drucksenkenden) ... Mitteln" ins-besondere für eine zweite Indikation zulässig?

3. Wäre nach schweizerischem Rechtein Anspruch "Zusammensetzung beste-hend aus Stoff X und einem pharmazeu-tischen Trägermaterial zur Anwendungals ... (z. B. blutdrucksenkendes) ...Mittel" insbesondere auch für einezweite Indikation zulässig?

4. a) Ist es nach Ansicht des Amtes vonBedeutung, ob die Zusammensetzungder Arzneimittelform für die zweite Indi-kation dieselbe ist, wie diejenige für dievorbekannte erste Indikation?

b) Spielt es dabei allenfalls eine Rolle,ob die Verschiedenheit der Zusammen-setzung der zweiten Indikation von der-jenigen der vorbekannten erstenIndikationaa) in einer Verschiedenheit der Arznei-form (Tablette, Salbe usw.) oder

bb) bei gleicher Arzneiform nur in derVerschiedenheit des Trägermaterialsoder

cc) bei gleichem Trägermaterial nur inder Verschiedenheit der Dosisbereicheliegt?

c) Ist es allenfalls entscheidend, ob dieVerschiedenheit der Zusammensetzungdie Anwendung für die zweite Indikation erst ermöglicht?

Das Amt nimmt zu diesen Fragen wiefolgt Stellung:Vorauszuschicken ist, daß das Amt inverbindlicher Weise lediglich die Fragebeantworten kann, ob es aufgrundbestimmter Ansprüche ein Patent er-teilt oder verweigert. Im letztgenanntenFall muß zudem eine Überprüfung durchdas Bundesgericht aufgrund einer Ver-waltungsgerichtsbeschwerde ausdrück-lich vorbehalten bleiben.Der Entscheid über die Frage, ob ein vomAmt erteiltes Patent zu Recht besteht, istsodann Sache der ordentlichen Gerichte;hier kann das Amt lediglich eine unver-bindliche Meinung vertreten, sofern sichin den Gesetzesmaterialien entspre-chende Anhaltspunkte finden lassen.

B. With reference to the German deci-sion, FIPO has been asked the followingquestions on the premise "that the in-vention to be protected is inventive":

1. Would such a claim also be admissi-ble under Swiss law?

2. Would a claim for the "use of com-pound X to produce ... (e.g. blood-pressure lowering) agents" in parti-cular for a second medical use be ad-missible under Swiss law?

3. Would a claim for a "compositioncomprising substance X and a pharma-ceutical carrier for use as a ... (e.g.blood-pressure lowering) agent"again in particular for a second medicaluse be admissible under Swiss law?

4. (a) Does the Office consider it impor-tant whether or not the pharmaceuticalformulation is the same for the seconduse as for the previously known firstuse?

(b) Would it be relevant were the dif-ference between the composition for thesecond and that for the known first useto reside:

(aa) in a difference in pharmaceuticalformulation (tablet, ointment etc.),(bb) where the pharmaceutical formula-tion is the same, in the use of a differentcarrier, or

(cc) where the carrier is the same, onlyin a different dosage range?

(c) Is it crucial whether the difference incomposition is precisely what makes itsapplication for the second use possible?

The Office´s position with respect to theabove questions is as follows:First of all it should be noted that theOffice can give a binding reply solely tothe question as to whether it grantsor refuses patents on the basis of givenclaims. Cases of refusal are subject toreview by the Federal Supreme Court onthe basis of an appeal.

A decision as to whether a patentgranted by the Office is valid in law isthereafter a matter for the Courts. TheOffice can merely offer an opinion, pro-vided there is sufficient basis for thisin the travaux préparatoires.

1/1984, p. 26 s.: cf. également ladécision rendue en sens inverse auRoyaume-Uni, Journal officiel de l´OEB n°5/1984, p. 233 s.).

B. Les questions suivantes, faisant réfé-rence à cette décision et partant toutesde l´hypothèse que "l´invention pourlaquelle la protection est demandéeimplique une activité inventive", ontété posées à l´OFPI.

1. Une telle revendication serait-elleégalement admissible en droit suisse?

2. Une revendication ayant pour objet"l´utilisation du composé X pour la fabri-cation de moyens destinés à combattre ...(par exemple, de médicaments hypoten-seurs)" serait-elle admissible en droitsuisse, notamment pour une deuxièmeapplication thérapeutique?

3. Une revendication ayant pour objet"une composition constituée par unesubstance X et un excipient pharmaceu-tique, destinée à être utilisée commemoyen pour combattre ... (par exemple,comme médicament hypotenseur)"serait-elle admissible en droit suisse,notamment pour une deuxième applica-tion thérapeutique?

4. a) Importe-t-il, aux yeux de l´Office,que la composition de la forme pharma-ceutique prévue pour la deuxième appli-cation thérapeutique soit la même quecelle prévue pour la première applica-tion déjà connue?b) Le fait que la différence entre la com-position prévue pour la deuxième appli-cation thérapeutique et celle prévuepour la première application déjà connueréside en ce que

aa) la forme pharmaceutique (comprimé,pommade, etc.) est différente ou que,bb) la forme pharmaceutique étant lamême, l´excipient seul est différent, ouque

cc) l´excipient étant le même, la poso-logie seule est différente

joue-t-il éventuellement un rôle?c) Le fait que l´utilisation pour ladeuxième application thérapeutiquen´est possible que grâce à la différencede composition peut-il constituer un cri-tère déterminant?L´Office se prononce comme suit surces questions:Il y a lieu de préciser, à titre préliminaire,que l´Office ne saurait s´engager que surla question de savoir s´il délivre ou refuseun brevet sur la base de revendicationsdonnées. Dans la dernière éventualité,c´est au Tribunal fédéral seul qu´il in-combe de procéder à une révision à lasuite de l´introduction d´un recours dedroit administratif.Il appartient ensuite aux juridictionsordinaires de décider si le brevet a étédélivré à juste titre par l´Office. Dansce domaine, l´Office peut seulementémettre un avis qui ne saurait le lier,dans la mesure où les travaux prépara-toires fournissent des élémentspertinents.

Page 33: 22. November 1984 22 November 1984 22 novembre 1984 ...archive.epo.org/epo/pubs/oj1984/p549_604.pdf · tout Etat contractant doit, pour ce qui le concerne, excuser pour des motifs

Im einzelnen vertritt das Amt folgendeAuffassung:

1. Ein Anspruch "Verwendung der Ver-bindung X zur Behandlung von . .. (Indi-kationsgebiet) ..." ist nach schwei-zerischem Recht nicht zulässig, da einVerwendungspatentanspruch in dieserallgemeinen Fassung eine therapeu-tische Behandlung einschließt, die nachArt. 2 lit. b PatG1) von der Patentierungausgeschlossen ist. Die Rechtsprechungdes Bundesgerichts zum alten Patent-gesetz (PMMBI. 1975 I 33ff.) ist nachAnsicht des Amtes auch unter dem revi-dierten Patentgesetz noch gültig. Diesergibt sich unmißverständlich aus derBotschaft über die Änderung des Patent-gesetzes, wonach "auf dem Gebiet derTherapie, der Chirurgie und der Diagnosesämtliche Verfahren zur Behandlung desmenschlichen oder tierischen Körpers(auch wenn sie in Form einer ´Verwen-dung´ beansprucht werden)" nichtpatentiert werden können (Botschaft Nr.76.021, S. 68; BBI. 1976 II 68).

2. Hingegen ist unbestritten, daß der"gewerblich anwendbare" bzw nachschweizerischem Recht von derPatentierung nicht ausgeschlossene Teilder Verwendung, nämlich die Herrich-tung des therapeutischen Mittels, pa-tentfähig ist. Als Bedingung für die Pa-tenterteilung verlangt das Amt aber eineentsprechende Präzisierung bzw. Ein-schränkung des Patentanspruchs, näm-lich auf die "Verwendung einer Verbin-dung ... zur Herstellung von Mittelngegen ..., sofern dies durch die ur-sprüngliche Offenbarung belegt ist. Einsolcher Anspruch ist durchwegs zulässigund kann auch aufgestellt werden, wennes sich um eine zweite (oder weitere)Indikation eines bekannten Arzneistoffeshandelt. Das Amt prüft dabei nicht, obim Falle einer zweiten Indikation die Arz-neiform die gleiche ist wie bei der vor-bekannten ersten Indikation. Aus denGesetzesmaterialien läßt sich unseresWissens in dieser Beziehung nichtsableiten.

3. Das Amt müßte sodann ein Patentmit einem Anspruch "Zusammensetzungbestehend aus Stoff X und einem phar-mazeutischen Trägermaterial zur An-wendung als ... (z. B. blutdrucksen-kendes) ... Mittel" auch dann erteilen,wenn aus der Schilderung des Standesder Technik in der Beschreibung hervor-geht, daß die Zusammensetzung quali-tativ und quantitativ die gleiche ist wiefür die erste Indikation. Aus der Bot-schaft geht indessen hervor, daß dieAusnahmebestimmung von Art. 7c

The Office´s opinion on the specificquestions is as follows:1. A claim for the "use of compound Xfor the treatment of ... (indication) ..."is inadmissible under Swiss law, since ause claim drafted in such a general wayincludes therapeutic treatment, which isexcluded from patent protection by Sec-tion 2 (b) of the Swiss Law on Patentsfor Inventions (PA)1). Federal Courtcase law under the old Patents Law(Swiss Patent, Design and TrademarkJournal (PMMBI.) 1975 I 33 et seq.) isstill valid, in the Office´s opinion, underthe revised Law. This is made quite clearin the Message to Parliament on amend-ment of the Patents Law according towhich "in the field of therapy, surgeryand diagnostics methods for treatmentof the human or animal body (evenwhere claimed in ´use´ form)" are, with-out exception, not patentable (Messageto Parliament No. 76.021, p. 68; SwissFederal Gazette (BBI.), 1976 II 68).

2. It is, however, uncontested that theaspect of use "susceptible of industrialapplication" and thus not excluded frompatentability under Swiss law, i.e. theformulation of the pharmaceutical pre-paration, is patentable. For such a patentto be granted, however, the Office re-quires the claim to be limited specificallyto the "use of a compound ... for theproduction of agents against ...", pro-vided this is supported by the originaldisclosure. Such a claim is perfectlyadmissible and can also be presented fora second (or subsequent) use of a knownmedicament. The Office does not exa-mine whether, in the case of a seconduse, the pharmaceutical formulation isthe same as that for the known firstindication. The travaux préparatoiresprovide no information on this subjectas far as we know.

3. The Office would turther have to granta patent with a claim for a "compositioncomprising substance X and a pharma-ceutical carrier for use as a ... (e.g.blood-pressure lowering) ... agent" evenwhere the discussion of the prior art inthe description discloses that the com-position is qualitatively and quantita-tively the same as that for the firstmedical use. It is clear from the Messageto Parliament, however, that the excep-

L´Office adopte en particulier la positionsuivante:1. Une revendication ayant pour objet"l´utilisation du composé X pour letraitement de ... (champ d´application)"n´est pas admissible en droit suisse,étant donné que présentée sous cetteformulation générale, une revendicationd´utilisation inclut un traitement théra-peutique qui, aux termes de l´article2 (b) de la LBI1), est exclu de la breve-tabilité. L´Office estime que la juris-prudence élaborée par le Tribunal fédé-ral sous l´égide de l´ancienne loi sur lesbrevets (FBDM 1975 I, p. 33 s.) demeuretoujours valable sous la loi revisée, ainsiqu´il ressort indubitablement du mes-sage du Conseil fédéral à l´Assembléefédérale concernant la revision de la loifédérale sur les brevets d´invention, auxtermes duquel ne peuvent être brevetées"dans le domaine de la thérapie, de lachirurgie et du diagnostic, l´ensembledes méthodes de traitement du corpshumain ou animal (même si elles sontrevendiquées sous la forme d´une "utili-sation")" (message n° 76.021, p. 68;FF 1976 Il 68).

2. En revanche, il ne fait aucun douteque la partie de l´utilisation qui est "sus-ceptible d´application industrielle" etqui n´est pas exclue de la brevetabilitéen droit suisse, à savoir le conditionne-ment de l´agent thérapeutique, est bre-vetable. L´Office exige toutefois, commecondition de la délivrance du brevet,que la revendication soit précisée oulimitée en conséquence, à savoir à"l´utilisation d´un composé ... pour lafabrication de moyens destinés à com-battre ...", dans la mesure où ceci res-sort de l´exposé initial. Une telle reven-dication est parfaitement admissible etpeut être présentée même s´il s´agitd´une deuxième (ou d´une autre) appli-cation thérapeutique d´une substancepharmaceutique connue. L´Office nevérifie pas si, dans le cas d´une deu-xième application thérapeutique, laforme pharmaceutique est la même quepour la première application déjà con-nue. Les travaux préparatoires ne per-mettent pas, à notre connaissance, detirer de conclusion à ce sujet.

3. De même l´Office devrait délivrerun brevet revendiquant une "composi-tion constituée par la substance X et unexcipient pharmaceutique, destinée àêtre utilisée comme moyen pour com-battre ... (par exemple, comme médi-cament hypotenseur)", même s´il ressortde l´état de la technique exposé dans ladescription que cette composition estqualitativement et quantitativement lamême que pour la première application.Il ressort toutefois du message du Con-seil fédéral que l´exception visée à l´arti-

1) Anmerkung der Redaktion: Art. 2 (b) PatG lautetwie folgt:

Von der Patentierung sind ausgeschlossen: Ver-fahren der Chirurgie, Therapie und Diagnostik.die an menschlichen oder tierischen Körpernangewendet werden."

1) Editorial note: The text of section 2 (b) PA isas follows:

"The following shall not be patentable: ...methods of surgical or therapeutic treatmentand of diagnosis applied to the human bodyor to the bodies of animals." (WIPO translation)

" Note de la rédaction: l´article 2 (b) de la LBI selit comme suit:

"Ne peuvent être brevetées: les méthodes detraitement chirurgical ou thérapeutique et lesméthodes de diagnostic appliquées au corpshumain ou animal. "

Page 34: 22. November 1984 22 November 1984 22 novembre 1984 ...archive.epo.org/epo/pubs/oj1984/p549_604.pdf · tout Etat contractant doit, pour ce qui le concerne, excuser pour des motifs

PatG2) und der zugehörige Arzneimittel-anspruch nur für die erste Indikationvorgesehen sind; eine weitere Anwen-dung auf dem Gebiet der Therapie,Chirurgie oder Diagnose ist nicht mehrneu (Botschaft, a.a.O., S. 71; BBI. 1976II 71). Nach Ansicht des Amtes solltedaher jede zweite und weitere Indika-tion nur in Form des Verwendungspa-tentanspruches, wie er in der zweitenFrage umschrieben ist, beanspruchtwerden oder gegebenenfalls in der äqui-valenten Form eines "Verfahrens zurHerstellung eines Arzneimittels ge-gen ...".

4. Zu den letzten Fragen (s. oben B, Ziff.4) kann das Amt nicht Stellung nehmen,weil es diese Punkte bei der Patent-gesuchsprüfung nicht in Erwägung zuziehen hat; der Arzneimittelsektor unter-steht nicht der amtlichen Vorprüfung3).Zudem finden sich in den Gesetzes-materialien keine nützlichen Hinweise.

Das Amt möchte aber darauf aufmerk-sam machen, daß alle diese Einzelfragennur unter dem Blickwinkel der Neuheitgestellt wurden, da der Fragestellerbei allen Fragen davon ausging, "daßdie zu schützende Erfindung erfinde-risch ist". Das Amt ist indessen der An-sicht, daß der Richter bei der Anwen-dung von Art. 26 Ziff. 1 PatG4) im fort-laufenden Denkprozeß vorerst unter-sucht, welche Merkmale eines be-stimmten Anspruchs noch neu sind, underst dann prüft, ob diese (neuen) Merk-male bzw. ein Teil davon eine erfinde-rische Tätigkeit begründen.

Im übrigen muß amtlicherseits auch dieFrage offenbleiben, welche Operationennoch zur Herrichtung eines Arzneimittelsgehören und insbesondere, ob die Eti-kettierung bzw. Verpackung sowie dasBeilegen einer Dosierungsanweisungnoch dazuzuzählen sind. Das Amt würdeauf jeden Fall solche Angaben in derBeschreibung eines Patentgesuchs zu-lassen, dessen Patentanspruch auf dieVerwendung eines Stoffes zur Her-stellung eines bestimmten Arzneimittelslautet.

tions contained in Section 7c PA2) andclaims thereunder to a medicament areintended only for the first use; a subse-quent use in the field of therapy, sur-gery or diagnostics is no longer novel(Message to Parliament, loc. cit., p. 71;Federal Gazette 1976 II 71). The Officeconsiders that every second or subse-quent use should therefore be presentedonly in the form of a use claim as set outin the second question or, where appli-cable, in the equivalent form of a "Pro-cess for the production of a medica-ment against ...".

4. The Office is unable to give an opi-nion on the last questions (cf. B.4.above) since it is not required to con-sider these points in its examinationof a patent application, the pharmaceu-tical field not being subject to officialpreliminary examination3). Moreover,there is nothing of relevance in the tra-vaux préparatoires.

The Office would, nevertheless, like topoint out that all these specific questionswere posed solely from the point ofview of novelty, the premise being "thatthe invention for which protection issought is inventive". However, the Officeconsiders that logically, when applyingSection 26 (1) PA4), a judge would firststudy which features of a given claimare still new and only then would heexamine whether those (new) features,or some of them, involve an inventivestep.

The Office is also unable to rule on whichoperations still appertain to the formula-tion of a medicament and, in particular,whether the labelling, packaging or en-closure of dosage instructions shouldalso be included. The Office would inany event allow such information in apatent application´s description wherethe claim is directed to the use of a sub-stance for the production of a particularmedicament.

cle 7c de la LBI2) et que les revendica-tions de médicament correspondantesne sont prévues que pour la premièreapplication. Toute autre utilisation dansle domaine de la thérapie, de la chirurgieou du diagnostic n´est plus nouvelle(message précité, p. 71; FF 1976 Il 71).L´Office estime par conséquent quetoute deuxième (ou toute autre) appli-cation thérapeutique ne devrait êtrerevendiquée que sous la forme d´unerevendication d´utilisation, telle qu´elleest définie dans la deuxième questionou, le cas échéant, sous la forme équi-valente d´un "procédé de fabricationd´un médicament contre ...".4. L´Office ne peut se prononcer sur lesdernières questions (voir ci-dessus B -point 4), étant donné qu´il n´a pas àen tenir compte lors de l´examen de lademande de brevet. Le secteur pharma-ceutique n´est pas soumis à l´examenpréalable.3) En outre, les travaux pré-paratoires ne fournissent pas de préci-sions utiles à ce sujet.

L´Office aimerait toutefois attirer l´atten-tion sur le fait que toutes ces questionsn´ont été posées que sous l´angle de lanouveauté, puisqu´il a été supposé que"l´invention pour laquelle la protectionétait demandée impliquait une activitéinventive". L´Office estime toutefois quele juge, lorsqu´il applique l´article 26 (1)de la LBI4), détermine tout d´abord,dans une seule et même démarche in-tellectuelle, quelles sont les caractéris-tiques d´une revendication donnée quisont encore nouvelles et n´examinequ´ensuite si ces caractéristiques (nou-velles) ou certaines d´entre elles peuventêtre considérées comme impliquant uneactivité inventive.Par ailleurs, l´Office ne peut pas nonplus se prononcer sur la question desavoir quelles sont les opérations quientrent encore dans le conditionnementd´un médicament et notamment si l´éti-quetage ou l´emballage ainsi que lesindications de posologie en font encorepartie. En tout état de cause, l´Officeadmettrait de telles indications dans ladescription d´une demande de brevetdont la revendication porte sur l´utilisa-tion d´une substance en vue de la fabri-cation d´un médicament déterminé.

2) Anmerkung der Redaktion: Art. 7c PatG lautetwie folgt:

"Stoffe und Stoffgemische, die als solche, abernicht in bezug auf ihre Verwendung in einemchirurgischen, therapeutischen und diagno-stischen Verfahren nach Artikel 2 Buchstabe bzum Stand der Technik gehören oder Gegenstand eines älteren Rechts sind, gelten als neusoweit sie nur für eine solche Verwendungbestimmt sind."

3) Anmerkung der Redaktion: Das BAGE prüftPatentanmeldungen grundsätzlich nur aufPatentfähigkeit und Patentierungsverbote (Art.59 PatG). Die "amtliche Vorprüfung", ob eine Er-findung neu ist und sich nicht in naheliegenderWeise aus dem Stand der Technik ergibt. erfolgtnur für bestimmte Erfindungen auf dem Gebietder Textilveredelung und der Zeitmessungstech-nik (Art. 87 (21 PatG).

4) Anmerkung der Redaktion: Feststellung derNichtigkeit des Patents durch den Richter man-gels Patentierbarkeit.

2) Editorial note: The text of Section 7c PA isas follows:"Substances or compounds which are included

as such in the state of the art or are the subject of a prior right but which do not meetthese conditions with respect to their use forthe implementation of a method of surgical ortherapeutic treatment or of diagnosis (Section2 (b)). shall be deemed to be new to the extentthat they are intended solely for such use(WIPO translation)

3) Editorial note: FIPO generally examines patentapplications solely on the basis of patentabilityand the exclusions therefrom (Section 59 PA).The "official preliminary examination as towhether an invention is new and is not obviousfrom the state of the art is conducted only forcertain inventions in the fields of improvementof textile fibres and chronometric technology(Section 87 (2) PA).

4) Editorial note: Declaration by a court that apatent is invalid on the grounds of non-patentability.

2) Note de la rédaction l´article 7c de la LBI selit comme suit:

"Les substances ou compositions qui, en tantque telles, sont comprises dans l´état de la tech-nique ou font l´objet d´un droit antérieur maisne répondent pas à ces conditions quant àleur utilisation pour la mise en oeuvre d´uneméthode de traitement chirurgical ou théra-peutique ou d´une méthode de diagnostic (art.2 let b). sont réputées nouvelles dans lamesure où elles ne sont destinées qu´à unetelle utilisation.

3) Note de la rédaction: l´OFPI examine en prin-cipe les demandes de brevet uniquement quantà la brevetabilité et à l´exclusion de la breve-tabilité (art 59 LBI). Seules certaines inven-tions dans le domaine du perfectionnementdes fibres textiles et de la technique de la mesuredu temps sont soumises à l´examen préalableen vue de déterminer si elles sont nouvelles etsi elles ne découlent pas d´une manière évidentede l´état de la technique (art. 87 (2) LBI).

4) Note de la rédaction: constatation par le juge dela nullité du brevet pour défaut de brevetabilité.

Page 35: 22. November 1984 22 November 1984 22 novembre 1984 ...archive.epo.org/epo/pubs/oj1984/p549_604.pdf · tout Etat contractant doit, pour ce qui le concerne, excuser pour des motifs

INTERNATIONALE VERTRÄGEPCT

Änderung der Vereinbarungzwischen der WIPO und der EPOnach dem PCT 1)

1. Änderung des Artikels 8 Absatz 2und des Anhangs B Teil II der Verein-barung

1.1 Die Versammlung des PCT-Ver-bandes hat auf ihrer 11. (7. außeror-dentlichen) Tagung am 3. Februar 1984eine Änderung des Artikels 8 Absatz 2der Vereinbarung zwischen der WIPOund der EPO nach dem PCT (im fol-genden "Vereinbarung" genannt) geneh-migt.

Mit dieser Änderung soll die Fassungdieser Bestimmung an die von der Ver-sammlung des PCT-Verbandes auf ihrer5. Tagung vom 9. bis 16. Juni 19802)

geänderte Fassung der Regel 41.1 ange-paßt werden.

1.2 Eine entsprechende Änderung desAnhangs B Teil II der Vereinbarungist zwischen der EPO und der WIPO ver-einbart worden.

1.3 Beide Änderungen sind im PCT-Blatt(Nr. 6/1984, S. 664) veröffentlicht wor-den und am 1. März 1984 wirksam ge-worden.

2. Änderung des Artikels 3 Absatz 2und des Anhangs A der Vereinbarung2.1 Die Versammlung des PCT-Ver-bandes hat auf ihrer 12. (8. außerordent-lichen) Tagung am 28. September 1984eine Änderung des Artikels 3 Absatz 2der Vereinbarung über die Zuständigkeitdes EPA als mit der vorläufigen inter-nationalen Prüfung beauftragte Behördegenehmigt.

2.2 Eine entsprechende Änderung desAnhangs A ist zwischen der EPO undder WIPO vereinbart worden.

2.3 Beide Änderungen sind im PCT-Blatt Nr. 25/1984, S. 3095 veröffent-licht worden und am 25. Oktober 1984wirksam geworden.

3. Änderung der Aufstellung der Ge-bühren und sonstigen Kosten in An-hang B Teil I der Vereinbarung

3.1 Die EPO hat der WIPO eine Än-derung der Aufstellung der Gebühren undsonstigen Kosten in Anhang B Teil I derVereinbarung mitgeteilt.

Mit dieser Änderung soll die Aufstellungan den Beschluß des Verwaltungsratsder Europäischen Patentorganisationvom 8. Juni 1984 über dieNeufestsetzung der in Artikel 2 derGebührenordnung vorgesehenen Ge-bühren3) sowie an den Beschluß desVerwaltungsrats vom 9. Dezember 1983über die Ermäßigung der Gebühren fürdie internationale Recherche und vor-

INTERNATIONAL TREATIESPCT

Amendment of the Agreementbetween WIPO and the EPOunder the PCT 1)

1. Amendment to Article 8 (2) andAnnex B, Part II, of the Agreement

1.1 At its 11th (7th extraordinary)session, the Assembly of the PCT Unionapproved, on 3 February 1984, anamendment to Article 8 (2) of the Agree-ment between WIPO and the EPO underthe PCT (hereinafter the "Agreement").

The purpose of the amendment is toharmonise the text of this provisionwith that in Rule 41.1, as amendedby the Assembly of the PCT Union atits 5th session held from 9 to 16 June19802).

1.2 The EPO and WIPO have agreed toan amendment resulting therefrom toAnex B, Part II, to the Agreement.

1.3 These two amendments were pub-lished in the PCT Gazette (No. 6/1984,p. 664), and took effect on 1 March1984.

2. Amendment to Article 3 (2) andAnnex A of the Agreement2.1 At its 12th (8th extraordinary)session, the Assembly of the PCT Unionapproved, on 28 September 1984, anamendment to Article 3 (2) of theAgreement, regarding the competenceof the EPO as an InternationalPreliminary Examining Authority.

2.2 The EPO and WIPO have agreed toan amendment resulting therefrom toAnnex A.

2.3 These two amendments were pub-lished in PCT Gazette No. 25/1984, p.3095, and took effect on 25 October1984.

3. Amendment to the Schedule of feesand other charges set out in Annex B,Part I, to the Agreement3.1 The EPO has notified WIPO of anamendment to the Schedule of fees andother charges set out in Annex B, Part I,to the Agreement.

The purpose of the amendment is tobring the Schedule into line with theDecision of the Administrative Council ofthe European Patent Organisation of 8June 1984 revising the amounts of thefees provided for in Article 2 of the Rulesrelating to Fees3), and with the Decisionof the Administrative Council of 9December 1983 on the reduction of feesfor the international search and pre-

TRAITES INTERNATIONAUXPCT

Modification de l´Accord entrel´OMPI et l´OEB concernant lePCT1)

1. Modification de l´article 8.2) et del´annexe B, partie Il de l´Accord

1.1 Lors de sa onzième session (7e

session extraordinaire), l´Assemblée del´Union du PCT a approuvé, le 3 février1984, une modification de l´article 8.2)de l´accord entre l´OMPI et l´OEBconcernant le PCT (ci-après dénommél´ "Accord").

La modification a eu pour but d´har-moniser le texte de cette dispositionavec celui de la règle 41.1, tel quemodifié par l´Assemblée de l´Union duPCT lors de sa cinquième session, quis´est tenue du 9 au 16 juin 1980 2).

1.2 Une modification consécutive del´annexe B, partie II de l´Accord a étéconvenue entre l´OEB et l´OMPI.

1.3 Ces deux modifications ont étépubliées dans la Gazette du PCT (n°6/1984, p. 668) et sont entrées envigueur le 1er mars 1984.

2. Modification de l´article 3.2) et del´annexe A de l´Accord2.1 Lors de sa douzième session (8e

session extraordinaire), l´Assemblée del´Union du PCT a approuvé, le 28 sep-tembre 1984, une modification de l´article3.2) de l´Accord, relatif à la compétencede l´OEB en qualité d´administrationchargée de l´examen préliminaire inter-national.2.2 Une modification consécutive del´annexe A a été convenue entre l´OEBet l´OMPI.

2.3 Ces deux modifications ont étépubliées dans la Gazette du PCTn° 25/1984, p. 3102, et sont entréesen vigueur le 25 octobre 1984.

3. Modification du tableau des taxes etautres droits figurant à l´annexe B,partie I de l´Accord3.1 L´OEB a notifié à l´OMPI une modi-fication du tableau des taxes et autresdroits figurant à l´annexe B, partie I del´Accord.La modification a pour but d´adapterle tableau à la décision du Conseild´administration de l´Organisation euro-péenne des brevets du 8 juin 1984 por-tant révision du montant des taxes pré-vues à l´article 2 du réglement relatif auxtaxes3), ainsi qu´à la décision du Conseild´administration du 9 décembre 1983portant réduction des taxes de rechercheet d´examen préliminaire relatives aux

1) Vgl. ABI. 4/1978, S. 249 f.; 4/1979, S. 139;10/1981. S. 470 und 530.

2) Vgl. ABI. 9/1980, S. 315.3) Vgl. ABI. 7/1984, S. 297 f.

1) See OJ 4/1978, p. 249 et seq., 4/1979, p. 139;10/1981, p. 470, and 11/1981. p. 530.

2) See OJ 9/1980, p. 315.3) See OJ 7/1984, p. 297 et seq.

1) Cf. JO n° 4/1978, p. 249 s.; n° 4/1979. p. 139;n° 10/1981, p. 470 et n° 11/1981, p. 530.

2) Cf. JO n° 9/1980, p. 315.3) Cf. JO n° 7/1984, p. 297 s.

Page 36: 22. November 1984 22 November 1984 22 novembre 1984 ...archive.epo.org/epo/pubs/oj1984/p549_604.pdf · tout Etat contractant doit, pour ce qui le concerne, excuser pour des motifs

läufige Prüfung einer internationalenAnmeldung zugunsten der Staatsange-hörigen von Entwicklungsländern1) inder Fassung vom 9. Juni 19842) ange-paßt werden.

3.2 Die Änderung der Aufstellung derGebühren und sonstigen Kosten ist imPCT-Blatt Nr. 25/1984, S. 3097 veröf-fentlicht worden und wird am 3. Januar1985 wirksam.

4. Geänderte Fassung der oben ge-nannten Bestimmungen der Verein-barung

Artikel 3Zuständigkeit der Behörde

(1) [Keine Änderung

(2) Vorbehaltlich des Absatzes 3 wird dieBehörde, sofern das Anmeldeamt die Be-hörde zu diesem Zweck angegeben hat,als mit der internationalen vorläufigenPrüfung beauftragte Behörde für alleinternationalen Anmeldungen tätig, diebei dem Anmeldeamt eines Vertrags-staats oder dem für einen Vertrags-staat handelnden Anmeldeamt einge-reicht worden sind und für die dieBehörde, das Königl. Patent- und Regi-strieramt von Schweden oder das Öster-reichische Patentamt als InternationaleRecherchenbehörde tätig wird oder tätiggeworden ist.(3) [Keine Änderung .. .]

Artikel 8

Gebühren und sonstige Kosten(1) [Keine Änderung ... .]

(2) Die Behörde erstattet zu den in An-hang B angegebenen Bedingungen undin dem dort vorgesehenen Umfang dieinternationale Recherchengebühr ganzoder teilweise, sofern ein internationalerRecherchenbericht ganz oder teilweiseauf die Ergebnisse einer von dieserBehörde durchgeführten Recherche ge-stützt werden kann.

ANHANG ASPRACHEN ZUM ZWECKE DERANWENDUNG VON ARTIKEL 3

ABSATZ 1 DER VEREINBARUNGZum Zwecke der Anwendung von Arti-kel 3 Absatz 1 der Vereinbarung müssendie Anmeldungen in einer der folgendenSprachen eingereicht werden:Deutsch,Englisch,Französisch,Niederländisch, wenn die internationaleAnmeldung bei der Zentralbehörde fürden gewerblichen Rechtsschutz derNiederlande oder bei der Zentralbe-hörde für den gewerblichen Rechts-schutz Belgiens eingereicht wird.

liminary examination of an internationalapplication in favour of nationals ofdeveloping countries1), as amended on 9June 19842).

3.2 The amendment to the Schedule offees and other charges was published inPCT Gazette No. 25/1984, p. 3097 andwill take effect on 3 January 1985.

4. Text of the amended provisions inthe Agreement

Article 3

Competence of Authority(1) (No change ...]

(2) Subject to paragraph (3), theAuthority undertakes to act as an Inter-national Preliminary ExaminingAuthority, provided that the receivingOffice specifies the Authority for thatpurpose, for all international applications which are filed with the re-ceiving Offices of, or acting for, all Con-tracting States and with respect towhich the Authority, the Royal Patentand Registration Office of Sweden or theAustrian Patent Office acts or has actedas an International Searching Authority.

(3) [No change ...]

Article 8Fees and Other Charges

(1) [No change ...]

(2) The Authority shall, to the extent andunder the conditions set out in Annex Bof this Agreement, refund the whole orpart of the international search fee paidwhere an international search report canbe wholly or partially based on theresults of an earlier search made by theAuthority. "

ANNEX ALANGUAGES SPECIFIED FOR THE

PURPOSES OF ARTICLE 3 (1) OF THEAGREEMENT

For the purposes of Article 3 (1) of theAgreement, applications shall be filed inone of the following languages:

English,French,German,Dutch, where the international application is filed with the central industrialproperty Office of the Netherlands, orwith the central industrial property officeof Belgium.

demandes internationales déposées pardes ressortissants des pays en déve-loppement1), telle que modifiée le 9 juin19842).

3.2 La modification relative au tableaudes taxes et autres droits a étépubliée dans la Gazette du PCTn° 25/1984 et entrera en vigueur le 3janvier 1985.

4. Texte des dispositions modifiées del´Accord:

Article 3

Compétence de l´Administration(1) [Sans changement ...](2) Sous réserve du paragraphe 3,l´Administration agit en qualité d´admini-stration chargée de l´examen préli-minaire international, pourvu que l´officerécepteur ait indiqué l´Administration àcette fin, à l´égard de toute demande in-ternationale déposée auprès de l´officerécepteur de tout Etat contractant ouagissant pour tout Etat contractant, etconcernant laquelle soit l´Administration,soit l´Office autrichien des brevets, soitl´Office royal des brevets et de l´en-registrement de la Suède agit ou a agien qualité d´administration chargée de larecherche internationale.

(3) [Sans changement...]

Article 8Taxes et droits

(1) [Sans changement ...]

(2) L´Administration remboursera, selonles modalités spécifiées à l´annexe Bdu présent Accord, et dans les limitesde celles-ci, tout ou partie de la taxede recherche internationale lorsqu´unrapport de recherche internationale peutse baser entièrement ou partiellementsur les résultats d´une recherche effec-tuée par ladite Administration.

ANNEXE AINDICATION DES LANGUES AUX FINS

DE L´ARTICLE 3 (1) DE L´ACCORD

Aux fins de l´article 3.1) de l´Accord, lesdemandes doivent être déposées dansl´une des langues suivantes:

l´allemand,l´anglais,

le francais,le néerlandais, en ce qui concerne lesdemandes internationales déposées auprès du Service central de la propriétéindustrielle des Pays-Bas ou auprès duService central de la propriété indus-trielle de la Belgique.

1) Vgl. ABI. 1/1984, S. 3 f.2) Vgl. ABI 7/1984, S. 297 f.

1) See OJ 1/1984, p. 3 et seq.2) See OJ 7/1984. p. 297 et seq.

1) Cf. JO n° 1 1984. p 3 s2) Cf. JO n° 7 1984 p 297 s

...]

Page 37: 22. November 1984 22 November 1984 22 novembre 1984 ...archive.epo.org/epo/pubs/oj1984/p549_604.pdf · tout Etat contractant doit, pour ce qui le concerne, excuser pour des motifs
Page 38: 22. November 1984 22 November 1984 22 novembre 1984 ...archive.epo.org/epo/pubs/oj1984/p549_604.pdf · tout Etat contractant doit, pour ce qui le concerne, excuser pour des motifs

PCT

Änderung des Gebührenverzeichnisses

Die Versammlung des PCT-Verbandeshat in der Zeit vom 24. bis 28. Sep-tember 1984 in Genf ihre 12. (8. außer-ordentliche) Tagung abgehalten undmit Wirkung ab 1. Januar 1985 dieBeträge der im Gebührenverzeichnisim Anhang zur PCT-Ausführungsordnungaufgeführten Gebühren neu festge-setzt sowie Position 2 des Gebühren-verzeichnisses geändert. Die Gegen-werte der neuen Beträge in anderenWährungen als Schweizer Frankensind aufgrund der am 24. September1984 geltenden Wechselkurse fest-gelegt worden1). Das Gebührenver-zeichnis in der ab 1. Januar 1985 ver-bindlichen Fassung wird nachstehendwiedergegeben.

Gebührenverzeichnis

Gebühr Betrag

1. Grundgebühr:(Regel 15.2 Absatz a)falls die internationale Anmeldung nichtmehr als 30 Blätter enthält

654 Schweizer Frankenfalls die internationale Anmeldung mehrals 30 Blätter enthält

654 Schweizer Frankenund 13 Franken für jedes30 Blätter übersteigendeBlatt

2. Bestimmungsgebühr:(Regel 15.2 Absatz a)

158 Schweizer Frankenfür jede gebührenpflichtigeBestimmung, höchstensjedoch 1580 SchweizerFranken (jede über 10hinausgehende Bestim-mung ist gebührenfrei)

3. Bearbeitungsgebühr:(Regel 57.2 Absatz a)

200 Schweizer Franken4. Zusätzliche Bearbeitungsgebühr:(Regel 57.2 Absatz b)

200 Schweizer FrankenZuschlagsgebühr5. Zuschlagsgebühr wegen verspäteterZahlung:(Regel 16bis.2 Absatz a)

Mindestbetrag:248 Schweizer FrankenHöchstbetrag:624 Schweizer Franken

PCT

Amendment of the Schedule of FeesThe Assembly of the PCT Union held itstwelfth (8th extraordinary) session inGeneva from 24 to 28 September 1984.It revised with effect as of 1 January1985 the amounts of the fees appearingin the Schedule of Fees annexed to theRegulations under the PCT and amendedno. 2 of the Schedule of Fees. Theequivalents of the revised amounts incurrencies other than Swiss francs havebeen determined on the basis of theexchange rates applicable on 24 Sep-tember 19841). The Schedule of Feesbinding as from 1 January 1985 is re-produced below.

Schedule of Fees

Fees Amounts

1. Basic Fee:(Rule 15.2(1))if the international application containsnot more than 30 sheets

654 Swiss francsif the international application containsmore than 30 sheets

654 Swiss francs plus13 Swiss francs for eachsheet in excess of 30sheets

2. Designation Fee:(Rule 15.2(a))

158 Swiss francs perdesignation for which thefee is due, with amaximum of 1580 Swissfrancs, any such designa-tion in excess of 10 beingfree of charge

3. Handling Fee:(Rule 57.2(a))

200 Swiss francs4. Supplement to the Handling Fee:(Rule 57.2(b))

200 Swiss francsSurcharges

5. Surcharge for late payment:(Rule 16bis.2(a))

Minimum:248 Swiss francsMaximum:624 Swiss francs

PCT

Modification du barème de taxesL´Assemblée de l´Union PCT a tenu sadouzième session (8e session extra-ordinaire) à Genève du 24 au 28 sep-tembre 1984. Elle a procédé, avec effetau 1er janvier 1985, à la révision dumontant des taxes mentionnées dans lebarème de taxes figurant en annexe aurèglement d´exécution du PCT ainsiqu´à une modification du poste 2 dubarème de taxes. Les contre-valeurs desnouveaux montants exprimées dans desmonnaies autres que le franc suisse ontété fixées sur la base des cours dechange en vigueur au 24 septembre19841). La barème de taxes est reprisci-après dans la version applicable àcompter du 1er janvier 1985.

Barème de taxesTaxes Montants1. Taxe de base:(règle 15.2.a))si la demande internationale ne com-porte pas plus de 30 feuilles

654 francs suissessi la demande internationale comporteplus de 30 feuilles

654 francs suisses plus13 francs suisses parfeuille à compter de la 31 e

2. Taxe de désignation:(règle 15.2.a))

158 francs suisses pardésignation soumise à lataxe, avec un maximum de1580 francs suisses, toutedésignation (soumise à lataxe) à compter de la 11eétant gratuite

3. Taxe de traitement:(règle 57.2,a))

200 francs suisses4. Supplément à la taxe de traitement:(règle 57.2.b))

200 francs suissesSurtaxes

5. Surtaxe pour paiement tardif:(règle 16bis.2.a))

Minimum:248 francs suissesMaximum:624 francs suisses

1) Vgl. PCT-Blatt Nr. 25/1984.S .3095. 1) Cf. PCT Gazette No . 25/1984, p. 3095. 1) Cf. Gazette du PCT n° 25/1984, p. 3101

Page 39: 22. November 1984 22 November 1984 22 novembre 1984 ...archive.epo.org/epo/pubs/oj1984/p549_604.pdf · tout Etat contractant doit, pour ce qui le concerne, excuser pour des motifs

HINWEISE AUF LITERATURZUM EUROPÄISCHEN

PATENTRECHT

Unter dieser Rubrik werden in regel-mäßigen Abständen bibliographischeHinweise auf dem Europäischen Pa-tentamt bekannt gewordene Literaturzum europäischen Patentrecht und zum"europäisierten" nationalen Patentrechtder Vertragsstaaten veröffentlicht.

Diese Hinweise erheben keinen An-spruch auf Vollständigkeit und sind nichtals Empfehlung der aufgeführten Bücheroder Zeitschriftenaufsätze durch dasEPA zu verstehen. Sie dienen aus-schließlich der Information der am euro-päischen Patentrecht Interessierten undbedeuten nicht, daß das EPA mit in denaufgeführten Werken vertretenen Auf-fassungen übereinstimmt.Literaturbestellungen nimmt die Biblio-thek des EPA nicht entgegen, da siekeine öffentliche Bibliothek ist. Büchersind über den Fachbuchhandel zu be-ziehen und Zeitschriftenaufsätze könnenin öffentlichen Fachbibliotheken ange-fordert werden.

INFORMATION CONCERNINGLITERATURE ON EUROPEAN

PATENT LAW

Under this heading, bibliographical dataconcerning literature on European patentlaw and "Europeanised" national patentlaw of the Contracting States which hascome to the attention of the EuropeanPatent Office are published at regularintervals.This information does not claim to becomprehensive and should not be under-stood as recommendation by the EPOof the books or periodical articles listed.Its sole purpose is to keep those in-terested in European patent lawinformed and it in no way impliesendorsement by the EPO of the viewsexpressed in the works listed.

The library of the European Patent Officedoes not accept orders for literaturesince it is not a public library. Booksshould be obtained from specialist book-sellers. Articles appearing in periodicalscan be ordered from specialist publiclibraries.

RENSEIGNEMENTS BIBLIO-GRAPHIQUES RELATIFS AU

DROIT EUROPEEN DESBREVETS

Sous cette rubrique paraissent régulière-ment des renseignements bibliographi-ques sur la littérature relative au droiteuropéen et à I´ "euro-droit" des brevetsdes Etats contractants dont l´OEB auraeu connaissance.

Ces renseignements ne prétendent pasêtre exhaustifs. Ils n´impliquent pasdavantage une quelconque recom-mandation de la part de l´Office à proposdes ouvrages ou articles de revue men-tionnés. Ils sont exclusivement destinésà informer les milieux intéressés. Lesopinions exprimées dans ces ouvragesou articles n´engagent que leurs auteurs.

N´étant pas une bibliothèque publique,la bibliothèque de l´OEB n´accepte pasde commandes d´ouvrages ou de revuesLes livres sont en vente dans leslibrairies spécialisées et il est possiblede se procurer les articles de publica-tions auprès des bibliothèques publiquesspécialisées.

Abkürzungen / Abbreviations / Abréviations

APLA American Patent Law Association Quarterly (US)BIE Bijblad bij De Industriele Eigendom (NL)

CIPA The Journal of the Chartered Institute of Patent Agents (GB)

EIPR European Intellectual Property Review (GB)GRUR Gewerblicher Rechtsschutz und Urheberrecht (DE)

GRUR Int. Gewerblicher Rechtsschutz und Urheberrecht, Internationaler Teil (DE)IIC International Review of Industrial Property and Copyright Law (DE)Ind. Prop. Industrial Property (WIPO)

Mitt. Mitteilungen der deutschen Patentanwälte (DE)Prop. Ind. Propriété industrielle (OMPI)

WPI World Patent Information (GB)

Bücher / Books / Livres

Beier, F. K. / Haertel, K. / Schricker, G. (Hrsg./ed./éd.)

Münchner Gemeinschaftskommentar.Heymanns, München, 1 3 Lieferung, 112, 256, 84.

Blanco White, T. A.

Patents for inventions and the protection of industrialdesigns.Stevens & Sons, London, 1983, XXXV, 304.

Gall, G.

European Patent Applications Questions and Answers.Oyez Longman, London, 1984, 231.

WIPO/OMPI (Hrsg/ed./éd.)

IPK, Internationale Patent Klassifikation / IPC, InternationalPatent Classification / CIB, Classification internationale desbrevets4. revidierte Ausgabe 1984 / 4th Edition 1984 / Quatrièmeédition, 1984.

Phillips, J. / Hoolahan, M. J.

Employees´ inventions in the United Kingdom: Law andpractice.ESC Publishing, Oxford, 1982, XVIII, 164.

Volmer, B. / Gaul, D.

Arbeitnehmererfindergesetz.2. Auflage, C. H. Beck, München, 1983, XXIV, 2012.

2. Zeitschriftenaufsätze / Articles

Asahina, S.Notes for foreigners filing Japanese patent applications.Journal of the Japanese Group of AIPPI, International edi-tion, 2, 1983, 53 64.

Bardehle, H.

Period of Grace for Inventive Disclosure.Ind. Prop., 1982, 279.

Bartels, B.Die Vorteile des PCT für den Anmelder.Mitt, 9, 1983, 162 167.

Beier, F. K.

One hundred years of international co-operation: the rpleof the Paris Convention in the past, present and future.IIC, 1, 1984, 1 20.

Page 40: 22. November 1984 22 November 1984 22 novembre 1984 ...archive.epo.org/epo/pubs/oj1984/p549_604.pdf · tout Etat contractant doit, pour ce qui le concerne, excuser pour des motifs

Beier, F. K.

Governmental promotion of innovation and the patentsystem.IIC, 1982, 545565.

Beton, J.Recent Developments in the PCT and the EPO.CIPA, 1983 ,283.

Bruchhausen, K.Wann gehört die Anwendung eines Stoffes (oder Stoffge-misches) zur therapeutischen Behandlung des menschlichen(oder tierischen) Körpers oder in Diagnostizierverfahren,die am menschlichen (oder tierischen) Körper vorgenommenwerden, zum Stand der Technik?GRUR Int., 1982, 641.

Burton, D. / Foster, J. H.

Reducing the high costs of patént litigation: a practicalguide.Patent and Trademark Review, 10, 1983, 395 413.

Cawthra, B.Role of the lawyer in the EPO.EIPR. 3, 1984. 63 64.

Chisum, D.Copyright, computer programs and the Apple Cases: a com-promise solution.EIPR, 9, 1983, 233 237.

Coaracy, R. G.

Guidelines for the planning and organisation of a patentdocumentation and information centre.WPI, 1, 1983, 4 9.

Czekay, H. F.Deduktive Formulierung von Patentansprüchen.GRUR, 2, 1984, 83 90.

Daniszewski, J. P.

Patent Information Retrieval in Practice.APLA, 11, 1983, 120 125.

Delorme, J.

Two Decisions of the Administrative Council of the EPO.WPI, 1983, 207208.

Delorme, J.

Organising the distribution of certain EPO data bases.WPI, 2, 1984, 7475.

Dietz, A.

Das Problem des Rechtsschutzes von Computerprogrammenin Deutschland und Frankreich: die kategoriale Heraus-forderung des Urheberrechts.BIE, 11, 1983, 305 311.

Dinné, E.

Zum Schutzbereich der 2. Indikation.Mitt., 6, 1984, 105 106.

Dolder, F.

Nachwirkende Nichtangriffspflichten des Arbeitnehmererfin-ders im schweizerischen Recht.GRUR Int., 1982, 158.

Gall, G.

La délivrance du brevet: l´articulation entre la phase euro-péenne et la phase nationale.Echanges ASPI, 14, 1983, 1 22.

Gall, G.

Jahresgebühren in Sonderverfahren des EPÜ.Mitt., 9, 1984, 161 166.

Guerin, M.

Colloque de Munich: rédaction et interprétation des reven-dications, 2729 avril 1983.Echanges ASPI, 14, 1983, 23 31.

Häußer, E.Wechselwirkungen zwischen Erfindungshöhe, Anspruchs-fassung und Schutzumfang.GRUR, 1982. 723.

Kedrovski, O. V. / Verkhovski, S. J.Computerisation of information processes and developmentof selective patent information dissemination systems.WPI, 1, 1984, 1823.

Kolle, G.Der Rechtsschutz der Computersoftware in der Bundes-republik Deutschland.GRUR, 1982, 443.

Martin, J. P.L´interprétation des revendications de brevet et la sécuritédes tiers.Gazette du Palais, 32/33, 1984, 5 7.

Mathews, J. A.Document Delivery Systems for Today and Tomorrow.APLA, 11, 1983, 102 106.

Matsui, S.Use invention of a chemical substance.WPI, 1, 1984, 12 17.

Minvielle-Devaux, I. deThe patenting of additional medical indications.CIPA, 4, 1984, 120 123.

Rahn, G.

The role of industrial property in economic development: theJapanese Experience.IIC, 14, 1983, 449 492.

Ryan, D. J.

Limitation de la responsabilité des mandataires en propriétéindustrielle / Limiting the liability of industrial propertyagents.Prop. Ind., 9, 1983, 294 300.Ind. Prop., 9, 1983, 273 278.

Schwanhäusser, H.Der Schutzumfang von Patenten nach neuem Recht. EineZwischenbilanz.Mitt., 1982, 186.

Schwendy, K.

The Bundespatentgericht of the Federal Republic ofGermany.EIPR, 9, 1983, 243 249.

Schweyer, B.

Zuständigkeit internationaler Schiedsgerichte zur Ent-scheidung über Fragen der Nichtigkeit und Verletzung vonPatenten.GRUR Int., 1983, 149 156.

Singer, R.

Developments in the case law of the EPO concerningEuropean and Euro-PCT patent applications.Ind. Prop., 12, 1983, 387 401.

Page 41: 22. November 1984 22 November 1984 22 novembre 1984 ...archive.epo.org/epo/pubs/oj1984/p549_604.pdf · tout Etat contractant doit, pour ce qui le concerne, excuser pour des motifs

Stern, R. H.

The legal protection of computer software and computer-related innovations in the United States.Ind. Prop., 1982, 152.

Straus, J.Die international-privatrechtliche Beurteilung von Arbeit-nehmererfindungen im europäischen Patentrecht.GRUR Int., 1, 1984, 1 7.

van Benthem, J. B. / Wallace, N. W. P.The problem of assessing inventive step in the Europearpatent procedure.IIC, 4, 1978, 297 308.van Benthem, J. B.

The European patent system, today and in the future.EIPR, 7, 1984, 182 188.

von Falck, K.

Freiheit und Bindung des Patentverletzungsrichters.GRUR, 1984, 392 397.

Page 42: 22. November 1984 22 November 1984 22 novembre 1984 ...archive.epo.org/epo/pubs/oj1984/p549_604.pdf · tout Etat contractant doit, pour ce qui le concerne, excuser pour des motifs

GEBÜHRENHinweise für die Zahlung vonGebühren, Auslagen und Ver-kaufspreisen*WICHTIGER HINWEIS: Ab 3. Januar1985 gelten neue Gebührenbeträge(siehe ABI. 7/1984, S. 297). Das Ge-bührenverzeichnis des EPA in der ab3. Januar 1985 geltenden Fassungist als Beilage zum ABI. 11/1984erschienen.1. Geltende Fassung der Gebühren-ordnung und des Gebührenverzeich-nissesDie ab 1. November 1981 geltendeFassung der Gebührenordnung ist inder Textausgabe Europäisches Patent-übereinkommen (2. Auflage Juni 1981)sowie im Amtsblatt 9/1981, S. 419 abge-druckt. Auf die Vorschriften der Artikel5 bis 8 GebO wird besonders hinge-wiesen.

Das derzeit geltende vollständige Ver-zeichnis der Gebühren, Auslagen undVerkaufspreise des EPA ist in der Beilagezum ABI. 8/1984 veröffentlicht.

2. Wichtige Ausführungsvorschriftenzur GebührenordnungNeue Beträge der in Artikel 2 der Ge-bührenordnung vorgesehenen Gebühren:ABI. 7/1981, S. 201.Gegenwerte der Gebühren, Auslagenund Verkaufspreise in den Währungenbestimmter Vertragsstaaten: Beilagezum ABI. 8/1984.

Vorschriften über das laufende Konto:ABI. 1/1982, S. 15 (vgl. auch ABI.7/1984, S. 321).

3. Zahlung und Rückerstattung vonGebühren und Auslagen

Es wird empfohlen, bei jeder Zahlungdas EPA-Formblatt für die Zahlung vonGebühren oder Auslagen (Form 1010)zu verwenden, das beim EPA undden Patentbehörden der Vertragsstaatenkostenlos erhältlich ist. Es ist fernerim Leitfaden für Anmelder abgedrucktund kann daraus kopiert werden.

Zur Zahlung von Gebühren berechtigtePersonen: ABI. 9/1980, S. 303.Rückerstattung der europäischen Re-cherchengebühr: ABI. 5/1980, S. 112;5/1983, S. 189; 6/1983, S. 219/238.

Teilweise Rückerstattung des Pauschal-preises für eine Standardrecherche zueiner türkischen Patentanmeldung: ABI.2/1983, S. 62.

Zahlung und Rückerstattung der Prüfungs-gebühr: ABI. 2/1979, S. 61; 9/1981,S.349.

Zahlung von Jahresgebühren für euro-päische Patentanmeldungen und Pa-tente: ABI. 5/1982, S. 199; 6/1984, S.272.

FEES

Guidance for the payment offees, costs and prices*

IMPORTANT NOTICE: From 3 January1985 new amounts of fees are appli-

cable (see OJ 7/1984, p. 297). The EPO´schedule of fees in force as of 3January 1985 has been published as asupplement to OJ 11/1984.

1. Effective version of the Rules re-lating to Fees and the fees schedule

The Rules relating to Fees in the versioneffective as from 1 November 1981 arereproduced in the 2nd edition (June1981) of the European Patent Conven-tion and in OJ 9/1981, p. 419. Attentionis drawn in particular to Articles 5 to 8of the Rules relating to Fees.

The full schedule of fees, costs andprices of the EPO now in force ispublished in the supplement to OJ8/1984.

2. Important implementing rules tothe Rules relating to FeesNew amounts of the fees provided for inArticle 2 of the Rules relating to Fees: OJ7/1981, p.201.Equivalents of the fees, costs and pricesin the currencies of some ContractingStates: supplement to OJ 8/1984.

Arrangements for deposit accounts: OJ1/1982, p. 15 (see also OJ 7/1984,p. 321).

3. Payment and refund of fees andcostsIt is recommended that the EPO form forthe settlement of fees or costs (Form1010), obtainable free of charge fromthe EPO and the patent authorities of theContracting States, be used for eachpayment. It is also reproduced in andcan be copied from the Guide forapplicants.

Persons entitled to make payment offees: OJ 9/1980, p. 303.Refund of the European search fee: OJ5/1980, p. 112; 5/1983, p. 189;6/1983, p. 219/238.Partial refund of the contractual fee paidfor a standard search on a Turkish patentapplication: OJ 2/1983, p. 62.

Payment and refund of the examinationfee: OJ 2/1979, p. 61: 9/1981, p. 349.

Paying renewal fees for European patentapplications and patents: OJ 5/1982, p.199; 6/1984, p.272.

TAXESAvis concernant le paiementdes taxes, frais et tarifs devente*

AVIS IMPORTANT: des taxes d´un nou-veau montant seront applicables àcompter du 3 janvier 1985 (voir n°7/1984, p. 297). Le barème des taxesde l´OEB en vigueur à compter du 3janvier 1985 a été publié en supplé-ment au JO n° 11/1984.1. Version applicable du règlementrelatif aux taxes et du barème destaxesLe texte du règlement relatif auxtaxes dans sa version applicable àcompter du 1er novembre 1981 figuredans la 2e édition (juin 1981) de laConvention sur le brevet européen demême que dans le JO n° 9/1981, p.419. L´attention est tout particulière-ment attirée sur les dispositions desarticles 5 à 8 du règlement relatif auxtaxes.Le barème complet actuellement envigueur des taxes, frais et tarifs devente de l´OEB figure dans le supplé-ment au JO n° 8/1984.

2. Mesures d´application importantesdu règlement relatif aux taxesNouveaux montants des taxes prévuesà l´article 2 du règlement relatif auxtaxes: JO n° 7/1981, p. 201.Contre-valeurs des taxes, frais et tarifsde vente dans les monnaies de certainsEtats contractants: supplément au JOn° 8/1984.

Réglementation applicable aux comptescourants: JO n° 1/1982, p. 15 (voirégalement JO n° 7/1984. p. 321).

3. Règlement et remboursement destaxes et frais

Il est recommandé d´utiliser pour toutpaiement le bordereau de règlementde taxes ou de frais (OEB Form 1010)qui peut être obtenu gratuitement auprèsde l´OEB et des services centraux de lapropriété industrielle des Etats con-tractants. Ce bordereau est par ailleursreproduit dans le Guide du déposantet peut servir d´original à une copie.

Personnes habilitées à acquitter lestaxes: JO n° 9/1980, p. 303.

Remboursement de la taxe de rechercheeuropéenne: JO nos 5/1980. p. 112;5/1983, p. 189; 6/1983, p. 219 et 238.Remboursement partiel de la redevanceforfaitaire acquittée au titre d´unerecherche standard relative à une de-mande de brevet turque: JO n° 2/1983,p. 62.

Paiement et remboursement de la taxed´examen: JO nos 2/1979, p. 61 et9/1981, p. 349.Paiement des taxes annuelles pour lesdemandes de brevet européen et pourles brevets européens: JO nos 5/1982,.p. 199; 6/1984, p. 272.

* Änderungen und Ergänzungen gegenüber dem imvorangehenden Amtsblatt veröffentlichten Ge-bührenhinweis sind durch eine senkrechte Liniegekennzeichnet.

* Amendments and additions to the fees guidancepublished in the previous Official Journal are in-dicated by a vertical line alongside

* Les modifications et les compléments apportéspar rapport à l´avis concernant les taxes publié auJournal officiel précédent sont indiqués par untrait vertical

Page 43: 22. November 1984 22 November 1984 22 novembre 1984 ...archive.epo.org/epo/pubs/oj1984/p549_604.pdf · tout Etat contractant doit, pour ce qui le concerne, excuser pour des motifs

Zweites Bankkonto der EPO in Frank-reich: ABI. 10/1984, S. 495,

Das Verzeichnis der für die EPO eröffnetenBank- und Postscheckkonten sowie derentsprechenden Zahlungswährungen istauf der dritten Umschlagseite diesesHeftes abgedruckt.

4. Sonstige Mitteilungen über Gebührenund VerkaufspreiseGebührenpflichtige Patentansprüche beiAnmeldungen mit mehreren Anspruchs-reihen: ABI. 6-7/1979, S. 292; 3/1981,S. 68.

Veröffentlichungen des EuropäischenPatentamts: Informationsblatt des EPA,4. Auflage Mai 1983 (kostenlos beimEPA erhältlich).

5. Hinweise für Euro-PCT-Anmel-dungen (EPA als Bestimmungsamtbzw. ausgewähltes Amt)

Fristen und Verfahrenshandlungen vordem EPA als Bestimmungsamt bzw.ausgewähltem Amt: ABI. 3/1979, S.110; 11-12/1979, S. 479; 9/1980, S.313.Wegfall der europäischen Recherchenge-bühr, wenn der internationale Recherchenbericht vom EPA, vom ÖsterreichischenPatentamt oder vom schwedischen Pa-tentamt erstellt wurde: ABI. 1/1979, S.4; 6-7/1979, S. 248.

Herabsetzung der europäischen Recher-chengebühr, wenn der internationaleRecherchenbericht vom Patent- undMarkenamt der Vereinigten Staaten,vom japanischen Patentamt, vom Staat-lichen Komitee der UdSSR für Erfin-dungen und Entdeckungen oder vomaustralischen Patentamt erstellt wurde:ABI. 9/1979, S. 368; 1/1981, S. 5.Rückzahlung der Prüfungsgebühr beiVerbindung einer europäischen Patent-anmeldung mit einer Euro-PCT-Anmel-dung: ABI. 9/1981, S. 349.

6. Hinweise für internationale Anmel-dungen (EPA als Anmeldeamt, Inter-nationale Recherchenbehörde oder alsmit der internationalen vorläufigenPrüfung beauftragte Behörde)

Fälligkeit und Zahlung der Gebührenfür internationale Anmeldungen: ABI.6-7/1979, S. 290; 10/1979, S. 427.Rückerstattung der internationalen Re-cherchengebühr: ABI. 5/1980, S. 112;5/1983, S. 189.

Second bank account for the EPO inFrance: OJ 10/1984, p. 495.

The list of bank and giro accountsopened in the name of the EuropeanPatent Organisation and correspondingcurrencies for payment is printed on theinside back cover of this issue.

4. Other Notices concerning fees andprices

Claims incurring fees where the applica-tion includes more than one set ofclaims: OJ 6-7/1979, p. 292; 3/1981,p. 68.

Publications of the European PatentOffice: European Patent Office Informa-tion brochure, 4th. edition, May 1983(available free of charge from the EPO).

5. Information concerning Euro-PCTapplications (EPO as designated orelected Office)

Deadlines and procedural steps beforethe EPO as a designated or electedOffice: OJ 3/1979, p. 110; 11-12/1979,p. 479; 9/1980, p. 313.

European search fee waived if the inter-national search report has been drawnup by the EPO, the Austrian PatentOffice or the Swedish Patent Office: OJ1/1979, p. 4; 2/1979, p. 50: 6-7/1979,p. 248.

Reduction of the European search fee ifthe international search report has beendrawn up by the United States Patentand Trademark Office, the JapanesePatent Office, the USSR State Com-mittee for Inventions and Discoveries, orthe Australian Patent Office: OJ 9/1979,p. 368; 1/1981, p.5.

Refund of the examination fee where aEuropean patent application is con-solidated with a Euro-PCT application:OJ 9/1981, p.349.

6. Guidance for international applica-tions (EPO as receiving Office, Inter-national Searching Authority or Inter-national Preliminary Examining Au-thority)

Time and airangements for payment ofthe fees for international applications:OJ 6-7/1979, p. 290; 10/1979, p. 427.

Refund of the international search fee:OJ 5/1980, p. 112; 5 /1983, p. 189.

Deuxième compte bancaire de l´OEB enFrance: JO n° 10/1984, p. 495.

La liste des comptes bancaires et dechèques postaux ouverts au nom del´Organisation européenne des brevetsainsi que des monnaies de paiementcorrespondantes est reproduite entroisième page de couverture de cenuméro.

4. Autres communications relatives auxtaxes et tarifs de venteRevendications donnant lieu au paie-ment de taxes lorsque la demande com-prend plusieurs séries de revendications:JO nos 6-7/1979, p. 292 et 3/1981, p.68.Publications de l´Office européen desbrevets: Brochure d´information del´Office européen des brevets, 4e édition,mai 1983 (peut être obtenue gratuite-ment auprès de l´OEB).

5. Avis concernant les demandeseuro-PCT (OEB en tant qu´officedésigné ou élu)Délais et actes de procédure effectuésdevant l´OEB agissant en qualité d´officedésigné ou élu: JO nos 3/1979, p. 110,11-12/1979, p. 479 et 9/1980, p. 313.

Suppression de la taxe de rechercheeuropéenne prévue, dans le cas où lerapport de recherche internationale aété établi par l´OEB, l´Office autrichiendes brevets ou l´Office suédois desbrevets: JO nos 1/1979, p. 4 et6-7/1979, p. 248.Réduction de la taxe de recherchedans le cas où le rapport de rechercheinternationale a été établi par l´Officedes brevets et des marques des Etats-Unis, l´Office japonais des brevets, leComité d´Etat de l´URSS pour lesinventions et les découvertes ou l´Officeaustralien des brevets: JO nos 9/1979,p. 368 et 1/1981, p. 5.Remboursement de la taxe d´examen encas de jonction d´une demande debrevet européen et d´une demande euro-PCT: JO n° 9 /1981, p. 349.

6. Avis concernant les demandes in-ternationales (OEB en tant qu´officerécepteur, administration chargée dela recherche internationale ou admi-nistration chargée de l´examen pré-liminaire international)Exigibilité et paiement des taxes pourles demandes internationales: JO nos

6-7/1979, p. 290 et 10/1979, p. 427.Remboursement de la taxe de rechercheinternationale: JO nos 5/1980, p. 112;5/1983, p. 189.

Ein Auszug aus der Gebührenordnung,die bei Einreichung einer europäischenPatentanmeldung oder einer interna-tionalen Anmeldung fällig werdendenGebühren sowie einige andere der amhäufigsten anfallenden Gebühren wer-den nachstehend wiedergegeben.

An extract from the Rules relating toFees, the fees falling due when a Euro-pean patent application or internationalapplication is filed and certain feesfrequently arising are set out below.

Un extrait du règlement relatif aux taxes,la liste des taxes exigibles lors du dépôtd´une demande de brevet européen oud´une demande internationale ainsi quela liste de quelques autres taxes le plusfréquemment appliquées sont repris´ci-après.

Page 44: 22. November 1984 22 November 1984 22 novembre 1984 ...archive.epo.org/epo/pubs/oj1984/p549_604.pdf · tout Etat contractant doit, pour ce qui le concerne, excuser pour des motifs

Auszug aus der Gebührenordnung

Artikel 5

Entrichtung der Gebühren

(1) Vorbehaltlich Artikel 6 sind die an das Amt zuzahlenden Gebühren in Deutschen Mark oder einerfrei in Deutsche Mark konvertierbaren Währungzu entrichten:

a) durch Einzahlung oder Überweisung auf einBankkonto des Amts,

b) durch Einzahlung oder Überweisung auf einPostscheckkonto des Amts,

c) durch Postanweisung.

d) durch Übergabe oder Übersendung von Schecks,die an die Order des Amts lauten, oder

e) durch Barzahlung.

12) Der Präsident des Amts kann zulassen. daß dieGebühren auf andere Art als in Absatz 1 vorgesehen entrichtet werden, insbesondere durchGebührenmarken oder mit Hilfe laufender Kontenbeim Amt.

Artikel 8

Maßgebender Zahlungstag

(1 ) Als Tag des Eingangs einer Zahlung beim Amtgilt:

a) im Fall des Artikels 5 Absatz 1 Buchstaben a undb der Tag, an dem der eingezahlte oder über-wiesene Betrag auf einem Bank- oder Postscheck-konto des Amts gutgeschrieben wird;

b) im Fall des Artikels 5 Absatz 1 Buchstaben c unde der Tag des Eingangs des Betrags der Postanweisung oder der Einzahlung des Bargeldbetragsoder der Tag. an dem der Betrag der Postan-weisung auf einem Postscheckkonto des Amts gut-geschrieben wird:

c) im Fall des Artikels 5 Absatz 1 Buchstabe d derTag an dem der Scheck beim Amt eingeht, soferndieser Scheck eingelöst wird.

(2)* Läßt der Präsident des Amts gemäß Artikel 5Absatz 2 zu, daß die Gebuhren auf andere Art alsin Artikel 5 Absatz 1 vorgesehen entrichtet wer-den, so bestimmt er auch den Tag. an dem dieseZahlung als eingegangen gilt

(3)* Gilt eine Gebuhrenzahlung gemäß den Ab-satzen 1 und 2 erst nach Ablauf der Frist als ein-gegangen. innerhalb der sie hätte erfolgen müssen,so gilt diese Frist als eingehalten, wenn dem Amtnachgewiesen wird. daß der Einzahler spätestenszehn Tage vor Ablauf der genannten Frist in einemVertragsstaat:

a) die Zahlung des Betrags bei einem Bankinstitutoder Postamt veranlaßt hat oder

bl einen Auftrag zur Überweisung des zu entrich-tenden Betrags einem Bankinstitut oder Postscheckamt formgerecht erteilt hat oder

c) dem Postamt einen an das Amt gerichteten Briefübergeben hat, in dem ein dem Artikel 5 Absatz 1Buchstabe d entsprechender Scheck enthalten ist.sofern dieser Scheck eingelöst wird.

Das Amt kann den Einzahler auffordern, innerhalbeiner vom Amt zu bestimmenden Frist diesenNachweis zu erbringen. Kommt der Einzahlerdieser Aufforderung nicht nach oder ist der Nach-weis ungenügend, so gilt die Zahlungsfrist alsversäumt.

Extract from the Rules relatingto Fees

Article 5

Payment of fees

(1) Subject to the provisions of Article 6 the feesdue to the Office shall be paid in Deutsche Mark orin a currency freely convertible into Deutsche Mark:

(a) by payment or transfer to a bank account heldby the Office

(b) by payment or transfer to a Giro account heldby the Office(c) by money order

(d) by delivery or remittance of cheques which aremade payable to the Office or

(e) by cash payment

(2) The President of the Office may allow othermethods of paying fees than those set out in para-graph 1 in particular by means of fees vouchers ordeposit accounts held with the European PatentOffice

Article 8

Date to be considered as the date on whichpayment is made

(1) The date on which any payment shall be con-sidered to have been made to the Office shall beas follows:

(a) in the cases referred to in Article 5, paragraph1(a) and (b); the date on which the amount of thepayment or of the transfer is entered in a bankaccount or a Giro account held by the Office:

(b) in the cases referred to in Article 5, paragraph1(c) and (e); the date of receipt of the amount ofthe money order or of the cash payment or thedate on which the amount of the money order isentered in a Giro account held by the Office;

(c) in the case referred to in Article 5, paragraph1(d): the date of receipt of the cheque at the Office,provided that the cheque is met.

(2)* Where the President of the Office allows, inaccordance with the provisions of Article 5, para-graph 2, other methods of paying fees than thoseset out in Article 5. paragraph 1 he shall also laydown the date on which such payments shall beconsidered to have been made

(3)* Where under the provisions of paragraphs 1and 2 payment of a fee is not considered to havebeen made until after the expiry of the period inwhich it should have been made it shall be con-sidered that this period has been observed ifevidence is provided to the Office that the personwho made the payment fulfilled one of the follow-ing conditions in a Contracting State not later thanten days before the expiry of such period:

(a) he effected the payment through a bankingestablishment or a post office

(b) he duly gave an order to a banking establishment or a post office to transfer the amount of thepayment;

(c) he dispatched at a post office a letter bearingthe address of the Office and containing a chequewithin the meaning of Article 5 paragraph 1(d),provided that the cheque is met.

The Office may request the person who made thepayment to produce such evidence within suchperiod as it may specify If he fails to comply withthis request or if the evidence is insufficient theperiod for payment shall be considered not to havebeen observed.

Extrait du règlement relatif auxtaxes

Article 5

Paiement des taxes

(1) Sous réserve des dispositions de l´article 6les taxes à payer à l´Office doivent être acquittéesen Deutsche Mark ou en monnaie librement con-vertible en Deutsche Mark:

a) par versement ou virement à un comptebancaire de l´Office,

b) par versement ou virement à un compte dechèques postaux de l´Office,

c) par mandat postal,

d) par remise ou envoi de chèques établis à l´ordrede l´Office ou

e) par paiement en espèces.

(2) Le Président de l´Office peut autoriser le paie-ment des taxes par d´autres moyens que ceuxprévus au paragraphe 1, notamment par timbresfiscaux et par comptes courants ouverts auprèsde l´Office.

Article 8

Date à laquelle le paiement est réputé effectué

(1) La date à laquelle tout paiement est réputéeffectué auprès de l´Office est fixée comme suit:

a) dans les cas visés à l´article 5, paragraphe 1,lettres a) et b): date à laquelle le montant du verse-ment ou du virement est porté au crédit d´uncompte bancaire ou d´un compte de chèquespostaux de l´Office;

b) dans les cas visés à l´article 5, paragraphe 1,lettres c) et e): date de l´encaissement du mon-tant du mandat postal ou des espèces, ou date àlaquelle le montant du mandat postal est porté aucrédit d´un compte de chèques postaux de l´Office;

c) dans le cas visé à l´article 5, paragraphe 1, lettred); date de réception du chèque par l´Office sousréserve de l´encaissement de ce chèque.

(2)* Lorsque le Président de l´Office autorise,conformément aux dispositions de l´article 5. para-graphe 2, le paiement des taxes par d´autresmoyens que ceux prévus au paragraphe 1 duditarticle, il fixe également la date à laquelle cepaiement est réputé effectué.

(3)* Lorsque, en vertu des dispositions des para-graphes 1 et 2, le paiement d une taxe n´est réputéeffectué qu´après l´expiration du délai dans lequelil aurait dû intervenir, ce délai est considérécomme respecté si la preuve est apportée àl´Office que la personne qui a effectué le paiementdans un Etat contractant a rempli au plus tard dixjours avant l´expiration dudit délai l´une des con-ditions ci-après:

a) avoir effectué le paiement auprès d´un établisse-ment bancaire ou d´un bureau de poste:

b) avoir donné un ordre de virement, en bonneet due forme, du montant du paiement à un éta-blissement bancaire ou à un bureau de chèquespostaux:

ci avoir déposé dans un bureau de poste unelettre portant adresse du siège de l´Office etcontenant un chèque visé à l´article 5, paragraphe1 lettre d), sous réserve de l´encaissement de cechéque.

L´Office peut inviter la personne qui a effectuéle paiement à en apporter la preuve, dans ledélai qu´il détermine. S il n´est pas donné suiteà cette invitation ou si la preuve apportée n´estpas suffisante, le délai de paiement est considérécomme n´ayant pas été respecté

* Der Zahlungstag bei Abbuchung von einem laufendenKonto bestimmt sich nach Nr 6.3 bis 6.10 der Vor-schriften über das laufende Konto (ABI. 1/1982, S. 16).

* The date of payment when debiting a deposit accountis determined by points 6.3 to 6.10 of the Arrangements for deposit accounts IOJ 1 1982 p 16).

´ La date de paiement est déterminée, en cas de débitd´un compte courant selon les points 6.3 à 6.10 de laréglementation applicable aux comptes courants (JO

n° 1 1982 p 16).

´

Page 45: 22. November 1984 22 November 1984 22 novembre 1984 ...archive.epo.org/epo/pubs/oj1984/p549_604.pdf · tout Etat contractant doit, pour ce qui le concerne, excuser pour des motifs
Page 46: 22. November 1984 22 November 1984 22 novembre 1984 ...archive.epo.org/epo/pubs/oj1984/p549_604.pdf · tout Etat contractant doit, pour ce qui le concerne, excuser pour des motifs
Page 47: 22. November 1984 22 November 1984 22 novembre 1984 ...archive.epo.org/epo/pubs/oj1984/p549_604.pdf · tout Etat contractant doit, pour ce qui le concerne, excuser pour des motifs
Page 48: 22. November 1984 22 November 1984 22 novembre 1984 ...archive.epo.org/epo/pubs/oj1984/p549_604.pdf · tout Etat contractant doit, pour ce qui le concerne, excuser pour des motifs
Page 49: 22. November 1984 22 November 1984 22 novembre 1984 ...archive.epo.org/epo/pubs/oj1984/p549_604.pdf · tout Etat contractant doit, pour ce qui le concerne, excuser pour des motifs
Page 50: 22. November 1984 22 November 1984 22 novembre 1984 ...archive.epo.org/epo/pubs/oj1984/p549_604.pdf · tout Etat contractant doit, pour ce qui le concerne, excuser pour des motifs
Page 51: 22. November 1984 22 November 1984 22 novembre 1984 ...archive.epo.org/epo/pubs/oj1984/p549_604.pdf · tout Etat contractant doit, pour ce qui le concerne, excuser pour des motifs
Page 52: 22. November 1984 22 November 1984 22 novembre 1984 ...archive.epo.org/epo/pubs/oj1984/p549_604.pdf · tout Etat contractant doit, pour ce qui le concerne, excuser pour des motifs
Page 53: 22. November 1984 22 November 1984 22 novembre 1984 ...archive.epo.org/epo/pubs/oj1984/p549_604.pdf · tout Etat contractant doit, pour ce qui le concerne, excuser pour des motifs
Page 54: 22. November 1984 22 November 1984 22 novembre 1984 ...archive.epo.org/epo/pubs/oj1984/p549_604.pdf · tout Etat contractant doit, pour ce qui le concerne, excuser pour des motifs
Page 55: 22. November 1984 22 November 1984 22 novembre 1984 ...archive.epo.org/epo/pubs/oj1984/p549_604.pdf · tout Etat contractant doit, pour ce qui le concerne, excuser pour des motifs

Amtsblatt desEuropäischenPatentamts

23. November 1984

BEILAGE ZUMAMTSBLATT 11/1984

Official Journalof the EuropeanPatent Office

23 November 1984

SUPPLEMENT TO OFFICIALJOURNAL 11/1984

Journal officielde l´Officeeuropéen desbrevets

23 novembre 1984

SUPPLEMENT AU JOURNALOFFICIEL n° 11/1984

GEBÜHREN

Beschluß des Präsidenten desEuropäischen Patentamts vom8. November 1984 über dieNeufestsetzung derVerwaltungsgebühren,Auslagen und Verkaufspreisedes EPA und der Gegenwerteder Gebühren, Differenzbeträgefür Jahresgebühren, Auslagenund Verkaufspreise in denWährungen der Vertragsstaaten

Artikel 1(1.) Gemäß Artikel 3 Absätze 1 und 2 derGebührenordnung werden die in derAusführungsordnung zum EuropäischenPatentübereinkommen genannten Ver-waltungsgebühren und die Gebührenund Auslagen für andere als in Artikel 2der Gebührenordnung genannte Amts-handlungen sowie die Verkaufspreise fürVeröffentlichungen in Deutschen Markwie im nachstehenden Gebührenver-zeichnis angegeben neu festgesetzt.

(2.) Gemäß Artikel 6 Absatz 4 der Ge-bührenordnung werden die Gegenwerte

a) der in Artikel 2 der Gebührenordnungvorgesehenen Gebühren, die durch Be-schluß des Verwaltungsrats der Euro-päischen Patentorganisation vom 8. Juni1984 (Amtsblatt 7/1984, S. 297) mitWirkung vom 3. Januar 1985 neu fest-gesetzt worden sind,

b) der Differenzbeträge zwischen denvor dem 3. Januar 1985 und den abdiesem Zeitpunkt geltenden Sätzen derJahresgebühren für europäische Patent-anmeldungen für das 3. bis 8. Jahr,gerechnet vom Anmeldetag an1),

FEES

Decision of the President ofthe European Patent Officedated 8 November 1984revising the amounts of theEPO administrative fees, costsand prices and the equivalentsof fees, reneweal feedifferences, costs and pricesin the currencies of theContracting States

Article 1

(1.) In accordance with Article 3 (1) and(2) of the Rules relating to Fees, theamounts of the administrative feesprovided for in the ImplementingRegulations to the European PatentConvention and the amounts of the feesand costs for any services rendered bythe Office other than those specifiedin Article 2 of the Rules relating toFees; and the prices of publications,shall be laid down anew in DeutscheMark as shown in the fees schedulebelow.

(2.) In accordance with Article 6 (4) ofthe Rules relating to Fees, the equiva-lents

a) of the fees provided for in Article 2of the Rules relating to Fees, whichhave been revised with effect from3 January 1985 by decision of theAdministrative Council of the Euro-pean Patent Organisation of 8 June1984 (Official Journal 7/1984, p. 297),

b) of the differences between therates applicable prior to and thoseapplicable with effect from 3 January1985 in respect of renewal fees forEuropean patent applications for the3rd to the 8th year, calculated fromthe date of application1),

TAXES

Décision du Président del´Office européen des brevets du8 novembre 1984 relative à larévision des montants des taxesd´administration, frais et tarifsde vente de l´OEB, ainsi que descontre-valeurs des taxes,différences entre les anciens etles nouveaux montants destaxes annuelles, frais et tarifsde vente dans les monnaies desEtats contractants

Article premier

(1.) Conformément à l´article 3, para-graphes 1 et 2 du règlement relatif auxtaxes, les montants en Deutsche Markdes taxes d´administration prévues dansle règlement d´exécution de la CBE et destaxes et frais à payer pour toute presta-tion de service assurée par l´Office, autreque celles visées à l´article 2 du règlementrelatif aux taxes, ainsi que les tarifs devente en Deutsche Mark applicables auxpublications, sont révisés comme in-diqué dans le barème des taxes figurantci-après.

(2.) Conformément à l´article 6, para-graphe 4 du règlement relatif aux taxes,les contre-valeurs

a) des taxes prévues à l´article 2 duditrèglement, qui ont été révisées aveceffet à compter du 3 janvier 1985 pardécision du Conseil d´administration del´Organisation européenne des brevets,en date du 8 juin 1984 (Journal officieln° 7/1984, page 297),

b) des différences entre le montantapplicable avant le 3 janvier 1985 et àcompter de cette date, des taxesannuelles pour les demandes de breveteuropéen pour la 3ème à la 8ème annéeà compter de la date de dépôt1).

1) Am 3. Januar 1985 tritt der Beschluß desVerwaltungsrats der Europäischen Patentorga-nisation vom 8. Juni 1984 über die Neufest-setzung der in Art. 2 der Gebührenordnung vor-gesehenen Gebühren und die Änderung derGebührenordnung (Amtsblatt 7/1984, S. 297)in Kraft. Die Jahresgebühren für das 3. bis 8.Jahr sind um DM 30, DM 40, DM 50, DM 60,

(Fortsetzung umseitig)

1) The decision of the Administrative Council of8 June 1984 revising the amounts of the feesprovided for in Article 2 of the Rules relatingto Fees and amending the Rules relating toFees (Official Journal 7/1984, p. 297) willenter into force on 3 January 1985. Therenewal fees for the 3rd to 8th years have beenraised by DM 30, DM 40, DM 50, DM 60,

(Continued over)

1) La décision du 8 juin 1984 du Conseil d´ad-ministration de l´Organisation européenne desbrevets portant révision du montant des taxesprévues à l´article 2 du règlement relatif auxtaxes et modification du règlement relatif auxtaxes (Journal officiel n° 7/1984, p. 297)entrera en vigueur le 3 janvier 1985. Les taxesannuelles pour la 3ème à la 8ème année ont

(Suite au verso)

Page 56: 22. November 1984 22 November 1984 22 novembre 1984 ...archive.epo.org/epo/pubs/oj1984/p549_604.pdf · tout Etat contractant doit, pour ce qui le concerne, excuser pour des motifs

c) der vor dem 3. Januar 1985 gelten-den Sätze der Jahresgebühren für euro-päische Patentanmeldungen für das3. bis 7. Jahr, gerechnet vom Anmel-detag an,d) der nach Absatz 1 neu festgesetztenVerwaltungsgebühren, Gebühren, Aus-lagen und Verkaufspreise

in anderen Währungen als in DeutschenMark, wie im nachstehenden Gebühren-verzeichnis angegeben, festgesetzt.

(3.) Das Gebührenverzeichnis ist Bestand-teil dieses Beschlusses.

Artikel 2(1.) Die nach Artikel 1 Absatz 1 fest-gesetzten neuen Beträge der Verwal-tungsgebühren, Gebühren, Auslagenund Verkaufspreise sowie die nach Arti-kel 1 Absatz 2 festgesetzten Gegen-werte sind für Zahlungen ab 3. Januar1985 verbindlich2).

(2.) Artikel 3 des Beschlusses des Ver-waltungsrats der Europäischen Patent-organisation vom 8. Juni 1984 (Amts-blatt 7/1984, S. 297) ist entsprechendanzuwenden.

Artikel 3

Dieser Beschluß tritt am 3. Januar 1985in Kraft.

Geschehen zu München am 8. Novem-ber 1984

J. B. van Benthem

c) of the rates applicable prior to3 January 1985 in respect of renewalfees for European patent applicationsfor the 3rd to the 7th year, calculatedfrom the date of application,d) of the administrative fees, fees,costs and prices revised in accordancewith paragraph 1

shall be laid down in currencies otherthan Deutsche Mark as shown in thefees schedule below.(3.) The fees schedule shall form part ofthis decision.

Article 2(1.) The new amounts of the adminis-trative fees, fees, costs and prices laiddown in accordance with Article 1 (1)and the equivalents laid down inaccordance with Article 1 (2) shall bebinding on payments made on or after3 January 1985 2).

(2.) Article 3 of the decision of theAdministrative Council of the Euro-pean Patent Organisation of 8 June1984 (Official Journal 7/1984, p. 297)shall apply mutatis mutandis.

Article 3This decision shall enter into force on3 January 1985.

Done at Munich on 8 November 1984.

J. B. van Benthem

c) des montants en vigueur avant le3 janvier 1985 des taxes annuelles pourles demandes de brevet européen pourla 3ème à la 7ème année à compter dela date de dépôt,d) des taxes d´administration, des taxes,frais et tarifs de vente révisés conformé-ment au paragraphe 1

sont fixées dans les monnaies autres quele Deutsche Mark comme indiqué dansle barème des taxes figurant ci-après.(3.) Le barème des taxes est partie inté-grante de la présente décision.

Article 2

(1.) Les nouveaux montants des taxesd´administration, taxes, frais et tarifs devente fixés conformément à l´articlepremier, paragraphe 1, ainsi que lescontre-valeurs fixées conformément àl´article premier, paragraphe 2, sontapplicables aux paiements effectués àcompter du 3 janvier 1985 2).(2.) L´article 3 de la décision du Conseild´administration de l´Organisation euro-péenne des brevets du 8 juin 1984(Journal officiel n° 7/1984, p. 297) estapplicable.

Article 3La présente décision entre en vigueurle 3 janvier 1985.

Fait à Munich le 8 novembre 1984.

J. B. van Benthem

1) Fortsetzung

DM 70 und DM 90 erhöht worden. Jahres-gebühren sind in Höhe des Satzes zu entrichten,der am Tag ihrer Fälligkeit gilt (Regel 37 (1)Satz 3 EPÜ). Maßgebend ist also nicht derZahlungstag, sondern der Fälligkeitstag derJahresgebühr (vgl. Regel 37111 Satz 1 EPÜ undArt. 2 des obengenannten Beschlusses des Ver-waltungsrats). Für Jahresgebühren, die vor dem3. Januar 1985 entrichtet, jedoch erst nachdiesem Zeitpunkt fällig geworden sind bzw. fälligwerden, muß daher der Differenzbetrag zwischendem alten und dem am Fälligkeitstag geltendenSatz nachgezahlt werden. Hierfür werden mitdiesem Beschluß Gegenwerte festgesetzt.

2) Maßgebender Zahlungstag ist der Tag, an demeine Zahlung gemäß den Vorschriften von Artikel8 (1) und (2) der Gebührenordnung als bei demEPA eingegangen gilt.

1) continued

DM 70 and DM 90. Renewal fees must bepaid in accordance with the rate in force onthe day on which they fall due (Rule 37 (1),1st sentence, EPC). What counts, therefore, isnot the day on which the renewal fee is paid,but the day on which it falls due (cf. Rule37 (1), 1st sentence, EPC and Article 2 of theabove-mentioned decision of the Adminis-trative Council). In the case of renewal feespaid before 3 January 1985 but not due untilafter that date, therefore, the differencebetween the old rate and that in force on theday on which they fall due must be paid inaddition. This decision lays down the relevantequivalents.

2) The ruling date of payment is that on whichany payment, in accordance with Article8 (1) and (2) of the Rules relating to Fees, isconsidered to have been made to the EPO.

1) Suiteété relevées respectivement de 30, 40, 50, 60,70 et 90 DM. Les taxes annuelles sont payées autaux en vigueur au jour de l´échéance (règle37 (1), troisième phrase de la CBE). Ce n´estdonc pas la date du paiement qui est détermi-nante, mais la date d´échéance de la taxeannuelle (cf. règle 37 (1), première phrase dela CBE et article 2 de la décision susmentionnéedu Conseil d´administration). En ce qui concerneles taxes annuelles qui ont été acquittées avantle 3 janvier 1985, mais qui ne sont venues oune viennent à échéance qu´après cette date, ladifférence entre l´ancien taux et le taux envigueur à la date de l´échéance doit donc fairel´objet d´un paiement complémentaire. Descontre-valeurs sont fixées à cette fin par laprésente décision.

2) La date décisive du paiement est la date àlaquelle tout paiement est réputé effectuéauprès de l´OEB selon l´article 8 (1) et (2) durèglement relatif aux taxes.

Präsident President Président

Page 57: 22. November 1984 22 November 1984 22 novembre 1984 ...archive.epo.org/epo/pubs/oj1984/p549_604.pdf · tout Etat contractant doit, pour ce qui le concerne, excuser pour des motifs
Page 58: 22. November 1984 22 November 1984 22 novembre 1984 ...archive.epo.org/epo/pubs/oj1984/p549_604.pdf · tout Etat contractant doit, pour ce qui le concerne, excuser pour des motifs
Page 59: 22. November 1984 22 November 1984 22 novembre 1984 ...archive.epo.org/epo/pubs/oj1984/p549_604.pdf · tout Etat contractant doit, pour ce qui le concerne, excuser pour des motifs
Page 60: 22. November 1984 22 November 1984 22 novembre 1984 ...archive.epo.org/epo/pubs/oj1984/p549_604.pdf · tout Etat contractant doit, pour ce qui le concerne, excuser pour des motifs
Page 61: 22. November 1984 22 November 1984 22 novembre 1984 ...archive.epo.org/epo/pubs/oj1984/p549_604.pdf · tout Etat contractant doit, pour ce qui le concerne, excuser pour des motifs
Page 62: 22. November 1984 22 November 1984 22 novembre 1984 ...archive.epo.org/epo/pubs/oj1984/p549_604.pdf · tout Etat contractant doit, pour ce qui le concerne, excuser pour des motifs
Page 63: 22. November 1984 22 November 1984 22 novembre 1984 ...archive.epo.org/epo/pubs/oj1984/p549_604.pdf · tout Etat contractant doit, pour ce qui le concerne, excuser pour des motifs
Page 64: 22. November 1984 22 November 1984 22 novembre 1984 ...archive.epo.org/epo/pubs/oj1984/p549_604.pdf · tout Etat contractant doit, pour ce qui le concerne, excuser pour des motifs
Page 65: 22. November 1984 22 November 1984 22 novembre 1984 ...archive.epo.org/epo/pubs/oj1984/p549_604.pdf · tout Etat contractant doit, pour ce qui le concerne, excuser pour des motifs
Page 66: 22. November 1984 22 November 1984 22 novembre 1984 ...archive.epo.org/epo/pubs/oj1984/p549_604.pdf · tout Etat contractant doit, pour ce qui le concerne, excuser pour des motifs
Page 67: 22. November 1984 22 November 1984 22 novembre 1984 ...archive.epo.org/epo/pubs/oj1984/p549_604.pdf · tout Etat contractant doit, pour ce qui le concerne, excuser pour des motifs
Page 68: 22. November 1984 22 November 1984 22 novembre 1984 ...archive.epo.org/epo/pubs/oj1984/p549_604.pdf · tout Etat contractant doit, pour ce qui le concerne, excuser pour des motifs
Page 69: 22. November 1984 22 November 1984 22 novembre 1984 ...archive.epo.org/epo/pubs/oj1984/p549_604.pdf · tout Etat contractant doit, pour ce qui le concerne, excuser pour des motifs
Page 70: 22. November 1984 22 November 1984 22 novembre 1984 ...archive.epo.org/epo/pubs/oj1984/p549_604.pdf · tout Etat contractant doit, pour ce qui le concerne, excuser pour des motifs
Page 71: 22. November 1984 22 November 1984 22 novembre 1984 ...archive.epo.org/epo/pubs/oj1984/p549_604.pdf · tout Etat contractant doit, pour ce qui le concerne, excuser pour des motifs